Old Q&A – Respiratory System

Old Q&A – Respiratory System

A 65-year-old man, his wife, and 38-year-old son have been your clinic patients for the last 15 years. In the evaluation of some mild hemoptysis of the 65-year-old man, a chest x-ray reveals a 4 cm right sided lung mass, hilar and mediastinal adenopathy, and several lytic lesions in his ribs and humerus. None of these findings were present on an x-ray performed 4 years earlier. He has a 50-pack year smoking history. When he returns to your office, you inform him that he likely has stage IV lung cancer and that you would like to refer him to an oncologist for further evaluation. He states that he wants no therapy whatsoever, and that he wants to keep this a secret from his family. The most appropriate response would be to

  A. call his son as soon as he leaves the office
  B. inform him that treatment will likely be curative and that he should really reconsider his decision
  C. investigate what it is that makes him feel uncomfortable in telling his family and provide counseling
  D. realize that he will likely “come to his senses” and give him a referral to the oncologist anyway
  E. tell him that he is probably just in denial and try to persuade him to tell his wife when he gets home
Explanation:

The correct answer is C. Patient confidentiality is one of the most important medical ethical issues facing physicians, and it certainly can pose dilemmas at times. This patient has just received horrible news and is likely just reacting without really thinking about the ramifications of his decision. However, there may be very important personal, social, or cultural reasons for his decision. It is important for you, as a physician, to explore these with him.

Calling his son (choice A) is inappropriate because it breaks confidentiality.

Although you will likely try to get the patient to reconsider his decision (choice B) telling him that therapy will likely be curative for stage IV lung cancer is not true. There is very little chance at a cure and palliative therapy is a much more reasonable expectation.

Giving him a referral to the oncologist because he is will “come to his senses” (choice D) is inappropriate. He obviously needs counseling, and the feelings as to why he does not want treatment and why he does not want his family to know, should be explored.

Although the patient may be in denial (choice E), patient confidentiality precludes you from unilaterally deciding to tell his wife. It is appropriate to try to understand the reasons why he does not want to tell his family, as opposed to trying to persuade him to tell his wife when he gets home.

 

A 24-year-old woman comes to the office because of a cough with “yellowish sputum production” for the past 2 days. She states that the cough has been keeping her up at night and it is bothering her co-workers. They insisted that she “go get medicine” so that she does not “infect the entire office.” She has no history of respiratory disease. Her temperature is 37 C (98.6 F), blood pressure is 110/80 mm Hg, pulse is 70/min, and respirations are 18/min. Physical examination is normal. The most appropriate next step in management is to

  A. admit her to the hospital for medical management
  B. obtain a sputum culture
  C. order a chest x-ray
  D. prescribe erythromycin, orally
  E. send her home with no medications
Explanation:

The correct answer is E. This patient most likely has acute bronchitis. Acute bronchitis in a healthy patient with no other medical conditions is often due to viral infection that is usually self-limited. Given that this patient has only had 2 days of symptoms, an antibiotic is not necessary and is inappropriate. If the symptoms persist for longer than 1 week, a macrolide antibiotic may be given. A chest x-ray and a sputum culture are not indicated.

Admission to the hospital for medical management (choice A) is inappropriate for a healthy patient with acute bronchitis.

A sputum culture (choice B) is used to identify organisms, but should only be used in an elderly patients with chronic disease that fail antibiotic therapy. This patient’s acute bronchitis is most likely due to a self-limited viral infection.

A chest x-ray (choice C) has no role in the diagnosis of acute bronchitis in a healthy patient.

Send the patient home with antibiotic therapy (choice D) is appropriate management for acute bronchitis in an elderly patient with chronic disease. A macrolide (erythromycin, azithromycin, clarithromycin) is the treatment of choice. It is not part of the initial treatment in a previously healthy patient.

 

 

A 45-year-old woman with severe reflux disease secondary to a hiatal hernia is admitted to the hospital with flank pain from a kidney stone. An abdominal CT shows multiple stones in the right ureter and renal pelvis. On the floor, she is given intramuscular meperidine every 4 hours for pain control. Early in the morning the patient is found to be obtunded in moderate respiratory distress with some evidence of vomitus on her lips and bed shirt. She had been given 3 additional doses of meperidine for pain control in the past 5 hours. A chest radiograph will most likely show a

  A. diffuse bilateral airspace disease
  B. diffuse bilateral interstitial infiltrates
  C. right lower lobe opacification
  D. right pleural effusion
  E. widened mediastinum
Explanation:

The correct answer is C. Aspiration of gastric contents causes severe lung inflammation. The traditional dogma that the acidic nature of the aspirate is critical has recently been reevaluated and it is now clear that large volumes of gastric contents of any pH are dangerous to the lung. Patients with severe reflux often regurgitate frequently throughout the day and at night will have small aspiration events, which will wake them from sleep by coughing. Once sedated, these people develop depressed cough reflexes and therefore are more likely to be unable to protect their airway during such regurgitations. This is most certainly what has occurred with this patient. The most common radiological finding is right lower lobe opacification (alveolar filling) or collapse.

Diffuse bilateral airspace disease (choice A) is characteristic of acute respiratory distress syndrome (ARDS) or very late stage aspiration which can lead to ARDS.

Diffuse bilateral interstitial infiltrates (choice B) are characteristic of pulmonary edema. This may be a late manifestation (a few days) of severe aspiration, but not an early one.

Pleural effusions (choice D) are not present in aspirations. A unilateral effusion can be found in cases of liver abscess or right sided diaphragmatic irritation or with Meigs syndrome (ovarian cancer and ipsilateral pleural effusion).

A widened mediastinum (choice E) is characteristic of an aortic arch dissection or of a pulmonary disease such as sarcoid.

 

A 78-year-old man who lives alone is brought to the emergency department by ambulance because of respiratory distress. According to the brief history obtained by the paramedics, he is having abdominal pain since the morning and reports a history of congestive heart failure, insulin dependent diabetes mellitus, hypertension, and peripheral vascular disease. On arrival to the hospital, he is very drowsy and his temperature is 36.7 C (98.0 F), pulse is 110/min and irregular, blood pressure is 90/54 mm Hg, respirations are 24/min, and oxygen saturation is 84%. He appears to be in great distress from his abdominal pain. Laboratory studies show:

After starting an intravenous catheter and administering a diuretic, you are getting ready to intubate the patient. The emergency department nurse conveys a message from the patient’s daughter in Florida that there is a living will written by the patient which mandates that under no circumstances should he be intubated, resuscitated by CPR or dependent on artificial ventilation or feeding. The nurse reports that the daughter was very emotional and adamant that the patient should just be made comfortable, and that she would sue if he was intubated or if CPR carried out. During that emotional conversation she forgot to leave her phone number. The most appropriate next step in management is to

  A. call a hospital administrator to make a decision
  B. call a hospital lawyer for advice
  C. intubate the patient
  D. respect the daughter’s wishes and keep the patient comfortable without intubation
  E. try and trace the daughter’s phone number and request a fax of the living will
Explanation:

The correct answer is C. The patient is in respiratory distress and needs intubation for airway control, better oxygenation, hemodynamic resuscitation, and to feel comfortable. Although every attempt should be made to respect the patient’s wishes and the family’s requests, in an emergency situation there is limited opportunity to check the validity of telephone messages and faxed documents. Medical emergency mandates appropriate action prior to legal concerns.

Calling the hospital administrator (choice A) and lawyer (choice B) are not advisable in an emergency situation for the reasons explained above.

A living will mandating “do not resuscitate or do not intubate” needs to be checked and certified by a hospital social worker or legal department for authenticity before implementation. In an emergency situation this is not practical. Even if this patient is intubated, once a valid living will is obtained the ventilator can be switched off. Hence, to respect the daughter’s wishes and keep the patient comfortable without intubation (choice D) is incorrect.

To try and trace the daughter’s phone number and request a fax of the living will (choice E) is not practical in an emergency situation, and the validity of the documents is questionable without being checked by hospital authorities.

 

A 29-year-old man is admitted to the hospital with fever and cough. The symptoms began roughly 1-month prior and have been intermittent. He states that his cough is often productive of thick secretions and that, despite normal food intake, he has lost about 10 pounds in the past month. He is a volunteer at a local hospital and has received no special health care personnel vaccinations or screening tests. On examination, the patient appears somewhat thin, tired, and is coughing intermittently. His temperature is 38.0 C (100.4 F) and respirations are 16/min. He has patchy bilateral rhonchi over all lung fields. Prior to initiating therapy for this condition, the laboratory test required to confirm the suspected diagnosis is a

  A. chest radiograph
  B. sputum acid-fast stain
  C. sputum culture
  D. sputum Gram stain
  E. tuberculin skin test
Explanation:

The correct answer is B. The patient likely has tuberculosis. Virtually all M. tuberculosis is transmitted by airborne particles that are 1 to 5 µm in diameter. The symptoms of tuberculosis are protean and nonspecific and can be classified as either systemic or organ-specific. Classic systemic symptoms include fever, night sweats, anorexia, weight loss, and weakness. However, since tuberculosis is associated with other illnesses that have similar symptoms, this lack of specificity can result in a delayed diagnosis or even a misdiagnosis. Organ-specific symptoms of pulmonary tuberculosis include cough, pleuritic pain, and hemoptysis. The requirement for diagnosis is the presence of the organism that appears by acid-fast staining in a sputum sample.

In patients with primary tuberculosis, chest radiographs (choice A) often show infiltrates in the middle or lower lung zones, with ipsilateral hilar adenopathy. These findings are non-specific and are not used for confirmation of the diagnosis.

A sputum culture (choice C) is not useful in this case since the organism responsible for TB is fastidious and is difficult to culture, and certainly does not grow rapidly.

The organism responsible for TB does not stain with traditional Gram stain dyes (choice D) and therefore requires special staining such as acid-fast in order to detect it.

Although it is imperfect, the gold standard for diagnosing latent tuberculosis infection remains the intradermal injection (choice E) of purified protein derivative (5 TU) into the volar or dorsal surface of the forearm (Mantoux method). The test has no role in the diagnosis of active infection.

 

A 56-year-old man comes to the clinic for a pre-employment physical examination. He feels well and denies any health problems. Past medical history is negative except for an appendectomy about 20 years ago. The patient drinks several alcoholic beverages per day and smokes “a lot” of cigarettes. A “screening” chest x-ray, which you ordered because it is asked for on the employment forms, is shown below and demonstrates a left hilar mass and emphysema.

In considering the most appropriate next step in management, the most relevant question to ask this patient at this time is:

  A. “Are your affairs in order?”
  B. “Do you have any allergies?”
  C. “How many packs of cigarettes do you smoke per day?”
  D. “What are your thoughts on end of life care?”
  E. “Would you consent to a lung biopsy?”
  F. “Would you consider chemotherapy or radiation treatment for cancer?”
Explanation:

The correct answer is B. The chest x-ray demonstrates a right mid-lung nodule and emphysema. There is a lung nodule that is likely to be cancer in this patient with a smoking history and radiographic emphysema. The next step is a CT scan of the thorax with contrast, and before administering iodinated contrast, an allergy history must be elicited. Prior allergies to iodinated contrast material or shellfish will require further questioning. If the allergy is minor such as mild hives, pruritus, or flushing, a pre-medication regimen of prednisone and diphenhydramine is necessary. More serious allergies such as anaphylaxis preclude the administration of intravenous contrast.

Questions about death (choice A and D) are premature. This nodule may be an artifact, pneumonia, a granuloma, or cancer. Moreover, a localized cancer may be curable.

While quantification of the patient’s smoking (choice C) is necessary for a complete history, it does not change the management of this patient.

A lung biopsy (choice E) is premature. A CT scan and possibly PET scan are necessary to evaluate this lesion noninvasively before an invasive procedure is carried out.

Even though this nodule is likely to be cancer, questions about cancer treatment (choice F) are premature.

 

A 53-year-old widowed woman comes to the office for a health maintenance examination. She is a new patient who recently moved to your city after her husband died in an office fire 6 months ago. She says that she has no complaints, except for a cough that she began to notice 4 months ago. She denies nasal discharge, “a tickle in the throat,” frequent throat clearing, heartburn and the sensation of regurgitation, fever, sputum production, cigarette smoking, illegal drug use, sexual activity, occupational exposures, and any other symptoms associated with a respiratory infection. She says that the cough is not seasonal or associated with wheezing. Her temperature is 37.0 C (98.6 F), blood pressure is 135/90 mm Hg, pulse is 70/min, and respirations are 14/min. Physical examination is unremarkable. The most appropriate next step is to

  A. order an electrocardiogram
  B. order an x-ray of the chest
  C. question her about medications
  D. refer her for fiberoptic bronchoscopy
  E. schedule her for pulmonary function tests
Explanation:

The correct answer is C. This patient has a chronic cough, which is usually considered chronic because it is lasting more than 3 weeks. It may be due to a variety of things. However, the important lesson in this question is that before you turn to diagnostic studies you need, to make sure that you have obtained a detailed history. The case history will provide the answer to almost every question that you will need to ask her, except what medications she takes. Since she is a new patient, you will need to find out if she is taking an ACE inhibitor, such as captopril or enalapril, which is a frequent cause of a chronic cough in hypertensive patients. They cause a cough in up to 20% of people taking them. The exact mechanism is unknown, but it is thought to somehow be related to bradykinin and substance P. The treatment for the cough is the discontinuation of the ACE inhibitor.

An electrocardiogram (choice A) is unnecessary at this time in this patient, complaining of a chronic cough. She is not complaining of chest pain and there is nothing in her history that suggests an arrhythmia. The most important next step, is to take a detailed history before you order diagnostic tests.

An x-ray of the chest (choice B) may be appropriate in the near future, but it is not the next step at this time. Before you order diagnostic studies, you need to make sure that you ask her any questions that might help you figure out the etiology of her cough. Asking her about medications is very important because ACE inhibitors cause a chronic cough in up to 20% of patients taking this medication.

A fiberoptic bronchoscopy (choice D) is used to obtain histologic and cytologic specimens and to visualize an endobronchial tumor. Before you turn to such a specialized study, you need to first obtain a detailed history. If the patient is not taking an ACE inhibitor, a chest x-ray should usually be performed, and if this is abnormal, sputum cytology, a high resolution CT scan, and fiberoptic bronchoscopy should be considered.

Pulmonary function tests (choice E) are used to assess airway hyperresponsiveness for patients in which you suspect asthma, and lung volumes and diffusion capacity in patients in which you suspect a diffuse interstitial lung disease. A detailed history is necessary before using any of these studies.

 

You are called to see a 75-year-old man who has metastatic lung cancer because of hypoxia, hypotension, and mental status changes. He has been your patient for many years and he has told you multiple times that he does not want to be placed on a respirator for any reason. On multiple occasions after his wife died, he has explained to you that if he was ever in a situation where mechanical respiration or any heroic measures should become necessary, that he would prefer to simply be made comfortable and be “allowed to go.” He has a living will, which states that if he was in a terminal condition he does not want any life sustaining treatments including hemodialysis, intubation, and cardiac resuscitation. Rather, he wants comfort care only. His two daughters and three sons are all present in the room with you. They explain to you that they are very upset by their father’s condition and that they want to place him on a ventilator to help him get through this episode. You explain that their father did not want aggressive medical care at the end of life, but the family insists on intervention. They threaten to sue you for malpractice if he dies. His temperature is 37.0 C (98.6 F), blood pressure is 75/40 mm Hg, pulse is 130/min, and respirations are 29/min. Physical examination shows a cachetic man in moderate respiratory distress. He is extremely disoriented and agitated and appears to be in pain. The most appropriate course of action at this time is to

  A. ask the family to leave and inject a lethal dose of morphine in accordance with the patients wishes
  B. do not intubate patient but administer morphine and dopamine together to alleviate his suffering while maintaining his blood pressure
  C. do not intubate the patient but administer morphine for comfort even though this might lower his blood pressure and respiratory rate and hasten the patient’s demise
  D. contact the hospital ethics committee to decide on the proper course of action
  E. intubate the patient for now; when the situation is more stable discuss the patient’s end of life wishes with the family
Explanation:

The correct answer is C. This patient has a living will, which states that if he was in a terminal condition he wants only comfort care. He has also clearly stated his wishes to you in the past. Intubation of this patient is clearly against his wishes. Starting morphine may cause respiratory suppression or worsen his hypotension which might hasten death. The potential to hasten death is an acceptable risk if the primary intention is to decrease patient suffering. Ideally, the patient’s wishes should be clearly explained early in his hospital course.

Injection of a “lethal dose” of morphine (choice A) is not acceptable management of this patient since its only purpose would be to hasten death. As stated above, it is acceptable to use a drug that may hasten death if the primary intention is to decrease suffering. Asking the family to leave and then injecting a lethal dose of medication is clearly not acceptable management.

Any time you start morphine on a patient you need to be aware of the possibility of worsening hypotension. Morphine is a mild vasodilator and therefore acts as a preload reducer. In this patient, hypotension already is prominent prior to starting morphine. Starting dopamine with the morphine (choice B) is not appropriate in this case since most physicians agree that, in this patient, starting vasopressors would constitute “heroic” measures.

Contacting the hospital ethics committee (choice D) is not appropriate in this patient. This patient needs immediate medical attention. There is no time for meetings now.

Intubation of this patient now (choice E) is clearly against the patient’s documented wishes. To ignore a patient living’s will and end of life issues is clearly wrong and would make completion of advance directives meaningless.

 

A 53-year-old woman who is a heavy smoker presents to the emergency department complaining of increasing shortness of breath for the past 3 days. She denies any history of asthma or coronary artery disease. Her temperature is 37.3 C (99.2 F), blood pressure is 150/90 mm Hg, heart rate is 110/min, and respiratory rate is 34/min. On examination, she is awake, alert, and oriented. Diffuse bilateral wheezes are heard on lung auscultation. Pulse oximetry measures 90% oxygen saturation on room air. An arterial blood gas is drawn and the results show:

A chest radiograph demonstrates bilateral, hyperinflated lungs with a flattened diaphragm. Sputum Gram stain shows a few polymorphonuclear cells, moderate number of epithelial cells, and a moderate number of Gram-positive cocci. She receives supplemental oxygen, albuterol nebulizer treatments, and steroids. Her symptoms improve and pulse oximetry now reads 93% saturation. The most appropriate next step is to

  A. add antibiotics to the treatment regimen
  B. do diffusion capacity testing by carbon monoxide
  C. intubate and begin mechanical ventilation
  D. obtain lung spirometry measurements
  E. start non-invasive positive pressure ventilation
Explanation:

The correct answer is A. The patient is a smoker who presented with progressive shortness of breath. Physical examination found diffuse wheezing and chest X-ray noted emphysema. In addition, she had an elevated pCO2 with acute respiratory acidosis and moderate hypoxia. These findings are consistent with an exacerbation of chronic obstructive lung disease. Such flares are treated with β2-agonists, anticholinergics, and steroids. In addition, antibiotics have also been shown to improve clinical outcome, and so they are part of the treatment regimen for chronic obstructive lung disease flares.

Diffusion capacity (choice B) for this patient will likely be low given her emphysema and is an important measurement for diagnosis, but it is not required in the acute management of this condition.

The patient has a normal mental status and is able to protect her airway. Her symptoms and oxygenation also improve with treatment. Thus, there is no current indication for intubation (choice C). Intubation is required if the patient has severe CO2 retention and/or hypoxia refractory to medical therapy. Intubation is also indicated if her condition is refractory to non-invasive ventilation, if she has severe acid-base disturbances, or if there is any change in her mental status that would compromise the airway.

Lung spirometry (choice D) will aid in the diagnosis of her disease but is not useful in management of her clinical course.

Non-invasive positive pressure ventilation (choice E) is indicated in patients with severe chronic obstructive pulmonary disease that is refractory to medical therapy. It is also useful in patients with increasing respiratory fatigue. A patient must be able to initiate breathing and tolerate the breathing mask. This patient has a normal mental status and her symptoms improve with treatment. Thus, she currently does not require any assistance in ventilation.

 

An 8-year-old boy is brought to the office by his mother because of recurrent episodes of “shortness of breath” and wheezing. These episodes typically occur when he is playing in the park with friends or when he is in the house at night. The symptoms are worst in the springtime and when he is watching television with his mother’s boyfriend. The mother’s boyfriend, who happens to smoke cigarettes, has been spending more and more time at the house, trying to bond with the patient. Pulmonary function tests show that the peak expiratory flow and forced respiratory volume per second are reduced during an attack and are normal during symptom-free intervals. Skin testing shows that he is allergic to grass and tree pollen, dust mites, animal dander, and a variety of other allergens. Laboratory studies show:

The most appropriate next step is to

  A. administer immunotherapy against identified allergens
  B. advise him to avoid all exercise
  C. advise him to try to avoid respiratory irritants, especially cigarette smoke
  D. advise the patient’s mother to use a humidifier and air cleaners at home
  E. prescribe inhaled sodium cromoglycate, oral corticosteroids, and oral theophylline
Explanation:

The correct answer is C. This patient has asthma, and the most crucial step in the management of asthma is avoidance of the triggering factors, e.g., allergens. Unfortunately, it is difficult to avoid specific types of allergens, such as pollens. Specific measures to eliminate or reduce exposure to dust mites and animal dander at home lead to a reduced frequency of attacks and hospitalization rates. Regardless of the allergens involved, elimination of respiratory irritants, especially cigarette smoke, is of crucial importance. The bronchial tree of asthmatic patients is highly reactive to any form of chemical or physical irritation. Thus the avoidance of passive smoke is important. The mother should ask her boyfriend to go smoke outside alone if he needs to, but he should not be allowed to smoke in the house.

It is not practical to administer immunotherapy against identified allergens (choice A) in this case because he is allergic to multiple airborne allergens, and it seems like he is especially responsive to cigarette smoke. Immunotherapy is of some benefit when a single allergen is identified. The most important step is to try to reduce exposure to avoidable allergens (smoke).

Avoidance of all exercise (choice B) is not appropriate because even though exercise triggers asthmatic attacks in some patients, this does not seem to be his main trigger.

Humidifiers and air cleaners (choice D) at home is not the appropriate management. Humidifiers favor the growth of dust mites, and air cleaners have not been shown to be uniformly effective in getting rid of dust mites.

It is inappropriate to prescribe inhaled sodium cromoglycate, oral corticosteroids, and oral theophylline (choice E) for this patient because the fewest number of drugs at the lowest effective doses should be used. Typically, a one drug regimen (a bronchodilator or an inhaled corticosteroid) for mild to moderate asthma or two drugs for more severe cases is sufficient to control asthma exacerbations. Oral corticosteroids are indicated in cases of severe asthma and are therefore, not for this patient.

 

A previously healthy 21-year-old college student comes to the clinic because of a headache, sore throat, muscle aches, and a constant, irritating, dry cough for six days. He says that he is “never sick” and has only been to this clinic for his “immunizations”. He exercises regularly, does not smoke cigarettes, and has an “occasional beer on the weekends with buddies.” His temperature is 38.8 C (101.8 F), blood pressure is 120/80 mm Hg, pulse is 68/min, and respirations are 16/min. Scattered rhonchi are heard in the left lower lobe. A chest x-ray shows diffuse interstitial infiltrates in the left lower lobe. A single dose of erythromycin therapy is given in the clinic. The most appropriate next step in management is to

  A. admit him to the hospital and begin administration of erythromycin, intravenously
  B. admit him to the hospital and begin administration of trimethoprim-sulfamethoxazole, intravenously
  C. admit him to the hospital for a cold agglutinin test
  D. give him a prescription for erythromycin and send him home
  E. recommend aspirin, fluids, and rest at home
Explanation:

The correct answer is D. This patient most likely has Mycoplasma pneumonia, which is a common cause of pneumonia in young adults and is typically treated with oral erythromycin as an outpatient. It is characterized by a dry cough, headache, myalgia, malaise, and fever. Physical examination is usually unremarkable except for diffuse rhonchi or fine rales. A chest x-ray shows diffuse interstitial or reticulonodular infiltrates, typically in the lower lobes. Given the patient’s age, history, physical examination, and chest x-ray findings, it is reasonable to assume that he has a community-acquired pneumonia. This is most likely due to Mycoplasma pneumoniae and empiric antimicrobial therapy with erythromycin should be prescribed. In these patients, a microbial diagnosis (with a sputum culture, transtracheal aspiration, bronchoscopy, or a blood culture) is often impractical and unnecessary. A cold agglutinin response is often associated with Mycoplasma pneumoniae. However, it is nonspecific and detected in less than 50% of cases.

Admitting him to the hospital and beginning administration of erythromycin, intravenously (choice A) is incorrect because a patient with Mycoplasma pneumonia, which is what this patient most likely has, is usually treated as an outpatient. The criteria for hospitalization of patients with pneumonia are ages >65, significant comorbidity, leukopenia, pneumonia due to Staphylococcus aureus, Gram-negative bacilli or anaerobes, suppurative complications, failure of outpatient management, inability to take oral medication, respirations >30/min, heart rate >140/min, hypotension, hypoxia, or acute alteration of mental status. The patient in this case does not meet any of these criteria.

Admitting him to the hospital and beginning administration of trimethoprim-sulfamethoxazole intravenously (choice B) is the management for patients with severe Pneumocystis carinii pneumonia (PCP), which is characterized by shortness of breath, a dry cough, fever, night sweats, rales or rhonchi, and bilateral patchy alveolar infiltrates. This is a common cause of pneumonia in immunocompromised patients, especially those with HIV and AIDS. The patient in this case does not appear to be immunocompromised, and he is not short of breath, which makes the diagnosis of PCP unlikely.

Admitting him to the hospital for a cold agglutinin test (choice C) is inappropriate because even though this patient most likely has Mycoplasma pneumonia, it can be treated with erythromycin as an outpatient, and a cold agglutinin test can be performed as an outpatient. A cold agglutinin response is often associated with Mycoplasma pneumoniae. However, it is nonspecific and detected in less than 50% of cases.

Recommending aspirin, fluids, and rest at home (choice E) is inappropriate treatment for this patient who most likely has Mycoplasma pneumonia, which needs to be treated with an antibiotic such as erythromycin.

 

A 23-year-old man with a childhood history of eczema presents to your office for the first time complaining of a non-productive coughthat started 4 months ago after a respiratory tract infection during the winter. He generally has the cough roughly once or twice a week, usually after strenuous exercise. He has not had any fevers at home and denies any hemoptysis. He smokes socially, roughly 1 pack a week, and binge drinks on the weekends. He denies any intravenous drug use, but has had several unprotected heterosexual relationships this past year. His temperature is 37.0 C (98.6 F), blood pressure is 110/80 mm Hg, pulse is 65/min, and respirations are 15/min. His physical examination is remarkable only for end expiratory wheezes on bilateral lower lung fields. At this time the most correct statement about his condition is:

  A. Bronchoscopy should be performed to rule out opportunistic infection
  B. A chest x-ray is necessary prior to starting any empiric therapy
  C. Inhaled steroids are an appropriate first line agent
  D. Pulmonary function tests would reveal a reduction in the FEV1/FVC ratio
  E. A trial of antibiotics against atypical pathogens such as mycoplasma or chlamydia would relieve this patient’s cough
Explanation:

The correct answer is D. This patient has a classic history for cough variant asthma. The diagnosis of asthma is helped by the history of atopy/eczema. His cough is predominant after exercising, but other allergens (e.g., cigarette smoke, dust, pollen) or cold weather could also induce asthma in many patients. Pulmonary function tests would reveal a decrease in the FEV1/FVC ratio, pathognomonic for obstructive lung disease.

There is no need for bronchoscopy (choice A) since in this patient there is low suspicion for an infectious process. His history of unprotected sexual intercourse does raise the suspicion for HIV, yet opportunistic pulmonary infections such as Pneumocystis Carinii pneumonia would be a late finding when the CD4 counts are less than 200.

A chest x-ray (choice B) would probably be unremarkable, since we have low suspicion for any pulmonic infection. Other causes of pulmonary wheezing and cough could include a foreign body, hypersensitivity pneumonitis, or intrathoracic lung mass. However, these are rarer etiologies and asthma is still primarily a clinical diagnosis.

Inhaled steroids (choice C) may be an appropriate treatment for mild or moderate persistent asthma. However, this patient only coughs roughly once or twice a week, and therefore would be considered to have mild intermittent asthma. An intermittent beta agonist would be the appropriate first line treatment for mild intermittent asthma.

There is no reason to suspect atypical pneumonia (choice E) in this afebrile patient with cough variant asthma.

 

A 56-year-old man is admitted to the intensive care unit for acute respiratory distress syndrome (ARDS). The patient was transferred from an outside hospital today after a 2-week hospitalization for pneumonia. During that time, the patient’s pulmonary status continued to deteriorate. One week ago he was intubated and placed on mechanical ventilation and over the past week, his oxygenation has worsened with a PaO2 of 66 on an inspired concentration of 100% oxygen. The patient has no other medical history except for rheumatoid arthritis. On transfer to the ICU, the patient is intubated and sedated on a mechanical ventilator. His chest radiograph shows patchy, bilateral, diffuse interstitial infiltrates. The most important intervention that will most benefit this patient is to

  A. keep the patient in a prone position during mechanical ventilation
  B. keep tidal volumes greater than 15 cc/kg
  C. limit peak inspiratory pressure to 45 cm H2O or less
  D. limit PEEP levels to less than 10 cm H2O
  E. limit tidal volumes to 6cc/kg
Explanation:

The correct answer is E. Although once limited to very specialized care units, patients with ARDS are becoming more prevalent in general medical ICU settings, in part due to the increasing incidence and recognition of the disorder. ARDS is an inflammatory condition of the lungs of unknown etiology but is associated with many conditions such as pneumonia, trauma, sepsis, and blood transfusions. The common denominator of all ARDS is profound hypoxia defined as a PaO2/FiO2 ratio of less than 200. Because of the profound hypoxia, the patients require mechanical ventilation for improvement. Ironically, many of the strategies employed over the years have actually contributed to or significantly worsened the injury of ARDS. The only effective intervention to date was recently shown in an NIH clinical trial. The mortality benefit conferred from this maneuver is substantial. The limitation of tidal volume to 6cc/kg or less is now standard of care and is required knowledge of any physician caring for critically ill patients.

Although there have been case reports that prone positioning during mechanical ventilation (choice A) is useful for these patients, the randomized trials indicate that there is no mortality benefit associated with this intervention.

The traditional teaching of keeping tidal volumes greater than 15 cc/kg (choice B) and of limiting peak inspiratory pressure to 45 cm H2O or less (choice C) is now incorrect and in fact, has been shown to be very detrimental to these patients. Experimental data have shown that ventilatory strategies that overdistend parts of the lung or allow the lung to cycle repeatedly between a collapsed state and an open state can lead to injuryXso-called ventilator-induced lung injury. PIPs should be limited to 35 cm H2O or less.

Limiting PEEP levels to less than 10 cm H2O (choice D) is opposite of what is required to manage these patients. PEEP values often exceed 10 cm H2O so that repeated cycling between a collapsed state and an open state is prevented. This is the so-called “open-lung” approach to ARDS management.

 

A 60-year-old man comes to the emergency department because of shortness of breath. He complains of a dry cough, but denies any fever, chills, or sweats. His past medical history is significant for a history of chronic obstructive pulmonary disease (COPD), hypertension, and alcoholism. His medications include an albuterol inhaler and furosemide. He appears to be in moderate respiratory distress. His temperature is 37.0 C (98.6 F), blood pressure is 146/98 mm Hg, pulse is 120/min, and respiratory rate is 36/min. His oxygen saturation on room air is 89%. His breath sounds are diminished bilaterally and he has diffuse wheezes. The remainder of the physical examination is unremarkable. A chest radiograph shows hyperexpanded lungs. An electrocardiogram shows sinus tachycardia. The most appropriate next diagnostic study is

  A. arterial blood gas analysis
  B. chest CT scan
  C. echocardiogram
  D. venous blood gas analysis
  E. ventilation-perfusion scan
Explanation:

The correct answer is A. In a patient with a history of chronic obstructive pulmonary disease (COPD), the constellation of described historical and physical findings with a chest radiograph showing no acute pathology indicates a COPD exacerbation. An arterial blood gas analysis, especially in the setting of a room air oxygen saturation of 89%, will more clearly define the patient’s oxygenation and ventilation status and assist in better management and triage.

A chest CT scan (choice B) in the setting of a chronic obstructive pulmonary disease exacerbation associated with a negative chest radiograph cannot be expected to provide additional useful information.

An echocardiogram (choice C) will offer no useful information since the patient’s respiratory distress, based upon the available history and physical exam, is due to an exacerbation of his chronic obstructive pulmonary disease.

A venous blood gas analysis (choice D) cannot provide any information regarding systemic oxygenation. It’s utility in this setting, therefore, is minimal.

Since there is no reason to suspect a pulmonary embolus, a ventilation-perfusion scan (choice E) will not provide any useful information in this instance.

 

A previously healthy 31-year-old woman comes to your office complaining of 1-day history of a cough and a fever. She reports that she was celebrating a job promotion 3 days prior and drank quite a bit of alcohol at a local bar. She had 2 episodes of vomiting that evening. She takes no regular medications and has only been using acetaminophen for fever suppression. Her temperature is 38.2 C (100.8 F). Her lungs have decreased breath sounds in the left base and right upper lobe. She has a cough that is productive of foul-smelling sputum. The remainder of her examination is unremarkable. The most appropriate management is to

  A. admit the patient to the hospital for clindamycin therapy
  B. admit the patient to the hospital for penicillin therapy
  C. admit the patient to the intensive care unit for levofloxacin therapy
  D. begin outpatient cefuroxime therapy
  E. begin outpatient erythromycin therapy
Explanation:

The correct answer is A. This is a patient who likely has pneumonia in the setting of likely aspiration. Since most pneumonia never have the etiologic agent identified, the treatment is empirical based upon patient locale at time of infection and presumed organisms based upon epidemiology. In this case, the presumed aspiration indicates that coverage for Gram-negative and anaerobic organisms is required. Clindamycin is a macrolide derivative that has activity against these agents. It is effective and is well-tolerated orally. Uncomplicated pneumonia such as community acquired or atypical infections rarely require hospitalization. For this patient with a likely anaerobic, purulent infection, a more monitored setting for therapy is required.

Penicillin (choice B) is an excellent choice for community acuquired pneumonia with the caveat that an increasing number of isolates of S. pneumonia are resistant. In some centers, this number is as high as 20%. However, penicillin has no activity against Gram-negative or anaerobic organisms.

Levofloxacin (choice C) is a fluoroquinolone that has broad activity against Gram-positive, Gram-negative, and some anaerobes. However, it does not have adequate coverage of anaerobic organisms to provide effective coverage for presumed aspiration. This patient has no objective findings that would warrant an ICU admission. Hemodynamic instability or respiratory distress requiring intubation would be classical reasons why patients with severe pneumonia may require an ICU stay.

Outpatient cefuroxime (choice D) is a second-generation cephalosporin that is standard outpatient therapy for community acquired pneumonia. It does not have the required broad Gram-negative (although it has some) coverage and it has no anaerobic coverage. This patient should however be hospitalized for observation during initial therapy.

Erythromycin (choice E) is a macrolide antibiotic that is also effective for both typical and atypical community acquired pneumonia but is only minimally useful in cases of aspiration pneumonia.

 

A 24-year-old African American woman comes to the clinic with a 2-week history of painful red “lumps” on her shins. She denies fevers, night sweats, cough, or sputum production. Her only past history was a broken arm when she was 10, and she does not take any medications. Her temperature is 37 C (98.6 F), blood pressure is 120/72 mm Hg, pulse is 68/min, and respirations are 16/min. Her lungs are clear and cardiac examination is normal. She has multiple bilateral, large, red, nodular lesions on her anterior tibial regions, which are painful to palpation. There is no purulent discharge. Laboratory studies show: leukocyte count 8,200/mm3, platelets 300,000/mm3, hematocrit 42%, BUN 16 mEq/L, and creatinine 0.9 mEq/L. A chest x-ray shows bilateral hilar adenopathy. Appropriate treatment for this patient should include

  A. antifungal therapy
  B. antituberculous therapy
  C. corticosteroids
  D. systemic chemotherapy
  E. systemic intravenous antibiotics
Explanation:

The correct answer is C. This patient has erythema nodosum in the setting of bilateral hilar adenopathy. Given the fact that she has no other underlying symptoms of infection, and is a young African American female, the erythema nodosum is most likely in the setting of sarcoidosis. The treatment in this case will often involve the use of corticosteroids.

Erythema nodosum can also be associated with several other disease processes such as streptococcal infections, upper respiratory infections, and inflammatory bowel diseases. The less common associations include tuberculosis, histoplasmosis, coccidioidomycosis, and drugs such as oral contraceptives and sulfonamides. She is on none of these drugs and has no symptoms of tuberculosis (choice B), a systemic or regional fungal infection(choice A), or systemic bacterial infection (choice E).

Hilar adenopathy is always a concern for a malignancy (choice D). However, this clinical scenario is much more classic for sarcoidosis. Biopsy proof should nevertheless be obtained. Therefore, chemotherapy is not indicated at this time.

 

 

A 68-year-old woman comes to the office for a health maintenance examination. She has had 5-7 episodes of “expectorated blood” in the past month that she describes as a “bit concerning.” She denies any other symptoms. She has been a patient of yours for 20 years and you have treated her for various “colds and flus” in the past, but she does not have any chronic medical conditions. She is a retired schoolteacher, gets regular exercise, and smokes a pack of cigarettes a day. She and her husband have become “world travelers” since both of their retirements. Her last mammogram, Pap smear, and colonoscopy were 1 year ago, and were normal, as they have always been. Her temperature is 37.0 C (98.6 F), blood pressure is 130/80 mm Hg, pulse is 65/min, and respirations are 16/min. Physical examination is unremarkable. The most appropriate next step is to

  A. obtain a sputum sample by transtracheal aspiration for cytology
  B. order a chest x-ray
  C. schedule fiberoptic bronchoscopy
  D. schedule a high-resolution CT scan
  E. reassure her that it is most likely nothing but to come back if she continues to have “expectorated blood”
Explanation:

The correct answer is B. This patient comes in for a routine examination but tells you something that could possibly be serious—that she has nonmassive (less than 100mL) hemoptysis (“expectorated blood”). Since she is a smoker and travels very frequently, you should not ignore this symptom. Since it is likely that the blood-streaked sputum is from the respiratory tract, a chest x-ray is the first diagnostic procedure that should be ordered.

Obtaining a sputum sample (choice A) by transtracheal aspiration is not indicated at this time because it is too invasive. Expectorated sputum should first try to be obtained. Blood in the sputum may occur in cases of bronchitis, pneumonia, bronchiectasis, a lung abscess, or an endobronchial tumor. Gram, fungal, and acid-fast stains will help diagnose an infectious cause, while cytology may be helpful to diagnose a tumor.

Fiberoptic bronchoscopy (choice C) is part of the evaluation of a patient with hemoptysis, but it is typically performed after a chest x-ray. It is the next step if a chest x-ray shows a mass, if the chest x-ray is normal and there are major risk factors for cancer, or if the chest x-ray is normal and there are no risk factors for cancer, but there is a recurrence of hemoptysis after weeks to months of observation.

A high-resolution CT scan (choice D) is usually only indicated after a chest x-ray is performed. If the chest x-ray shows a mass and a bronchoscopy fails to suggest a specific diagnosis, a HRCT is ordered. Also, if a chest x-ray shows parenchymal disease, a HRCT may be indicated for further evaluation.

It is inappropriate to reassure her that it is most likely nothing but to come back if she continues to have blood-streaked sputum (choice E) because hemoptysis can be the sign of serious disease, especially because she is a smoker and a “world traveler.” Even though she came to the office for a routine physical examination, a chest x-ray should be ordered at this time. Keep in mind that a chest x-ray is not part of a routine physical examination of an asymptomatic smoker.

 

A 31-year-old woman with primary pulmonary hypertension is admitted to the hospital because of increasing shortness of breath, dyspnea on exertion, and increasing home oxygen requirements. The agent that will selectively decrease her pulmonary arterial pressures is

  A. hydralazine
  B. nifedipine
  C. nitrous oxide
  D. prostacyclin I
  E. sodium nitroprusside
Explanation:

The correct answer is C. Nitrous oxide is a gas that is usually given in low doses, 20-80 ppm via inhalation. It then acts via cGMP to mediate vasodilation of the pulmonary vasculature without any systemic hemodynamic effects.

Hydralazine (choice A) is a potent generalized arterial dilator.

Nifedipine (choice B) is a calcium channel antagonist. In some persons with pulmonary hypertension, a therapeutic dose of this agent can be given without producing dramatic systemic hypotension. The majority of patients receiving this therapy have significant peripheral vasodilation as well.

Prostacyclin I (choice D) is used via direct pulmonary artery infusion to produce pulmonary vasodilation. However, it has moderate to severe side effects including increased GI motility and peripheral vasodilation.

Sodium nitroprusside (choice E) is a generalized arterial and venous vasodilator.

 

A 23-year-old woman comes to the emergency department because of a “severe asthma flare.” She reports that over the past hour, she has had progressively more difficulty breathing and that her medications at home have not helped her. She has a 7-year history of asthma with multiple hospitalizations. She was last hospitalized 3 years ago for a severe flare that required inpatient therapy with corticosteroids. Her current medications include albuterol 4 times daily, oral leukotriene inhibitors, cromolyn sodium, and theophylline. Her temperature is 37.0 C (98.6 F), blood pressure is 160/80 mm Hg, pulse is 90/min, and respirations are 32/min. Her breath sounds are scant with a prolonged expiratory phase. She appears to be moving minimal air. Albuterol and ipratropium nebulizers are initiated. An arterial blood gas is drawn and is most likely to show

  A. PaCO2 14 mm Hg, pH 7.22, PaO2 90 mm Hg
  B. PaCO2 14 mm Hg, pH 7.56, PaO2 86 mm Hg
  C. PaCO2 35 mm Hg, pH 7.22, PaO2 60 mm Hg
  D. PaCO2 35 mm Hg, pH 7.39, PaO2 98 mm Hg
  E. PaCO2 65 mm Hg, pH 7.24, PaO22 60 mm Hg
Explanation:

The correct answer is B. This patient has a severe asthma flare which caused her to hyperventilate to relieve her dyspnea. When looking at arterial blood gases, examine the pH to identify the acid-base disturbance, and then determine whether the acid-base disturbance is respiratory (change in CO2) or metabolic. The relationship between PaCO2 and pH determines whether the condition is acute or chronic. Chronic conditions have a pH closer to 7.4 than would be predicted based upon PCO2 because of compensation. For this patient, she is hyperventilating, but with no current inability to oxygenate. She should be alkalotic with a low PCO2 and her oxygenation should be nearly normal: PaCO2 14 mmHg, pH 7.56, PaO2 86 mm Hg.

A PaCO2 of 14 mm Hg, pH 7.22, PaO2 90 mm Hg (choice A) represents an acidosis that is likely metabolic since the PaCO2 is low. Oxygenation is normal. This is typical for a diabetic in ketoacidosis where they hyperventilate (Kussmaul breathing) to compensate for their systemic acidosis.

A PaCO2 of 35 mm Hg, pH 7.22, PaO2 60 mm Hg (choice C) reflects acidosis, likely metabolic since PCO2 is normal, with profound hypoxemia. This is typical for lactic acidosis of sepsis.

A PaCO2 of 35 mm Hg, pH 7.39, PaO2 98 mm Hg (choice D) represents a completely normal blood gas.

A PaCO2 of 65 mm Hg, pH 7.24, PaO2 60 mm Hg (choice E) reflects very late asthma. At this stage, PaCO2 has risen secondary to the inability to ventilate from severe bronchoconstriction, pH has fallen because of a respiratory induced acidosis, and oxygenation fails. When PaCO2 normalizes or becomes high in asthmatics, they have an impending respiratory failure in their near future.

 

A 37-year-old man with a history of allergic rhinitis comes to the office with a 3-day history of fever and cough. He was in his usual state of health until 3 days ago when he developed a cough productive of yellow-green sputum and fevers to 38.3 C (101.8 F). The fevers have been accompanied by drenching sweats. He has been experiencing right sided pleuritic chest pain. He denies shortness of breath, abdominal pain, weakness, or numbness. He has not had any sick contacts and has no recent travel outside of the United States. He has a 15-pack year history of smoking but denies any alcohol use or injection drug use. His temperature is 38.5 C (101.3 F), blood pressure is 132/74 mm/Hg, pulse is 82/min, respirations are 14/min, and oxygen saturation is 96%. Physical examination shows crackles at the right base A complete blood count and biochemical profile are all within normal limits. A chest x-ray shows a right lower lobe infiltrate. The most appropriate next step in the management of this patient is to

  A. admit the patient to the hospital for intravenous ceftriaxone therapy
  B. obtain a CT scan of the chest
  C. request a pulmonary consultation for bronchoscopy
  D. treat the patient as an outpatient with oral azithromycin therapy
  E. treat the patient as an outpatient with oral ciprofloxacin therapy
Explanation:

The correct answer is D. The patient’s history, exam, and x-ray are all consistent with a diagnosis of community acquired pneumonia (CAP). CAP can be safely treated as an outpatient in most circumstances. Exceptions to this rule are when the patient has an underlying medical condition (cardiac disease, pulmonary disease, diabetes, HIV, cirrhosis, renal disease, or malignancy), advanced age, or presents with a severe pneumonia manifested by unstable vital signs or bilobar pneumonia. The appropriate treatment for this patient is either a macrolide antibiotic or an extended spectrum fluoroquinolone as they will cover typical and atypical organisms.

As discussed previously this patient does not require intravenous therapy or hospital admission (choice A) unless his condition deteriorates. In addition ceftriaxone would not be adequate coverage as it does not cover the atypical organisms.

In the management of routine cases of CAP, a CT scan (choice B) is not necessary. A CT scan might be appropriate in situations where there is a concern for malignancy, underlying pulmonary disease, or non-resolving pneumonia. None of these conditions are present in this case.

This patient does not require a bronchoscopy (choice C). Potential indications for bronchoscopy are when there is a concern for an obstructive lesion on imaging studies, recurrent lobar pneumonia, or significant hemoptysis.

Ciprofloxacin (choice E) is not the antibiotic of choice for CAP. It does not provide adequate coverage against streptococcal pneumonia, which is the most common cause of CAP.

 

A 68-year-old man comes to the clinic because of progressive dyspnea on exertion (DOE) and shortness of breath over the last 7 months. He denies chest pain, orthopnea, or paroxysmal nocturnal dyspnea. His past medical history is significant only for mild osteoarthritis and an episode of pneumonia 20 years ago. His temperature is 37 C (98.6 F), blood pressure is 128/76 mm Hg, pulse is 98/min, respirations are 18/min, and oxygen saturation is 98%. His lungs are clear to auscultation and his heart is slightly tachycardic with no murmurs, rubs, or gallops. His abdomen is soft, nontender, with normal bowel sounds. His extremities have no edema. Rectal examination shows brown guaiac-positive stool. An electrocardiogram shows sinus tachycardia with a single PVC. Chest x-ray shows minimal scarring in the right lower lobe. Laboratory studies show a hematocrit of 27%, hemoglobin of 9.1 g/dL, platelets of 298,000mm3 , MCV 78 mm3, sodium of 139 mEq/l, potassium of 4.1 mEq/l, blood urea nitrogen of 16 mg/dL, and creatinine 0.9 mg/dL. The most appropriate next step in the patient’s management is a

  A. cardiac stress test to rule out 3 vessel coronary artery disease
  B. colonoscopy to rule out colon cancer
  C. high resolution CT scan (HRCT) to rule out pulmonary fibrosis
  D. iron pills and follow up in 3 months
  E. ventilation-perfusion (V/Q) lung scan to rule out chronic pulmonary emboli
Explanation:

The correct answer is B. It is important to remember that anemia can present with dyspnea on exertion and a complete blood count should always be part of this work up. This elderly patient has an iron deficiency anemia with hemoccult positive stool. A GI malignancy needs to be ruled out and colon cancer is the most likely etiology in this patient population. Therefore, a colonaoscopy is imperative.

Although ischemic cardiomyopathy or silent ischemia can certainly present with progressive dyspnea on exertion, there are other things in the patient’s history to suggest the cause of his symptoms. A cardiac workup (choice A) at this time is not the most pressing issue.

HRCT (choice C) is a very good test to evaluate for pulmonary fibrosis. It is likely that the CXR would show more abnormalities. The minimal scarring mentioned at the right lower lobe is likely from his prior pneumonia mentioned in the past medical history, and is not causing the patient any symptoms.

The patient is presenting with a microcytic anemia. The most likely cause is iron deficiency. However, it must always be remembered that the finding of a microcytic anemia should always prompt the immediate search for an underlying cause. So although the patient will likely require iron supplementation (choice D), follow up in 3 months with no other diagnostic test is not appropriate.

A V/Q scan (choice E) can rule out chronic pulmonary emboli, which is certainly a cause of progressive DOE, but as mentioned previously, there are other things in the patient’s history to suggest the cause of his symptoms.

 

A 31-year-old woman comes to the office for a follow-up visit. Two weeks ago, the patient underwent an echocardiogram for the evaluation of a systolic murmur. Her valves appeared normal but the echocardiogram disclosed elevated right ventricular systolic and diastolic pressures consistent with pulmonary hypertension. She has no primary lung disease and reports no symptoms of dyspnea or tachypnea. Her other past medical history is unremarkable and she takes only oral contraceptive pills for medications. The most appropriate next step in the management of this patient is to

  A. prescribe calcium channel blockers, orally
  B. prescribe nitric oxide, inhaled
  C. prescribe prostaglandin, intravenously
  D. refer the patient for oxygen diffusion capacity testing
  E. refer the patient for vasodilator response testing
Explanation:

The correct answer is E. The management of patients with pulmonary hypertension focuses on three issues: is the disease secondary to primary pulmonary disease (secondary pulmonary hypertension), is the patient responsive to vasodilator therapy, and, can the pulmonary pressures be made normal with medication. For this patient, the assumption is, given her age and lack of medical history, that her pulmonary hypertension is primary. In addition to pulmonary function testing to help verify this assumption, vasodilator testing to determine whether the pulmonary vasculature is responsive or not is the first step in the management of such patients.

Calcium channel blockers (choice A), inhaled nitric oxide (choice B) or intravenous prostaglandin (choice C) are all agents used in the management of primary pulmonary hypertension. The choice of these agents depends on whether the patient is vasodilator responsive and the side effect profile. Systemic agents such as calcium antagonists and prostaglandins are associated with often profound systemic hypotension which limits their utility is normalizing pulmonary pressures.

Referring the patient for oxygen diffusion capacity testing (choice D) is not necessary. Standard spirometry as part of a full battery of pulmonary function tests are indicated to assess whether the hypertension is primary or secondary, but diffusion testing is only useful in cases where hypoxemia coexists with existing pulmonary disease.

 

A 72-year-old smoker is admitted to the hospital for COPD exacerbation. Admission vitals are respirations 18/min, with a blood pressure of 180/100 mm Hg, and an oxygen saturation of 91%. He is started on nebulized albuterol and ipratropium bromide, as well as prednisolone intravenously. Admission chest radiograph reveals flattened hemidiaphragms, increased retrosternal clear space, and hyperlucent lungs. Given a suspicion of pulmonary embolus, a ventilation perfusion scan is performed demonstrating nonsegmental perfusion defects of the left upper lobe, with a small left lung and a complete absence of perfusion and ventilation of the entire right lung. The patient becomes acutely short of breath in the nuclear medicine department. His respirations are 30/min with otherwise normal vital signs. After supplemental oxygen (4 L/min by nasal cannula) and nebulizers are administered, the respirations become 29/min, with a blood pressure of 80/40 mmHg, and an oxygen saturation of 82%. A repeat chest radiograph is pending. The most appropriate management is to

  A. administer heparin, intravenously
  B. insert a chest tube on the left side
  C. insert a chest tube on the right side
  D. obtain a surgical consult for emergent lung volume reduction surgery
  E. send him for coronary artery catheterization
Explanation:

The correct answer is C. The patient is exhibiting clinical signs of a tension pneumothorax, including pulmonary and cardiac failure. The ventilation perfusion scan demonstrates lack of ventilation and perfusion of the right lung, which is consistent with a pneumothorax. A tension pneumothorax must be suspected given the diminished size of the left lung. A tension pneumothorax is a unilateral pneumothorax that becomes loculated by a one-way valve mechanism and compromises the contralateral lung and the venous return to the chest. Diagnosis is made by the lack of ipsilateral lung sounds due to cardiopulmonary collapse or chest radiograph. Treatment is immediate chest tube insertion to relieve the pressure in the right hemithorax.

The ventilation perfusion scan is consistent with a pneumothorax of the right lung with signs of tension, given the small left lung. The ventilation perfusion mismatches of the left lung apex are consistent with bullous disease, which is common in patients with COPD. A pulmonary embolus creates ventilation perfusion mismatches on the ventilation perfusion scan. Heparin would be an appropriate treatment if the ventilation perfusion scan was positive for pulmonary embolus (choice A).

The chest tube must be ipsilateral to the tension pneumothorax, not on the left side (choice B).

Volume reduction surgery (choice D) is a controversial method for reducing lung volumes in patients with emphysema. It is an elective procedure and is inappropriate for the management of tension pneumothorax.

The patient’s low blood pressure is due to the effect of the tension pneumothorax on preload, not due to intrinsic coronary artery disease as seen in a cardiac catheterization (choice E).

 

A 13-year-old boy is brought to the emergency department because of shortness of breath. He has a medical history significant for asthma and he has been admitted to the hospital several times in the past, most recently 3 months ago when he needed to be intubated for extreme respiratory distress. He lives at home with his mother and 2 cats. His medications include albuterol, ipatroprium, and steroid inhalers. As you approach him you notice that he is using all accessory muscles of respiration and you hear audible wheezing. His temperature is 37.0 C (98.6 F), blood pressure is 122/68 mmHg, pulse is 102/min, and his respiratory rate is 34/min. His oxygen saturation on room air is 94%. Physical examination is significant for diffuse expiratory wheezes. The factor in this patient’s history that constitutes a predictor of poor outcome with respect to his risk for morbidity from his asthma is

  A. age
  B. history of having a pet
  C. history of living in a single parent household
  D. history of previous intubation
  E. sex
Explanation:

The correct answer is D. A history of intubation related to an asthma exacerbation has been found to be a predictor of adverse outcome with respect to asthma. Other historical factors indicating poor outcome in this patient’s history are a recent hospitalization related to asthma and steroid use.

The patient’s age (choice A), while leaving hope that the patient will be able to “outgrow” his asthma as his airways increase in diameter with increasing age, is not a predictor of poor outcome.

The patient’s history of having a pet (choice B), while leaving open the possibility of providing a possible trigger for an asthma exacerbation, is not a predictor of adverse outcome.

The patient’s history of living in a single family household (choice C) is not a predictor of adverse outcome.

The patient’s sex (choice E) is not a predictor of adverse outcome.

 

An 80-year-old male is admitted to your inpatient medical service with a presumed aspiration pneumonia. He has a medical history significant for chronic renal failure, hypertension, aspiration pneumonias, and Alzheimer’s disease. His medications include furosemide, thiamine, and folate. He has a history of alcohol abuse in the past and currently is a nursing home resident. He is a cachectic old man who is confused, which you are told is his baseline mental status. He has a nasogastric tube in place, which has been used for tube feeds. His temperature is 37.8 C (100 F), blood pressure is 122/67 mm Hg, heart rate is 98/min, respiratory rate is 23/min, and his room air oxygen saturation is 97%. He has decreased breath sounds in his right lower lung field and his cardiac examination is unremarkable. A chest radiograph shows a right lower lobe consolidation, consistent with pneumonia. While reviewing his medical records, you notice that this patient has been admitted for aspiration 4 times over the past 12 months. The intervention that could be instituted to reduce his risk of aspiration pneumonia in the future is to

  A. change his nasogastric tube to a feeding tube placed via a nasogastric route
  B. change his nasogastric tube to a percutaneous gastrostomy tube
  C. change his nasogastric tube to a percutaneous jejunostomy tube
  D. change his nasogastric tube to an open (surgically placed) gastrostomy tube
  E. elevate the head of his bed
Explanation:

The correct answer is E. Of all the methods that are available to decrease the risk of aspiration pneumonias in chronically ill patients, the one that works best is the adherence to strict aspiration precautions. These precautions include elevating the head of the bed to limit passive reflux, strict monitoring during all oral intake, and choice of food that will decrease the risk of aspiration (thickened liquids etc.). Since this patient already has a nasogastric tube in place, the best intervention is to elevate the head of his bed. This will reduce the risk of pooling of oropharyngeal secretions and their subsequent aspiration.

Changing a nasogastric tube to a feeding tube via a nasogastric route (choice A) does not decrease the risk of aspiration. The narrower caliber of these tubes do, however, make it more comfortable for the patient.

Changing a nasogastric tube to a percutaneous gastrostomy tube (choice B) in chronically ill patients has not been shown to reduce the risk of aspiration pneumonias if they already have nasogastric tubes in place. This procedure, however, is used for patients who will need long-term enteral nutrition via a feeding tube since it is more comfortable.

Changing a nasogastric tube to a percutaneous jejunostomy tube (choice C) has not been shown to reduce the risk of aspiration in chronically ill patients who have nasogastric tubes in place. This procedure, however, is used for patients who will need long-term enteral nutrition via a feeding tube since it is more comfortable.

Changing a nasogastric tube to a surgically placed gastrostomy tube (choice D) has not been shown to reduce the risk of aspiration pneumonias in chronically ill patients who have nasogastric tubes in place. This procedure is recommended for patients who will need long-term enteral tube feedings (for comfort) who have contraindications to a percutaneous approach.

 

A 32-year-old African American woman returns to the clinic for a follow-up visit. She was seen 2 weeks prior with complaints of dyspnea, dry cough, chest pain, and tightness of the chest. An electrocardiogram was within normal limits. A chest x-ray was performed a week later which showed bilateral hilar lymphadenopathy with pulmonary infiltrate. While in clinic now, she points out a rash on her nasal tip that has been present for a number of months. There is a 4 x 2 cm violaceous, indurated plaque involving the nasal tip extending to the bilateral ala. There are a few tiny button-like papules in the center of the plaque. In addition, there are waxy, translucent lesions with flat tops on the face, lids, around the orbits, and in the nasolabial folds. The most appropriate next step in evaluation is to

  A. determine serum angiotensin-converting enzyme and serum calcium levels
  B. determine serum CEA marker level
  C. order a complete blood count
  D. refer her for a bronchoscopy
  E. send her for a lung biopsy
Explanation:

The correct answer is A. Determining serum angiotensin-converting enzyme (ACE) and serum calcium is the correct management for this patient with suspected sarcoidosis. Although a bronchoscopy (choice D) and a lung biopsy (choice E) will also help with diagnosis of sarcoidosis, serum ACE and calcium levels are less invasive tests to obtain for diagnosis. Sarcoidosis involves multisystems including lungs, eyes, peripheral lymph nodes, spleen, gastrointestinal tract, hearing, and musculoskeletal systems. Hypercalcemia may occur in any stage of sarcoidosis. Corticosteroids lower the raised calcium level to normal by inhibiting the peripheral action of 1,25(OH)2D3 and by metabolizing the compound to an inactive metabolite. The serum ACE level is also raised in 60% of patients. ACE activity is higher in patients with hilar adenopathy and pulmonary infiltration.

A complete blood count (choice C) is incorrect, because hemolytic anemia is rare in sarcoid and leukopenia alone is too non-specific for diagnosis of sarcoid.

Serum CEA level (choice B) is incorrect, because this is a non-specific tumor marker used to monitor for colon carcinoma recurrence, as well as some lung adenocarcinoma responses to chemotherapy.

 

A 73-year-old woman comes to the office complaining of a 6-month history of progressive shortness of breath. She has a 30-pack year smoking history but quit 25 years ago. She has no other significant medical history. She tells you that her exercise tolerance has been slowly decreasing over the last 6 months and now she gets tired walking 1-2 blocks. One year ago, she was able to walk a mile without symptoms. She reports a cough but denies sputum production, fevers, or chills. Her temperature is 37.0 C (98.6 F), blood pressure is 135/90 mm Hg, pulse is 90/min, and respirations are 22/min. Physical examination shows late inspiratory crackles. There is no significant lower extremity edema. A chest x-ray reveals a diffuse ground glass appearance. Pulmonary function tests are obtained. You would expect the study to show:

FVC FEV1 FEV1/FVC RV TLC Diffusion Broncho-dilator response
  A. Dec Decreased Decreased Increased Increased Normal Positive
  B. Dec Decreased Normal Decreased Decreased Normal Negative
  C. Dec Decreased Increased Decreased Decreased Decreased Negative
  D. Dec Decreased Decreased Increased Increased Decreased Negative
  E. Normal Normal Normal Normal Normal Normal Negative
Explanation:

The correct answer is C. This patient has interstitial fibrosis. Clues to the diagnosis include a history of mildly progressive shortness of breath with a non-productive cough. Physical exam findings of late crackles with inspiration, and the lack of signs of CHF or reactive airway disease such as wheezing suggest interstitial fibrosis. A chest x-ray revealing a ground glass pattern or a reticular nodular pattern also suggests the diagnosis. Pulmonary function tests reveal decreased lung volume with decreased forced vital capacity and FEV1. The ratio of FEV1/FVC might be normal or increased. Diffusion capacity should be decreased.

Patients with asthma have a similar PFT pattern to patients with other obstructive lung disease such as emphysema or bronchitis (choice A) but should have a good bronchodilator response since the pathology of asthma involves bronchospasm. Patients with COPD with bronchospasm will have similar PFTs.

Patients with restriction secondary to obesity or kyphosis have PFTs which show a restrictive pattern (choice B) similar to a patient with interstitial lung disease. The two etiologies can be differentiated by either history or physical exam findings or by examination of the diffusion capacity. In obesity/kyphosis, the diffusion capacity is normal while in interstitial lung disease, the diffusion capacity is decreased.

Emphysema (choice D) patterns on PFT look identical to other obstructive patterns (e.g., asthma, chronic bronchitis) with two exceptions. First, the diffusion capacity in emphysema is decreased. Second, there should not be a significant bronchodilator response in pure emphysema. The caveat of this is that very often chronic bronchitis and emphysema coexist so that some bronchodilator response is possible.

This patient does have underlying lung disease so it would be unlikely for her to have normal PFTs (choice E).

 

You are the physician-on-call and are asked to see a 57-year-old woman who has been complaining of progressive shortness of breath over the past 2 days. The woman was admitted to the medical service 6 days ago after a fall and has been on bed rest for a nondisplaced pubic ramus fracture. She has been on deep vein thrombosis prophylaxis with subcutaneous heparin. Her past medical history is significant for type-II diabetes and dialysis-dependent renal failure secondary to diabetic nephropathy. She makes no urine at baseline. Her last dialysis run was 4 days ago, though she is usually dialyzed 3 times per week. She is complaining of shortness of breath but no chest pain. She is anxious-appearing, has a blood pressure of 160/105 mm Hg, respiratory rate of 30/min, and heart rate of 110/min. She has an oxygen saturation of 80% on room air and appears cyanotic. Physical examination reveals a jugular venous pressure of 10 cm and inspiratory crackles half way up from the bases upon auscultation of the lungs. An electrocardiogram shows a rate-related right bundle branch block but no ischemic changes. Chest x-ray obtained yesterday revealed interstitial edema and vascular redistribution to the apices. The patient’s husband has arrived from home and is very concerned about the recent events. In particular, he asks what has caused the current problem with her breathing. You suspect that a scheduling error may have delayed her last dialysis run and caused volume overload, but you are not completely sure. The most appropriate initial approach to take with the husband given that you have just met the patient for the first time is to

  A. acknowledge sympathetically that a mistake may have been made but will not likely cause permanent harm
  B. ask him to kindly wait until the hospital lawyer can be present before speaking to him about the matter
  C. explain that hospitals are complex institutions, making scheduling a difficult process at times
  D. explain that you are the physician on-call tonight and are not privy to all of the scheduling details, but that you will explore in detail whether a mistake has been made and that you will communicate these findings in a timely manner to both him and the patient
  E. speculate on how recent health maintenance organization’s cuts in reimbursement may have made it necessary to decrease the number of dialysis runs for each hospitalized patient
Explanation:

The correct answer is D. When you suspect a mistake has been made in the hospital, the first approach should always be to make an earnest effort to get all of the facts. This approach will immediately build a rapport with the husband as you become an advocate for the truth, readily demonstrate your interest in the patient’s specific case, and help identify process errors in the hospital.

While acknowledging that a mistake has been made (choice A) is often an important step, it usually does not go far enough because patients and their families are interested in why mistakes happen and whether they will happen again. It is simply unprofessional to not go further and explore all of the details of the mistake regardless of whether an injury actually occurs.

While seeking legal advice (choice B) may be an important step in dealing with this issue, the initial approach again should focus on a thorough investigation of the facts by the physician and relevant administration.

Philosophical approaches (choice C) are unsatisfactory because they seek to diminish any individual responsibility for a mistake happening and are unlikely to result in correcting a serious process error.

Speculating on cuts in insurance reimbursement (choice E) is inappropriate and unprofessional. The job of physicians is to care for their patients and be their advocates. This must occur regardless of the practice environment one finds themself in. Specifically, whether the patient population is wealthy and cash paying or whether they depend on insurance, medical decisions are based upon medical need and not insurance mandates. To acknowledge any differently is to commit medical malpractice.

 

A 102-year-old man is brought to the clinic from his apartment by a neighbor because of a cough productive of green sputum. He has a past medical history of a stroke 10 years ago with residual left arm weakness. The patient is widowed and lives alone. He takes no medications. Vital signs are: temperature 38.8 C (101.8 F), blood pressure 100/50 mm Hg, pulse 110/min. On physical examination, the patient has rhonchi in the right lower lung field. A chest x-ray reveals an infiltrate in the right lower and middle lobes. The patient has a score of 30/30 on a mini mental status examination. The patient states that he has “lived a good life and now wants to go home”. He refuses any intravenous medications and will only take pills. After discussing that the pneumonia could potentially be fatal without intravenous antibiotic treatment, the patient continues to ask to be sent home. The neighbor insists that the patient should be admitted to the hospital. The next step in the care of this patient is to

  A. admit the patient to hospital care with intravenous antibiotics
  B. admit the patient to hospital care without intravenous antibiotics
  C. admit the patient to psychiatric ward with antibiotics
  D. discharge the patient home with oral antibiotics
  E. discuss the case with the hospital attorney
  F. initiate antidepressant therapy and administer intravenous antibiotics
Explanation:

The correct answer is D. The principle of individual autonomy is central to medicine. The only exceptions are if the patient has no clearly expressed wishes documented and is unable to make a decision or if the patient is deemed incompetent to make a decision. This patient is clearly competent to refuse treatment despite the wishes of his physician or neighbor. It is important to make the consequences of his decision clear to the patient before discharge and to tell him that he should return immediately should he change his mind.

Hospital care with or without intravenous antibiotics (choices A and B) is inappropriate as this patient is refusing hospital care.

This patient is clearly competent to refuse treatment, so he should not be admitted to the psychiatry ward (choice C) or given antidepressant therapy (choice F).

Consultation with an attorney (choice E) is not necessary as the patient is clearly competent to refuse medical care.

 

A 2-year old boy is brought to the emergency department at 2:00 a.m. because of episodic coughing “fits”. He was well until one day earlier when he developed rhinorrhea and a temperature of 38.3 C (101 F). He has no prior history of respiratory illnesses, and no one else in the family is ill. On arrival to the hospital, he is coughing in a rapid, “barking” fashion, but appears otherwise well. His temperature is 38.2 C (100.8 F), heart rate is 120/min, respiratory rate is 50/min, and oxygen saturation is 96% on room air with coughing. Physical examination shows clear lungs, a normal cardiac and abdominal examination, and no rash. He continues to have repeated, episodic coughing with inspiratory stridor at rest. A frontal radiograph of the chest at this time is most likely to reveal

  A. cardiomegaly with pulmonary venous congestion
  B. left lower lobe infiltrate
  C. pneumothorax
  D. subglottic swelling
  E. thumb-shaped epiglottis
Explanation:

The correct answer is D. This child’s clinical picture is most consistent with infectious croup (laryngotracheobronchitis), commonly caused by parainfluenza viruses. The prodrome of upper respiratory illness with fever followed by a spasmodic, barking cough, is typical of this diagnosis. The classic sign on chest x-ray is the subglottic swelling ( “steeple sign,”), which is the result of viral-induced swelling of the subglottic tissue. This swelling can cause an incomplete airway obstruction, leading to stridor either at rest or during crying. Stridor at rest is an indication for treatment with steroids to decrease inflammation.

Dyspnea in a child can be the result of congenital heart disease and resulting congestive heart failure, manifested as cardiomegaly and pulmonary venous congestion on x-ray (choice A), but given the absence of prior symptoms, the normal oxygen saturation, and the lack of rales on lung examination, this is unlikely.

Fever and cough in a child also raises the possibility of pneumonia as a diagnosis. A lobar pneumonia (choice B) would be a surprising x-ray finding in this case given the child’s clear lung fields on examination and the presence of stridor, which indicates upper airway involvement.

Pneumothorax (choice C) can be a cause of sudden dyspnea in a child, especially in tall, male adolescents who may be predisposed to spontaneous pneumothoraces. This child’s prodrome and stridor on examination make this diagnosis unlikely.

Prior to the introduction of the vaccine against Haemophilus influenzae type b, epiglottitis was a common and much-feared diagnosis in pediatrics. Infection of the epiglottis by this organism led to rapidly progressive airway obstruction and potentially death. A lateral neck radiograph would demonstrate a thumb-shaped epiglottis (choice E) in these cases. This sign is not seen on frontal views of the chest.

 

A 67-year-old man comes to the clinic for an initial visit. He and his wife have just moved to the area from out of the state. He brought along his medical records which show that he has hypertension, mild peripheral vascular disease, and that he carries the diagnosis of emphysema. He tells you that he smokes 1 pack of cigarettes per day but refrains from all but social alcohol. His medications include thiazide, captopril, quinine, and albuterol inhalers as needed. He has never had pulmonary function testing. His temperature is 37.0 C (98.6 F), blood pressure is 135/85 mm Hg, pulse is 72/min, and respirations are 14/min. He has diffuse bilateral expiratory wheezes with a mildly prolonged expiratory time. His abdomen is obese, but non-tender and there is no fluid wave. The most appropriate intervention for this patient is to

  A. change captopril to lisinopril
  B. encourage him to quit smoking immediately
  C. increase his thiazide diuretic dose
  D. initiate home oxygen therapy
  E. obtain pulmonary function testing
Explanation:

The correct answer is B. The two interventions that have been shown to affect mortality, smoking cessation and oxygen therapy, should be foremost in the minds of all caregivers who manage patients with COPD. At any stage of the disease, smoking cessation is the most important intervention that can be taken to improve lifestyle and longevity. Other management strategies such as medications, rehabilitation, and even surgery are less effective, sometimes ineffective, when smoking is still practiced.

Changing ACE inhibitors from captopril to lisinopril, a 3 times per day drug to a once daily drug (choice A) is not required for this patient. Since this is a compliance/lifestyle issue, it should not take priority of a critical medical intervention. There is no difference in efficacy between the two drugs.

This patient has reasonably controlled blood pressure so that increasing his thiazide diuretic dose (choice C) may improve his blood pressure slightly, but its benefits are minimal when compared to those obtained with smoking cessation.

The decision to initiate home oxygen therapy (choice D) is based upon arterial oxygen pressures of less than 55 mm Hg (saturations less than 88%) and therefore a resting arterial blood gas is required before a decision to initiate long-term oxygen therapy is made.

Pulmonary function testing (choice E) is an important tool to stratify patients with COPD and to determine if they have an element of reversible bronchoconstriction. Since they are not therapeutic however, they do not take precedence over immediate medical interventions that have life-prolonging consequences.

 

A 72-year-old woman with a 6-month history of non-small cell lung cancer comes to the office because of neck and facial swelling. She denies any shortness of breath or hemoptysis. Physical examination shows dilated neck veins and edema of the face and right arm. A CT scan of the chest shows a right paratracheal mass with diminished opacification of the central venous structures. The most appropriate next step in the management of this patient is to

  A. administer dexamethasone every 6 hours
  B. begin chemotherapy
  C. biopsy the mass
  D. give her intravenous morphine
  E. recommend radiation therapy
Explanation:

The correct answer is E. The patient has a classic case of superior vena cava syndrome (SVCS), which is due to obstruction of the superior vena cava. The vast majority of cases of SVCS are caused by malignancies, with lung cancer being the most common. The most feared complication of SVCS is upper airway obstruction. Radiation therapy is the treatment of choice for most patients with SVCS.

Corticosteroids, such as dexamethasone, (choice A) are not the primary treatment of SVCS. In some malignancies which are steroid responsive or if there is significant inflammation, steroids can be started as an adjunct to radiation therapy.

Chemotherapy (choice B) is not the initial treatment of choice for patients with non-small lung cancer who present with SVCS. After the patient has been started on radiation, a chemotherapeutic regimen can be offered to the patient if it is appropriate.

In patients with known lung cancer, a biopsy of the mass (choice C) causing the SVCS is usually not necessary and treatment can commence once the clinical diagnosis is made. In patients without a history of cancer, every effort should be made to obtain a diagnosis before starting treatment, as there are benign causes of SVCS (e.g., thyroid enlargement, thrombosis).

Narcotics (choice D) are not direct therapy for SVCS. Of course if the patient has any significant pain due to their cancer then narcotics should be prescribed to help alleviate it. This patient is not complaining of any pain by history but like any cancer patient she should be asked directly whether she is experiencing any pain or discomfort.

 

A 34-year-old airline pilot is admitted to the hospital with a first episode of a spontaneous pneumothorax. The pneumothorax resolved after 2 days of pleural decompression by a chest tube. A chest x-ray shows complete resolution of the pneumothorax, and a CT scan of the chest shows an apical bulla in the right lung. While discussing further management of his condition, it is appropriate to advise the patient to

  A. avoid flying until after bullectomy
  B. carry oxygen with him during the flight
  C. change his career
  D. quit smoking
  E. resume his duties after 1 week of rest
Explanation:

The correct answer is A. A spontaneous pneumothorax can recur especially if associated with bulla. In patients with high risk due to the possibility of recurrence, a first episode of a pneumothorax deserves definitive treatment. Airline pilots and patients living away from hospital facilities are at risk for morbidity and mortality from recurrence of a pneumothorax when they are far from treatment facilities. Hence, this patient needs to be advised to undergo definitive treatment for resection of the bulla before resumption of duties.

Oxygen is helpful in treating a pneumothorax when it is of small percentage and asymptomatic. A sudden high percentage of a pneumothorax cannot be cured by oxygen (choice B).

A pneumothorax from a bulla can be treated and recurrence can be avoided by bullectomy and pleurodesis. Hence, changing careers is not essential (choice C).

Quitting smoking (choice D) has no association with avoiding the recurrence of a pneumothorax.

After appropriate treatment of pneumothorax, the recurrence risk is small and duties can be resumed as soon as possible. Taking rest has no association with avoiding recurrence of a pneumothorax (choice E).

 

A 45-year-old man with a history of recurrent deep venous thrombosis and known to have the factor V Leiden mutation comes to the emergency department because of an abrupt onset of severe dyspnea that started while he was walking to work. He also complains of sharp chest pain on inspiration and says that he has coughed up small amounts of blood. He is not currently taking any medications and he has no known drug allergies. His temperature is 37.8 C (100.1 F), his blood pressure is 100/60 mm Hg, pulse is 110/min, respirations are 30/min, and his oxygen saturation is 91% on room air. He appears mildly anxious and is clearly tachypneic, but he is able to speak in full sentences. Cardiac examination reveals a prominent P2. His lungs are clear. His right lower extremity has 2+ pretibial edema and his left lower extremity has no edema. A chest x-ray is unremarkable. A ventilation/ perfusion lung scan is consistent with a high probability for bilateral pulmonary emboli. The next most appropriate step in management is to

  A. administer intravenous unfractionated heparin and warfarin; discontinue the heparin as soon as the INR is therapeutic
  B. administer intravenous unfractionated heparin and warfarin; discontinue the heparin 2 days after a therapeutic INR is achieved
  C. begin therapy with subcutaneous low molecular weight heparin and warfarin and send the patient home to follow up in anticoagulation clinic in 1 week
  D. obtain an ultrasound of his right lower extremity to rule out deep venous thrombosis
  E. perform a CT angiogram to confirm the diagnosis of pulmonary embolism
Explanation:

The correct answer is B. In the setting of a high pretest probability for pulmonary embolism (as in this case with a patient who is known to be hypercoagulable and presents with classic symptoms), a high probability ventilation/perfusion scan is sufficient to make the diagnosis. Ventilation/perfusion scans may be read as negative, but are otherwise described in terms of the probabilities (low, intermediate, or high) of pulmonary embolism. The patient should be immediately anticoagulated with heparin to prevent further progression of thrombosis. Studies have demonstrated that low molecular weight heparin and unfractionated heparin have similar efficacies in this setting. Given the severity of his symptoms and abnormal vital signs, the patient should not be sent home until he has been monitored in the hospital and his condition stabilizes. This patient should be started on intravenous unfractionated heparin and warfarin and the heparin should be discontinued 2 days after a therapeutic INR is achieved.

Start intravenous unfractionated heparin and warfarin; discontinue the heparin as soon as the INR is therapeutic (choice A) is incorrect because the heparin must overlap with the warfarin for at least 2 days after the INR is therapeutic. Warfarin inhibits the synthesis of factors II, VII, IX, and X. The initial increase in INR seen with warfarin is due to inhibition of the factor with the shortest half-life, factor VII (the half life is approximately 7 hours). The antithrombotic effect of warfarin is thought to rely mainly on inhibition of factor II. Due to its longer half life, it can take up to 2 days for factor II levels to sufficiently decline. Therefore, if the heparin is discontinued as soon as the INR is therapeutic, the patient will be left essentially un-anticoagulated for 1-2 days.

Beginning subcutaneous low molecular weight heparin and warfarin and sending the patient home to follow up in anticoagulation clinic in 1 week (choice C) is incorrect due to the reasons described above. While low molecular weight heparin has similar efficacy to unfractionated heparin in this setting, there is no data to suggest nor any consensus that patients with pulmonary embolism can safely be treated as outpatients. This patient’s tachycardia, low blood pressure, high respiratory rate, and low oxygen saturation warrants cardiac monitoring and an inpatient stay.

Obtaining an ultrasound of his right lower extremity to rule out deep venous thrombosis (choice D) is incorrect. The patient already has a known history of deep venous thrombosis. While the physical finding of asymmetric lower extremity edema is strongly suggestive of a deep venous thrombosis in the right lower extremity, knowing this for certain would not change management. The patient will receive anticoagulation for his pulmonary embolism regardless of any findings on lower extremity ultrasound.

Performing a CT angiogram to confirm the diagnosis of pulmonary embolism (choice E) is incorrect because a CT angiogram is neither more sensitive nor more specific than ventilation/perfusion lung scan. Pulmonary angiogram (via fluoroscopy, not computed tomography) is the gold standard to make the diagnosis of pulmonary embolism. As noted above, a high probability scan is sufficient to make the diagnosis of pulmonary embolism in this setting. The PIOPED study provides data on ventilation/perfusion lung scanning for the diagnosis of pulmonary embolism, validating its use. While a CT angiogram is commonly used in practice, its routine use has not yet been validated in large trials.

 

A 67-year-old man presents to your office complaining of severe shortness of breath and a cough. The man is a long-time patient who you have been treating for chronic obstructive pulmonary disease (COPD). He has a 120 pack-year smoking history. He was recently hospitalized for a flare of his COPD and he has been intubated in the past for respiratory distress. His other medical history is notable for diabetes mellitus and hypercholesterolemia. He reports to you that for the past 3 days he has increasing dyspnea and a fever. He has gradually developed a cough. On examination, he is moderately dyspneic at rest and has marked dyspnea on exertion. His blood pressure is 130/80 mm Hg, heart rate is 78 /min, and respirations are 20 /min. The patient does not appear cyanotic. His lung exam demonstrates a markedly prolonged expiratory time and diffusely diminished breath sounds. The most appropriate intervention at this time is to

  A. admit the patient to the hospital
  B. obtain a chest radiograph and prescribe antibiotics if an infiltrate is present
  C. prescribe oral cefuroxime and oral steroids
  D. prescribe oral cefuroxime and see the patient in seven days
  E. refer the patient to the local emergency department
Explanation:

The correct answer is A. This patient has long-standing pulmonary disease and is suffering from some acute respiratory event, either infection or a COPD flare. His past medical history is enlightening because it can be appreciated that this patient has required ventilatory support for his pulmonary disease in the past. In the office, he is clearly in some distress. With his history and known lung disease, this patient should be admitted to the hospital for further care and observation.

Similar reasoning holds for obtaining a chest radiograph and prescribing antibiotics if an infiltrate is present (choice B). Knowing that his distress is due to pneumonia does not alter the fact that he needs to be treated in the hospital.

Prescribing oral cefuroxime and oral steroids (choice C), although a reasonable regimen for a COPD flare, fails to address how this patient’s acute respiratory compromise will be addressed. He has been intubated in the past for flares such as this and discharging the patient out of your care could result in serious harm or even death to this patient.

Prescribing oral cefuroxime and see the patient in seven days (choice D) is clearly inappropriate since the patient is presently in mild to moderate distress and the cause for this is unknown. Even if he does have an infection, the antibiotics will take days to be of any assistance.

It would be inappropriate to refer the patient to a local emergency department (choice E) where he may sit unobserved in triage or may be discharged home if another physician had a different impression of the patient.

 

A 71-year-old retired clerk comes to the clinic because of hoarseness. He says that he had a “chest cold” 4 weeks ago with congestion and a sore throat. He attributed the hoarseness of his voice to the cold, but has been worried for the last 2 weeks because the hoarseness has been persistent. He is a diabetic taking insulin twice a day. He smokes 1-2 packs of cigarettes a day and stopped drinking alcohol about 2 years ago. His only other medication is an aspirin a day. He has had no prior surgeries and no other medical problems. Laryngoscopy performed in the office demonstrates paralysis of the left vocal cord but no evidence of masses or inflammation in the larynx. The most likely etiology of this patient’s vocal cord paralysis is

  A. diabetic neuropathy
  B. laryngeal carcinoma
  C. lung carcinoma
  D. pneumonia
  E. stroke
Explanation:

The correct answer is C. Hoarseness of the voice is a symptom of vocal cord paralysis. Vocal cord paralysis is a result of injury to the ipsilateral recurrent laryngeal nerve, which is a branch of the vagus nerve supplying all the intrinsic muscles of the larynx. Unilateral vocal cord paralysis is much more common than bilateral disease by a factor of 3 to 1 and the left side is more commonly affected than the right side due to the longer course of the left recurrent laryngeal nerve. The most common cause of unilateral vocal cord paralysis is lung cancer. This patient’s history of smoking puts him at high risk for developing lung cancer.

Laryngeal carcinoma (choice A) can also present very early with symptoms of hoarseness. The most common laryngeal cancer is a glottic tumor arising from the true vocal cords. 90% of these tumors are squamous cell carcinomas. This is not a likely cause of this patient’s hoarseness because the laryngoscopy did not demonstrate a laryngeal mass or lesion. Laryngoscopy very readily detects lesions and is a good means for obtaining biopsies for further cytologic studies.

Diabetic neuropathy (choice B) is not the cause of this patient’s symptoms. Most diabetics over time will develop some neurologic involvement due to peripheral vascular disease. Classic signs and symptoms of diabetic neuropathy include numbness and loss of sensation in the distal extremities, particularly in the feet. Diabetic neuropathy is not known to cause damage to the recurrent laryngeal nerve and is thus not a cause of vocal cord paralysis.

Pneumonia (choice D) is not a cause of vocal cord paralysis. Primary lung cancers that occur in the lung apices are the most common cause of injury to the recurrent laryngeal nerve and thus vocal cord paralysis. Pneumonia is an infectious process that does not cause mass effect or damage to the nerve.

Stroke (choice E) can be a cause of vocal cord paralysis when an infarct occurs in the nucleus ambiguus of the medulla. The nucleus ambiguus is the origin of the vagus nerve in the brain. However, if a stroke were to occur in the medulla, there would be many more deficits than a focal vocal cord injury. This patient is presenting with isolated unilateral vocal cord paralysis which means the injury occurred somewhere after the left recurrent laryngeal nerve branched off the vagus nerve.

 

A 45-year-old woman is planning a trip from the United States to Hong Kong on a direct flight. She comes to the office inquiring about advice for any travel precautions that she should take. She has fibrocystic disease of the breast and takes oral contraceptive pills. Physical examination is unremarkable. Her estimated flying time is 18 hours and the total mileage is greater than 10,000. Concerning her risks associated specifically with this flight and her history, the most correct statement is:

  A. She is at increased risk for middle ear damage
  B. She is at increased risk for myocardial infarction
  C. She is at increased risk for tuberculosis
  D. She is at no increased risk for a pulmonary embolism
  E. She is at significantly increased risk for a pulmonary embolism
Explanation:

The correct answer is E. Based upon the classic Virchow triad of stasis, hypercoagulability, and endothelial damage, it had been widely speculated that prolonged air travel can be associated with an increased incidence of deep venous thrombosis and pulmonary embolism. Infact, this risk increases with increasing duration of the flight and for a flight this long, the relative risk is 4.7 fold. In general, any trip longer than 5,000 miles confers at least a 2-fold greater risk. The risk is not increased for trips less than 3,500 miles (choice D).

The pressurization of the cabin, which is required for modern-day air flight can cause damage to the middle ear if people fail to equalize the ambient pressure with the middle ear pressure. However, since this patient has no evidence of a middle ear infection that would cause her eustachian tubes to fail to offer this equalization (choice A), she has no increased risk.

At altitude, airliners routinely have ambient PaO2 of much less than sea level since cabins are not pressurized to one atmosphere. For this reason, arterial PaO2 is around 70 mm Hg. In persons with known coronary disease(choice B), this decrease in supply cannot be met with an increased flow and myocardial ischemia can occur.

Although the ambient air in airliners is recirculated and therefore exposes passengers to “common” pathogens, there is no epidemiological evidence that states that the risk of acquiring tuberculosis (choice C) is any greater than breathing ground level, non-recirculated air.

           

A 79-year-old man who lives in a retirement community is admitted to the hospital with chest pain, shortness of breath, and lethargy. His past medical history is significant for a right upper lobe resection for small cell lung cancer and a stroke many years ago. Since his stroke he has been relatively inactive and is minimally mobile in his wheelchair. His only medications include aspirin and a stool softener. His temperature is 37.0 C (98.6 F), pulse is 120/min, blood pressure is 100/60 mm Hg, and respirations are 24/min. The patient appears to be in moderate distress. Physical examination reveals lungs clear to auscultation, tachycardia without murmurs, and 1+ pitting edema in both lower extremities. He denies any pain in his legs. An electrocardiogram shows sinus tachycardia and a right bundle branch block. Chest x-ray shows mild cardiomegaly, volume loss in the right upper lung field, and bibasilar atelectasis. Arterial blood gas values while breathing room air are:

The most appropriate next step in the evaluation of this patient is

  A. bronchoscopy
  B. contrast venography
  C. pulmonary angiography
  D. ultrasound of both lower extremities
  E. ventilation/perfusion (V/Q) lung scan
Explanation:

The correct answer is E. This patient is presenting with signs and symptoms of pulmonary embolism (PE). His risk factors for PE include prior history of cancer and immobility. A ventilation/perfusion scan is a nuclear medicine study that is most often used for the diagnosis of PE. The demonstration of segments of lung that are ventilated, but not perfused is called a “V/Q” mismatch (V stands for ventilation and Q stands for perfusion). This is highly specific for PE. A ventilation/perfusion scan is the most appropriate diagnostic step in this patient particularly since he has a non-diagnostic chest x-ray. Patients with infiltrates or other parenchymal abnormalities are not ideal candidates for this type of imaging because these abnormalities can limit interpretation of the scan. Helical CT pulmonary angiography is a relatively new diagnostic study which is now commonly used for patients suspected of PE and have abnormal chest x-rays.

Bronchoscopy (choice A) is used for the evaluation of airways. Common indications for bronchoscopy include hemoptysis, chronic cough, and evaluation of central lung lesions identified on chest x-ray or CT of the chest. Bronchoscopy has no role in the evaluation of the pulmonary arteries.

Contrast venography (choice B) is an invasive method of evaluating the veins for thrombosis. This examination has been largely replaced by ultrasonography.

Pulmonary angiography (choice C) is the gold standard in the diagnosis of pulmonary embolism. The most common indication is a patient with an indeterminate or intermediate probability ventilation/perfusion (V/Q) scan who requires a definitive diagnosis. Due to the risks and complications associated with angiography, it is not a study that is performed in the initial work up. It is usually only performed when a definitive diagnosis is not achieved by either V/Q lung scan or helical CT pulmonary angiography.

Ultrasound of the lower extremities (choice D) is an indirect way of evaluating for pulmonary embolism. 90% of PE arise from deep vein thrombosis (DVT) in the lower extremities. Since the treatment of PE and proximal DVT is the same, many clinicians feel that the diagnosis of DVT is an adequate endpoint to decide on treatment. An important criterion in deciding whether or not to perform a lung scan or ultrasound depends on the stability of the patient. This patient is hypotensive, tachycardic, and will benefit from a ventilation/perfusion scan which may show a lack of perfusion to one or both lungs. In the case of a saddle embolus, anticoagulation is not adequate therapy and further intervention with possible thrombectomy is indicated. Ultrasound can be used when a ventilation/perfusion scan is intermediate or indeterminate probability.

 

An 86-year-old woman is brought to the emergency department by her daughter because of a 3-day history of increasing shortness of breath, a cough, and fever. She has had a copious amount of rust-brown sputum, nearly 3 tablespoons per day. She has also had decreased food and water intake over the past few weeks. A chest x-ray shows a right lower lobe pneumonia and laboratory studies show mild hyponatremia most likely from dehydration. She is admitted to the hospital for antibiotic therapy. You are called to examine her because of severe shortness of breath. Her temperature is 39.0 C (101.2 F), blood pressure is 105/70 mm Hg, pulse is 88/min, and respirations are 36/min. She appears to be in respiratory distress. According to the daughter, the patient had just eaten her dinner about 30 minutes ago. You decide that endotracheal intubation is indicated at this time. The most important adjunct maneuver during endotracheal intubation of this patient is

  A. aggressive oral suctioning during laryngoscopy
  B. application of cricoid pressure by an assistant
  C. beginning oral anti-acid therapy prior to intubation
  D. placement of a nasogastric tube for evacuation of the stomach
  E. placement of an orogastric tube for evacuation of the stomach
Explanation:

The correct answer is B. Any patient that has eaten solids or liquids within 8 hours prior to an intubation, or is a trauma patient that has eaten within 12 hours, is considered a high risk patient for aspiration. These patients require, in addition to any other intervention, cricoid pressure to minimize the risk of aspiration. The cricoid pressure must be applied by an assistant since the operator has no free hands with which to perform the maneuver.

Aggressive oral suctioning during laryngoscopy (choice A) is important to assist with visualization of the pharyngeal and laryngeal structures but in fact does little to minimize the risk of aspiration of gastric contents.

For a patient such as this, giving the patient oral anti-acid therapy prior to intubation (choice C) is not practical since she is in distress and will be unable to take oral medications without likely aspirating those.

Placement of a nasogastric tube (choice D) or orogastric tube (choice E) for evacuation of the stomach is a reasonable choice in non-emergent situations. For this patient, having her cooperate in order to place either one of these tubes is not going to be possible and would delay an urgent, if not emergent, intubation. These tubes are often used in the operating room by anesthesiologists to evacuate the stomach once the patient has been sedated.

 

A 29-year-old man is brought to the emergency department because of a severe asthma attack. Two hours ago, he began to wheeze and it has progressively worsened to the point where he is unable to move air. He has suffered from asthma since the age of 7 and has been hospitalized multiple times for asthma flares. His home medications include albuterol MDI, ipratropium bromide MDI, inhaled steroids, and oral theophylline. His temperature is 37.0 (98.6 F), blood pressure is 185/105 mm Hg, pulse is 90/min, and respirations are 90/min. Arterial blood gas on 100% oxygen is PaO2 56 mm Hg, PaCO2 48 mm Hg, pH 7.26. The decision is made to intubate the patient. After laryngoscopy, intubation, cuff inflation, and end-tidal carbon dioxide confirmation, the next step in confirming placement of the tube is to

  A. auscultate for breath sounds
  B. auscultate the stomach
  C. ask the patient to speak
  D. get a chest radiograph
  E. none is required, the tube is appropriately placed
Explanation:

The correct answer is A. The appropriate placement of an endotracheal tube in the trachea is of obvious importance. The recognition however of an inappropriately placed tube is very difficult. For this reason, multiple checks are in place such that all of them combined serve as a nearly 100% sensitive indicator for correct placement. After cuff inflation, most institutions will now check for end-tidal carbon dioxide. Once this is done, since that test is not completely sensitive, auscultation for BILATERAL breath sounds is crucial. This confirms not only tracheal placement, but also evaluates for a possible main stem placement.

Although auscultation of the stomach (choice B) is reasonable, it actually is not required in confirmation of tube placement. Most persons will listen for stomach air, but only after they listen for breath sounds since, if none are present, listening to the stomach would delay reintubation attempts.

Asking the patient to speak (choice C) is not a method to ensure tube placement through the cords. Most patients are either sedated from the intubation attempt or are acutely ill and not able to speak.

A chest radiograph (choice D) is useful to confirm tracheal placement of the tube that is sufficiently proximal to the carina to avoid patient coughing. It is however not a method to ensure tube placement since it requires minutes to perform, develop, and interpret.

Thinking that none is required, the tube is appropriately placed (choice E) is incorrect. Because the end-tidal carbon dioxide is confirmed, this does not guarantee appropriate tube placement. It is the most sensitive indicator of tube placement, but it does have a common false positive, which occurs when air insufflated into the stomach turns the color indicator. You need to auscultate for breath sounds.

 

A 22-year-old man with a history of asthma controlled with albuterol metered dose inhaler (MDI) alone complains of worsening symptoms of shortness of breath. Over the last 10 years he rarely needed his inhaler more than 2 times a month. However, over the last 4 months he reports requiring rescue inhaler treatment 4-5 times a week. A canister of albuterol, which used to last 2-3 months, is now being replaced every 3 weeks. He denies any other symptoms. The most appropriate pharmacotherapy at this time is

  A. cromolyn MDI
  B. ipratropium bromide MDI
  C. montelukast
  D. salmeterol MDI
  E. triamcinolone MDI
Explanation:

The correct answer is E. This patient has worsening of his asthma. Patients should not need albuterol rescue treatments more than 2 times each week. Patients should not wake up at night more than 2 times each month with asthma symptoms, and patients should not need 2 canisters of albuterol in any given month. If your patient fits any of the above criteria, his asthma is poorly controlled. The next treatment is steroid MDI such as triamcinolone. Remember: Asthma is inflammation and steroids decrease inflammation.

Cromolyn (choice A) is a mast-cell stabilizer. It is commonly used in children, but its role in adult asthma is not as well studied as steroids. It is not typically a first- or second-line treatment in asthma of the adult.

Ipratropium bromide (choice B) is an anticholinergic agent which is used for acute treatment of asthma and is helpful in bronchospasm associated with COPD. It is an effective agent in the acute setting of asthma but it takes about 45 minutes to have any effect. It is not the proper drug to be added if rescue inhalers are not providing adequate control of symptoms.

Montelukast (choice C) is a relatively new therapy for asthma. It is a leukotriene inhibitor and is useful in selective patients with asthma such as those with aspirin sensitivity or those that are unable to use MDI properly. Its use should be initiated in select patients on inhaled steroids who continue to have poor control of their asthma symptoms.

Salmeterol (choice D) is a long-acting beta agonist. It is a useful medication to be added to patients who are already on inhaled steroids but continue to have symptoms of asthma.

Remember, regular use of short-acting beta agonist is a sign of poor asthma control.

 

A 52-year-old woman is admitted to the hospital with shortness of breath, a productive cough with “yellowish sputum,” fevers, and chills. She has a medical history significant for non-insulin dependent diabetes and depression for which she takes glyburide and sertraline. She has an allergy to penicillin, to which she gets severe hives. Her temperature is 38 C (100.4 F), blood pressure is 123/67 mmHg, pulse is 102/min, and respirations are 25/min. Her oxygen saturation on room air is 96%. Physical examination shows decreased breath sounds over the lower right lung field with dullness to percussion. A chest radiograph shows consolidation in her right lower lobe. The most appropriate next step is to

  A. obtain a sputum sample for Gram stain and culture
  B. obtain a surgery consult for an open lung biopsy
  C. order a CT scan of the chest
  D. order a ventilation-perfusion scan
  E. send her for bronchoalveolar lavage
Explanation:

The correct answer is A. By history, exam, and chest radiography, this patient has pneumonia. In someone with a pneumonic process, it is advisable to send sputum for Gram stain and culture prior to the initiation of antibiotics in order to ensure that the choice of antibiotic agent will be appropriate. Empiric antibiotics are often started anyway, but it is good to know if the organism that is present is covered by the chosen antibiotic.

An open lung biopsy (choice B) is not used in the routine diagnosis of a lobar pneumonia. It is instead used in the diagnosis of more complex pulmonary pathologies such as the idiopathic interstitial pneumonias that are not amenable to diagnosis using routine methods.

A chest CT (choice C) will not add any additional information at this juncture since a chest radiograph is sufficient to confirm the presence of pneumonia in this patient.

A ventilation-perfusion scan (choice D) is used to diagnose the presence of a pulmonary embolus, which is not a part of the differential diagnosis with this patient. This type of scan is often uninterpretable in the setting of a lobar pneumonia.

A bronchoalveolar lavage (choice E) is not used in the diagnosis of a lobar pneumonia associated with a productive cough.

 

An 83-year-old female nursing home patient is brought to the emergency department after she is found down on her bedroom floor next to her walker. The nursing home staff reports that she appeared confused and disoriented. The patient suffered an embolic stroke 2 years ago, leaving her with residual dysarthria. The patient appears mildly dyspneic and cannot appropriately follow commands. Her temperature is 39.8 C (103.6 F), blood pressure is 110/70 mm Hg, and pulse is 70/min. Laboratory studies show a leukocyte count of 17,000/mm3. A chest x-ray shows a right lower lobe infiltrate. Gram stain of a sputum sample shows many neutrophils and Gram-negative rods. The most appropriate pharmacotherapy is

  A. cefuroxime
  B. clindamycin
  C. erythromycin
  D. levofloxacin
  E. penicillin G
Explanation:

The correct answer is D. The patient is a nursing home resident with a residual neurologic deficit from a stroke that affects her speech. She is found with an altered mental status by the nursing home staff. On evaluation, she has a temperature, elevated WBC, and obvious infiltrate on CXR. She probably an aspiration pneumonia. She is predisposed to aspiration pneumonia due to her stroke, which has affected her speech and likely her ability to swallow. In addition, the CXR infiltrate is in the right lower lobe, which is the likely place for aspirated contents to fall due to anatomy of the bronchi. Finally, she has Gram-negative rods in her sputum. Elderly individuals in long-term care facilities tend to have colonization of the oropharynx with Gram-negative rod bacteria. Levofloxacin has excellent coverage of most pathogens causing aspiration pneumonia and is a preferred antibiotic treatment. If the patient had known Pseudomonas colonization, a ceftazidime or piperacillin may have been better alternative choices.

Cefuroxime (choice A) is the drug of choice for community-acquired pneumonia, but does not have enough Gram-negative coverage in regards to aspiration pneumonias.

Clindamycin (choice B) is another traditional choice for aspiration pneumonia to cover anaerobic organisms. However, without evidence of anaerobic infection such as lung abscess on CXR, necrotizing pneumonia, severe periodontal disease, or putrid sputum, clindamycin is not indicated.

Erythromycin (choice C) covers atypical organisms, not commonly associated with aspiration pneumonia.

Penicillin G (choice E) was the traditional choice for aspiration pneumonia to cover Gram-positive organisms, but recent findings show that Gram-negative organisms predominate in elderly nursing home patients, making this an incorrect choice.

 

A 30-year-old man is brought to the emergency department because of shortness of breath. He had been diagnosed with asthma the previous month, but had not required medication. He has no other medical history, is on no medications, and has no allergies to any medications. He smokes a pack of cigarettes a day and drinks 6 cans of beer a week. He is anxious and is using his accessory muscles of respirations. His blood pressure is 135/88 mm Hg, pulse is 102/min, respiratory rate is 36/min, and room air oxygen saturation is 93%. His pulmonary examination is significant for diffuse expiratory wheezes and a markedly prolonged expiratory phase. He is already receiving supplemental oxygen by face mask. The most appropriate next step is to administer

  A. albuterol by nebulizer
  B. cromolyn, orally
  C. epinephrine, intravenously
  D. montelukast, orally
  E. magnesium, intravenously
Explanation:

The correct answer is A. Inhaled beta agonists such as albuterol constitute the first line of treatment of an asthma exacerbation. Both nebulizer and meter dose inhalations of beta agonists have been shown to work well during an asthma exacerbation.

Cromolyn (choice B), a mast cell stabilizer, is useful in the chronic setting in preventing asthma exacerbations, but has no use in the setting of an asthma exacerbation.

Epinephrine (choice C) is reserved for those cases when bronchospasm is refractory to beta agonists. Given the risk for hypertension and tachyarrythmias, epinephrine is not used as a first-line agent.

Montelukast (choice D), a leukotiene antagonist, as with cromolyn is used in the chronic setting to prevent asthma exacerbations, but has no role in the management of an acute exacerbation.

Magnesium (choice E), presumably through its smooth muscle relaxing properties, has been proposed as an alternative agent in the management of an asthma exacerbation. Studies have, however, found no role for it during an asthma exacerbation.

 

A 49-year-old man comes to the office for a health maintenance examination. He has had 5-7 episodes of blood-tinged sputum in the past month that he is very concerned about. He denies any other symptoms. He has been a patient of yours for 15 years and has been generally healthy. He does not have any chronic medical conditions. He is a superintendent of a large apartment building, gets regular exercise, eats a low-fat diet, and smokes 2 packs of cigarettes a day for the past 30 years. His temperature is 37.0 C (98.6 F), blood pressure is 130/80 mm Hg, pulse is 65/min, and respirations are 16/min. Physical examination is unremarkable. You order a chest x-ray, a complete blood count and coagulation profile, electrolytes, BUN and creatinine, a urinalysis, and send sputum samples for Gram, fungal, and acid-fast stains, cytology and schedule a follow-up visit in 2 weeks. He arrives for the appointment and you review the results with him, all which came back normal. He tells you that he has had increasing episodes of “blood in the sputum” and has even coughed up about 10mL blood over the past 2 weeks. Physical examination and vital signs are unchanged since the last visit. The most appropriate next step is to

  A. admit him to the hospital for immediate thoracic surgery
  B. order a chest x-ray
  C. schedule a bronchoscopy
  D. schedule a high-resolution CT scan
  E. reassure him that all of the tests were normal
Explanation:

The correct answer is C. This patient has non-massive hemoptysis, which is defined as less that 100 mL of expectorated blood over 24 hours. The work-up begins with history and physical examination. A chest x-ray and laboratory studies should be ordered initially. If the chest x-ray is normal and the patient has risk factors for cancer, like smoking 2 pack of cigarettes a day for the past 30 years, a bronchoscopy should be ordered to localize the bleeding site and look for an endobronchial mass. If none is found, a high resolution CT scan should be considered. If a mass is found on a study, referral to a thoracic surgeon is necessary.

It is inappropriate to admit him to the hospital for immediate thoracic surgery (choice A) at this time. He has non-massive hemoptysis and needs a bronchoscopy to be ordered to localize the bleeding site and look for an endobronchial mass. Surgery may be indicated in the future, but now, since he is hemodynamically stable, he needs further evaluation for his hemoptysis.

It is unnecessary to order another chest x-ray (choice B) at this time. Since his physical examination has not changed, it is unlikely that a chest x ray would have changed in 2 weeks. He has no symptoms that suggest pneumonia which may appear on a later x-ray. He requires a bronchoscopy to look for a mass that cannot be visualized on the x-ray.

A high-resolution CT scan (choice D) is usually only indicated after a chest x-ray and a bronchoscopy. An exception is when the chest x-ray suggests parenchymal disease, then the HCRT is usually performed before the bronchoscopy.

In this patient with hemoptysis and a significant smoking history, reassuring him that all of the tests were normal (choice E) is inappropriate. You need to find the cause of his symptoms by continuing with the diagnostic evaluation. A bronchoscopy is indicated at this time.

 

A 49-year-old woman comes to the office because of difficulty breathing, fevers reaching 40 C (104 F), and a productive cough with blood tinged sputum. She was recently diagnosed with ductal carcinoma of the breast and underwent a radical mastectomy with four rounds of adjuvant chemotherapy with vinblastine and doxorubicin. Her last infusion of chemotherapy was 5 days ago. A chest radiograph shows focal infiltrates in both lungs. Laboratory studies show:

You admit her to the hospital and start her on vancomycin and ceftazidime. Over the next 3 days, her clinical status continues to worsen. Blood cultures are negative. A bronchoscopy is performed and biopsy samples are obtained. The biopsy specimen shows septated, branching hyphae that are locally invading tissue. The most appropriate pharmacotherapy at this time is

  A. amphotericin B, intravenously
  B. fluconazole, intravenously
  C. fluconazole, orally
  D. itraconazole, intravenously
  E. rifampin plus isoniazid, orally
Explanation:

The correct answer is A. This is a case of pulmonary aspergillosis in a neutropenic patient after she underwent chemotherapy. The treatment of choice in neutropenic or immunosuppressed patients is amphotericin B intravenously.

Fluconazole (choices B and C) and itraconazole (choice D) are antifungal agents that have limited activity or no activity against aspergillus infections. Also, she should get intravenous therapy at this time.

Rifampin plus isoniazid (choice E) is a combination used to treat mycobacterial infections.

 

A 4-year-old girl with a history of asthma is brought to the emergency department by her mother because of “difficulty breathing.” She has had an upper respiratory infection for several days with low-grade fevers and began coughing last night. Her guardian did not refill the medication that you prescribed 3 months ago, when she had her last “asthma attack,” so your patient has not received any medication with the onset of these symptoms. As you observe your patient and ask her questions as to how she is feeling, you notice that she is unable to finish sentences without becoming short of breath. Her temperature is 38.3 C (101.0 F), pulse is 34/min, and oxygen saturation on room air of 92%. Physical examination shows nasal flaring and subcostal retractions, decreased air entry bilaterally with scattered inspiratory and expiratory wheezing, and a prolonged expiratory phase. The first priority in caring for this patient is to

  A. administer albuterol via metered dose inhaler
  B. administer prednisolone
  C. provide oxygen via face mask
  D. obtain a chest x-ray
  E. prepare to do an arterial blood gas
Explanation:

The correct answer is C. This patient appears to be in moderate respiratory distress and is hypoxic. As part of a pediatric advanced life support protocol, the most important initial interventions should pertain to airway, breathing, and circulation, the “ABCs”. The first intervention for this patient should be to provide oxygen because her room air saturation is below 95%.

As you are providing oxygen, you may prepare to give the patient albuterol, a beta-2 agonist, via nebulizer for early management of her bronchospasm. Albuterol via metered dose inhaler (choice A) is not appropriate in this patient because of the child’s age. Four-year-olds do not have the skill to master a metered dose inhaler in a way that will administer the medication correctly.

Prednisolone (choice B) is a steroid commonly used to treat reactive airway disease. It helps to decrease inflammation. Children having a significant asthma attack should be started on prednisolone early in the course of their management, but not prior to being started on oxygen if hypoxemic.

Obtaining a chest x-ray (choice D) is important in the evaluation of a child with cough and fever. This patient may have a pneumonia, which is exacerbating her reactive airway disease. Classic chest x-ray findings in children with asthma include hyperinflation and peribronchial cuffing. However, a chest x-ray is not an immediate priority in the patient in this vignette.

Preparing to do an arterial blood gas (choice E) is not appropriate in the initial management of the patient in the vignette. This patient is in moderate respiratory distress but has not received any medications or interventions to improve her bronchoconstriction. If this patient does not improve, or begins to become tired, she may require an arterial blood gas to evaluate her pCO2, which looks for carbon dioxide retention.

 

You are called to see a patient in the intensive care unit who was admitted with pneumonia and intubated for hypoxemia and acidosis. You learn that he is a 57-year-old man with severe asthma for which he was being treated with chronic steroids along with multiple inhalers. He has no other medical problems. Currently his temperature is 39 C (102 F), blood pressure is 75/40 mm Hg, pulse is 140/min, and respirations are 17/min. His physical examination is significant for bilateral wheezes with decreased breath sound at the right base. Despite aggressive hydration with normal saline, his blood pressure remains low. You suspect he is septic. You decide to place a Swan Ganz catheter to clarify his volume status and better understand his hemodynamic picture. The most likely finding to support your suspicion that he is septic would be

CI SVR PAWP
  A. Decreased Increased Normal
  B. Decreased Increased Decreased
  C. Increased Decreased Normal
  D. Decreased Normal Normal
  E. Increased Increased Decreased
Explanation:

The correct answer is C. This patient has septic shock. Septic shock is characterized by decreased blood pressure despite euvolemia and is often seen with severe infection, especially with Gram-negative organisms. Release of inflammatory mediators is responsible for decreasing SVR. Patients are thought to be in a hyperdynamic state with increased cardiac output. Volume status is reflected by PAWP and is classically normal but can be normal, low, or high depending on the aggressiveness of hydration.

Decreased CI, increased SVR, and normal PAWP (choice A) is an example of cardiogenic shock. This is common with cardiac tamponade or myocardial infarction. It should be thought of as pump failure. The cardiac index is low because the heart is not pumping well. The SVR is increased in an effort to maintain blood pressure. The PAWP is normal in these patients since volume is typically not the primary problem.

Hypovolemic shock (choice B) is demonstrated by a low cardiac index, an increased SVR, and a decreased PAWP. Simply, CI and PAWP are low because there isn’t enough blood volume to be pumped by the heart (low volume = low pre-load). SVR is elevated to attempt to increase blood pressure.

Obstructive shock, characterized decreased CI and normal SVR and PAWP (choice D), is typically caused by massive pulmonary embolus. Supportive care with IV fluids and vasoconstrictors along with possible embolectomy is indicated. Note: The only indication for embolectomy is hemodynamic instability.

Increased CI and SVR, and decreased PAWP (choice E) is not likely to be seen since Starling principles would suggest that it would be difficult to increase your cardiac output against a high pressure system (increased SVR) and a low intravascular volume (decreased SVR).

 

A 67-year-old man comes to the office complaining of a 12–month history of shortness of breath at rest and with mild exertion. He also reports wheezing on occasion that seems to correspond to the times when he is most short of breath. He has a cough that is persistent most of the year and is occasionally productive of scant sputum. He has a long smoking history of over 100 packs/ year, but has recently quit. His only other medical history is hypertension and hyperlipidemia for which he takes atenolol and simvastatin. A chest radiograph shows hyperinflation but clear lung fields and no evidence of a parenchymal or mediastinal mass. The most appropriate next step in management is to

  A. obtain spirometry
  B. order a chest CT scan
  C. prescribe albuterol inhalers
  D. prescribe corticosteroids
  E. prescribe home oxygen
Explanation:

The correct answer is A. This patient likely has COPD based upon his social history and symptoms. All patients who present complaining of shortness of breath, are over the age of 40, and have a smoking history or environmental exposure history to asbestos, beryllium, or dust, should have screening spirometry. Spirometry measures forced inspiratory and expiratory effort. The hallmark of COPD is decreased forced expiratory effort. The tests are noninvasive and can be performed in fully clothed patients.

A chest CT scan (choice B) is of minimal value in this case since the chest radiograph revealed no evidence of any abnormality except hyperinflation, which is to be expected in a case of COPD.

Therapeutic interventions such as albuterol inhalers (choice C), steroids (choice D), or home oxygen (choice E) are options to be considered once a diagnosis of COPD is established and its severity is quantified. Inhalers are excellent first-line drugs for patients that have reversible airflow obstruction demonstrated by full pulmonary function testing. Home oxygen is usually given for late-stage COPD in patients who have room air arterial oxygen pressures of less than 60 mm Hg. Steroids are very useful agents in acute exacerbations of COPD.

 

You are called to see a 67-year-old woman with severe COPD who was intubated on the medicine floor 30 minutes earlier because of respiratory distress. Her temperature is 37.0 C (98.6 F), blood pressure is 90/60 mm Hg, pulse is 133/min and regular, and respiratory rate is 24/min. Her ABG reveals the following: pH 7.21, PaCO2 80 mm Hg, PaO2 69 mm Hg, and her oxygen saturation is 91%. Her current ventilator settings are assist control (AC), respiratory rate (RR) of 12/min, tidal volume (TV) of 500 ml, fraction of inspired oxygen (FiO2) is 50%, positive end expiratory pressures (PEEP) of 5 mm/Hg. The most appropriate intervention at this time is to

  A. decrease set respiratory rate
  B. increase the fraction of inspired oxygen
  C. increase PEEP
  D. increase tidal volume
  E. make no adjustments at this time
Explanation:

The correct answer is D. The key to this question is to recognize that this patient remains acidotic and hypercarbic despite intubation. In order to “blow off” more carbon dioxide and normalize the blood pH, you must increase minute ventilation (MV). If you remember that MV=TV X RR, you can quickly recognize that the two ways to decrease carbon dioxide is to increase the RR or increase the TV.

Decreasing RR (choice A) will increase carbon dioxide because we are decreasing minute ventilation as described above. This will result in worsening acidosis. In addition, our set respiratory rate is lower than the patient’s actual rate. If we decreased the respiratory rate, our patient would continue to breathe at a high rate and receive the same tidal volume (thereby not even changing the MV).

Increasing FiO2 (choice B) will not be beneficial. First, the patient already has a paO2 of greater than 60 mm Hg. If you remember the hemoglobin binding curve, you should remember oxygen saturations remain above 90% for paO2>60 mm Hg. Therefore, our goal is to keep paO2>60 mm Hg for most patients. Second, maintaining patients on greater than 60% oxygen may have some associated lung toxicity. Therefore, if possible, keeping the inspired oxygen less than 60% is an important part of management. Of course, if high levels of inspired oxygen are necessary, you should not hesitate to use them. Please remember that patients on ventilators need not (and should not) have oxygen saturations of 100% or paO2 much greater than 60 mm Hg.

PEEP (choice C) is the use of positive airway pressures at the end of expiration. PEEP is useful in hypoxic respiratory failure such as ARDS or cardiogenic pulmonary edema. Low levels of PEEP can be used in COPD to keep airways open. Our patient’s oxygenation is acceptable so increasing PEEP would not be beneficial now. In addition, our patients blood pressure is only borderline acceptable. Increasing PEEP will decrease venous return to the heart and might lead to further reductions in blood pressures. High levels of PEEP might also predispose patients to barotrauma which is a form of ventilator induced lung damage.

Making no adjustments (choice E) is not acceptable. This patient is severely acidotic and hypercarbic and should not be left in this state.

 

A 23-year-old man with a childhood history of eczema presents to your office for the first time complaining of a non-productive coughthat started 4 months ago after a respiratory tract infection during the winter. He generally has the cough roughly once or twice a week, usually after strenuous exercise. He has not had any fevers at home and denies any hemoptysis. He smokes socially, roughly 1 pack a week, and binge drinks on the weekends. He denies any intravenous drug use, but has had several unprotected heterosexual relationships this past year. His temperature is 37.0 C (98.6 F), blood pressure is 110/80 mm Hg, pulse is 65/min, and respirations are 15/min. His physical examination is remarkable only for end expiratory wheezes on bilateral lower lung fields. At this time the most correct statement about his condition is:

  A. Bronchoscopy should be performed to rule out opportunistic infection
  B. A chest x-ray is necessary prior to starting any empiric therapy
  C. Inhaled steroids are an appropriate first line agent
  D. Pulmonary function tests would reveal a reduction in the FEV1/FVC ratio
  E. A trial of antibiotics against atypical pathogens such as mycoplasma or chlamydia would relieve this patient’s cough
Explanation:

The correct answer is D. This patient has a classic history for cough variant asthma. The diagnosis of asthma is helped by the history of atopy/eczema. His cough is predominant after exercising, but other allergens (e.g., cigarette smoke, dust, pollen) or cold weather could also induce asthma in many patients. Pulmonary function tests would reveal a decrease in the FEV1/FVC ratio, pathognomonic for obstructive lung disease.

There is no need for bronchoscopy (choice A) since in this patient there is low suspicion for an infectious process. His history of unprotected sexual intercourse does raise the suspicion for HIV, yet opportunistic pulmonary infections such as Pneumocystis Carinii pneumonia would be a late finding when the CD4 counts are less than 200.

A chest x-ray (choice B) would probably be unremarkable, since we have low suspicion for any pulmonic infection. Other causes of pulmonary wheezing and cough could include a foreign body, hypersensitivity pneumonitis, or intrathoracic lung mass. However, these are rarer etiologies and asthma is still primarily a clinical diagnosis.

Inhaled steroids (choice C) may be an appropriate treatment for mild or moderate persistent asthma. However, this patient only coughs roughly once or twice a week, and therefore would be considered to have mild intermittent asthma. An intermittent beta agonist would be the appropriate first line treatment for mild intermittent asthma.

There is no reason to suspect atypical pneumonia (choice E) in this afebrile patient with cough variant asthma.

 

You are called emergently to the medical floor where a 66-year-old man was found to be minimally responsive. His past medical history is unclear but his arm band lists allergies to penicillin and sulfa medications. On arrival, chest compressions are being performed and 2 operators are mask ventilating the patient. Evaluation with an electrocardiogram reveals sinus tachycardia and the diagnosis of pulseless electrical activity is made. Volume is infused and compressions are continued. The patient remains apneic, so mask ventilation continues. During masking, the patient appears to regurgitate large volumes of gastric contents. The most appropriate immediate next step in the management of this patient is to

  A. cease mask ventilation and suction the mouth
  B. continue masking the patient
  C. insert a nasogastric tube
  D. intubate the trachea and suction the airway
  E. intubate the trachea and ventilate
Explanation:

The correct answer is D. Aspiration of gastric contents causes severe lung inflammation. The traditional dogma that the acidic nature of the aspirate is critical has recently been reevaluated and it is now clear that large volumes of gastric contents of any pH are dangerous to the lung. A witnessed aspiration event has the best chances for airway lavage so in these cases an endotracheal tube should be placed, ventilation should be held for a moment, and a flexible suction catheter should be passed so that the airway can be evacuated of gastric contents. By ventilating (choice E) with a tube in the trachea, a 100% effective mechanism for dispersion and aerosolizing the material to the distal airways is occurring. This is to be avoided.

To cease mask ventilation and suction the mouth (choice A) would not address the material that is distal to the larynx that would become aerosolized to the distal airways upon resumption of mask ventilation.

To continue masking the patient (choice B) is the least desirable course of action to take with this patient. Continuing to bellow the material on the pharynx into the larynx and lungs will almost certainly result in severe aspiration disease and death for this patient.

The insertion of a nasogastric tube (choice C) has no role in the management of acute aspiration. The goals are securing the airway and evacuation of the material from the airway prior to any delivery of the aspirate to the distal airways.

 

You are called to intubate a patient on the surgical floor. The patient is postoperative day number 2 after a lumbar disc removal for sciatica. She had an uneventful postoperative course but has a past medical history significant for severe ischemic congestive heart failure. Four hours ago, she began to complain of shortness of breath and has progressively worsened despite diuretic therapy and supplemental oxygen. On arrival to the bedside, her temperature is 37.0 C (98.6 F), blood pressure is 170/105 mm Hg, pulse is 102/min, and respirations are 38/min. Oxygen saturation is 82% on 100% nonrebreathing mask. The patient is somnolent and minimally responsive. After laryngoscopy and insertion of the endotracheal tube, the next most appropriate step is to

  A. auscultate for air in the stomach
  B. auscultate for breath sounds
  C. check for end-tidal carbon dioxide
  D. deliver 100% inspired oxygen and initiate mechanical ventilation
  E. inflate the balloon on the endotracheal tube
Explanation:

The correct answer is E. Although the task of performing laryngoscopy and intubation of the trachea is a skill that requires practice to perfect, the steps immediately after placement of the tube are in fact more critical to the success of the procedure. Once a tube is placed, unless in an infant or small child where cuffless tubes are used, the balloon on the tube must be inflated with at least 3-4cc of air. This completes the protection of the airway from aspiration and seals the airway so that delivered tidal volumes do not “leak” around the tube.

Auscultation for air in the stomach (choice A) or breath sounds (choice B) are appropriate interventions once the cuff is inflated. Ensuring no air is heard in the stomach during delivery of tidal volume (first place to auscultate), followed by bilateral breath sounds, is standard procedure for tube placement verification.

At many institutions, checking for end-tidal carbon dioxide (choice C) with a portable monitor that is placed inline with the ambu-bag is now standard. These detect the presence of carbon dioxide by a color change. These devices are now packaged as part of standard emergency airway trays at many hospitals. However, the tube must be inflated first to seal the airway.

Once the tube is secured and determined to be in the trachea, delivery of 100% inspired oxygen and initiation of mechanical ventilation (choice D) is appropriate.

 

You are present during the intubation of an 87-year-old man who was found unconscious by his nurse. The patient was admitted to the hospital that morning after a syncopal episode. He has a past medical history of coronary artery disease and is status-post a Q-wave myocardial infarction and a 3-vessel bypass procedure 7 months ago. This morning, he had a syncopal episode that was preceded by palpitations and chest pressure. About 10 minutes ago, his nurse heard a fall and came to the room to find the patient on the floor, pulseless and apneic. A senior medicine attending places the endotracheal tube via direct laryngoscopy. The most sensitive method for the detection of an appropriately placed tube is

  A. absence of air heard in the stomach
  B. bilateral breath sounds
  C. detection of end-tidal carbon dioxide
  D. restoration of 100% saturation by pulse oximetry
  E. rise and fall of the chest with positive pressure respirations
Explanation:

The correct answer is C. Placement of an endotracheal tube via laryngoscopy can be a life-saving intervention. Ensuring proper placement of the tube however is critical to the success of the intervention. The most sensitive method of ensuring appropriate placement is colorimetric detection of end-tidal carbon dioxide. These devices are now packaged as part of standard emergency airway trays at many hospitals. A false-positive result can be obtained if a significant amount of air has been insufflated into the stomach and a false negative if the patient has no circulation (no delivery of carbon dioxide to the lungs) or has suffered a massive pulmonary embolism (large dead space).

The absence of air heard in the stomach (choice A) is reassuring, but auscultation is wholly unreliable in ascertaining whether a tube is placed in the esophagus. This is especially true in obese persons.

The presence of bilateral breath sounds (choice B) is reassuring as well, but transmitted breath sounds across the midline are a common cause for a false-positive result.

The restoration of 100% saturation by pulse oximetry (choice D) in no way offers any insight into appropriate placement of an endotracheal tube. If a patient has a large shunt, as for example with a large pneumonia, then the saturation of oxygen may not approach 100%, even with endotracheal intubation and mechanical ventilation. Also, patients can maintain 100% saturations via apneic oxygenation.

The rise and fall of the chest with positive pressure respirations (choice E) is not a good indicator of placement since patients with restrictive disorders of the lung or chest wall or with poor pulmonary compliance may not manifest any chest wall changes during ventilation.

 

A 45-year-old woman is planning a trip from the United States to Hong Kong on a direct flight. She comes to the office inquiring about advice for any travel precautions that she should take. She has fibrocystic disease of the breast and takes oral contraceptive pills. Physical examination is unremarkable. Her estimated flying time is 18 hours and the total mileage is greater than 10,000. Concerning her risks associated specifically with this flight and her history, the most correct statement is:

  A. She is at increased risk for middle ear damage
  B. She is at increased risk for myocardial infarction
  C. She is at increased risk for tuberculosis
  D. She is at no increased risk for a pulmonary embolism
  E. She is at significantly increased risk for a pulmonary embolism
Explanation:

The correct answer is E. Based upon the classic Virchow triad of stasis, hypercoagulability, and endothelial damage, it had been widely speculated that prolonged air travel can be associated with an increased incidence of deep venous thrombosis and pulmonary embolism. Infact, this risk increases with increasing duration of the flight and for a flight this long, the relative risk is 4.7 fold. In general, any trip longer than 5,000 miles confers at least a 2-fold greater risk. The risk is not increased for trips less than 3,500 miles (choice D).

The pressurization of the cabin, which is required for modern-day air flight can cause damage to the middle ear if people fail to equalize the ambient pressure with the middle ear pressure. However, since this patient has no evidence of a middle ear infection that would cause her eustachian tubes to fail to offer this equalization (choice A), she has no increased risk.

At altitude, airliners routinely have ambient PaO2 of much less than sea level since cabins are not pressurized to one atmosphere. For this reason, arterial PaO2 is around 70 mm Hg. In persons with known coronary disease(choice B), this decrease in supply cannot be met with an increased flow and myocardial ischemia can occur.

Although the ambient air in airliners is recirculated and therefore exposes passengers to “common” pathogens, there is no epidemiological evidence that states that the risk of acquiring tuberculosis (choice C) is any greater than breathing ground level, non-recirculated air.

 

A 60-year-old man comes to the emergency department because of shortness of breath. He complains of a dry cough, but denies any fever, chills, or sweats. His past medical history is significant for a history of chronic obstructive pulmonary disease (COPD), hypertension, and alcoholism. His medications include an albuterol inhaler and furosemide. He appears to be in moderate respiratory distress. His temperature is 37.0 C (98.6 F), blood pressure is 146/98 mm Hg, pulse is 120/min, and respiratory rate is 36/min. His oxygen saturation on room air is 89%. His breath sounds are diminished bilaterally and he has diffuse wheezes. The remainder of the physical examination is unremarkable. A chest radiograph shows hyperexpanded lungs. An electrocardiogram shows sinus tachycardia. The most appropriate next diagnostic study is

  A. arterial blood gas analysis
  B. chest CT scan
  C. echocardiogram
  D. venous blood gas analysis
  E. ventilation-perfusion scan
Explanation:

The correct answer is A. In a patient with a history of chronic obstructive pulmonary disease (COPD), the constellation of described historical and physical findings with a chest radiograph showing no acute pathology indicates a COPD exacerbation. An arterial blood gas analysis, especially in the setting of a room air oxygen saturation of 89%, will more clearly define the patient’s oxygenation and ventilation status and assist in better management and triage.

A chest CT scan (choice B) in the setting of a chronic obstructive pulmonary disease exacerbation associated with a negative chest radiograph cannot be expected to provide additional useful information.

An echocardiogram (choice C) will offer no useful information since the patient’s respiratory distress, based upon the available history and physical exam, is due to an exacerbation of his chronic obstructive pulmonary disease.

A venous blood gas analysis (choice D) cannot provide any information regarding systemic oxygenation. It’s utility in this setting, therefore, is minimal.

Since there is no reason to suspect a pulmonary embolus, a ventilation-perfusion scan (choice E) will not provide any useful information in this instance.

 

A 32-year-old African American woman returns to the clinic for a follow-up visit. She was seen 2 weeks prior with complaints of dyspnea, dry cough, chest pain, and tightness of the chest. An electrocardiogram was within normal limits. A chest x-ray was performed a week later which showed bilateral hilar lymphadenopathy with pulmonary infiltrate. While in clinic now, she points out a rash on her nasal tip that has been present for a number of months. There is a 4 x 2 cm violaceous, indurated plaque involving the nasal tip extending to the bilateral ala. There are a few tiny button-like papules in the center of the plaque. In addition, there are waxy, translucent lesions with flat tops on the face, lids, around the orbits, and in the nasolabial folds. The most appropriate next step in evaluation is to

  A. determine serum angiotensin-converting enzyme and serum calcium levels
  B. determine serum CEA marker level
  C. order a complete blood count
  D. refer her for a bronchoscopy
  E. send her for a lung biopsy
Explanation:

The correct answer is A. Determining serum angiotensin-converting enzyme (ACE) and serum calcium is the correct management for this patient with suspected sarcoidosis. Although a bronchoscopy (choice D) and a lung biopsy (choice E) will also help with diagnosis of sarcoidosis, serum ACE and calcium levels are less invasive tests to obtain for diagnosis. Sarcoidosis involves multisystems including lungs, eyes, peripheral lymph nodes, spleen, gastrointestinal tract, hearing, and musculoskeletal systems. Hypercalcemia may occur in any stage of sarcoidosis. Corticosteroids lower the raised calcium level to normal by inhibiting the peripheral action of 1,25(OH)2D3 and by metabolizing the compound to an inactive metabolite. The serum ACE level is also raised in 60% of patients. ACE activity is higher in patients with hilar adenopathy and pulmonary infiltration.

A complete blood count (choice C) is incorrect, because hemolytic anemia is rare in sarcoid and leukopenia alone is too non-specific for diagnosis of sarcoid.

Serum CEA level (choice B) is incorrect, because this is a non-specific tumor marker used to monitor for colon carcinoma recurrence, as well as some lung adenocarcinoma responses to chemotherapy.

 

A 21-year-old college student comes to the emergency department because of a 30-minute history of difficulty in breathing, and a sharp left-sided chest pain that came on suddenly when he was walking back to his dormitory after biology class. He says that he is generally very healthy and has never experienced anything like this in the past. He is 188 cm (6 ft 2 in) tall and weighs 70 kg (154 lb). His temperature is 37.0 C (98.6 F), blood pressure is 120/80 mm Hg, pulse is 75/min, and respirations are 22/min. Physical examination shows decreased breath sounds, decreased tactile fremitus, and increased resonance to percussion on the left side. The cardiac examination is normal. You order a chest x-ray and go to see the next patient. You hear a radiology technician call for help as you finally get to sit down to write your notes. You run over to the patient and find that it is your “shortness of breath and chest pain” patient and that he has not had the chest x-ray yet. He is now cyanotic and has severe dyspnea. Examination shows tracheal deviation and distended neck veins. The most appropriate immediate management is

  A. draw an arterial blood gas
  B. endotracheal intubation
  C. insert a chest tube on the left side
  D. insert a needle into the left 2nd intercostal space
  E. obtain a chest x-ray at the bedside
Explanation:

The correct answer is D. This patient has developed a tension pneumothorax, which is an emergency that requires immediate intervention with a needle on the side of the pneumothorax. He presented with a spontaneous pneumothorax, that progressed to a tension pneumothorax, and decompression to relieve the increased intrathoracic pressure is the first step in management. Young, thin men are at an increased risk for developing a spontaneous pneumothorax, and the exact cause is unknown.

Blood gases (choice A) are not the most important next step in management of this case. He requires a needle into the left 2nd intercostal space, which is a life-saving intervention in a patient with a tension pneumothorax. Blood gases may be drawn later on, after the needle and then the chest tube are inserted.

Endotracheal intubation (choice B) is the treatment for a flail chest, which presents with paradoxical chest wall motion, splinting, and crepitus of rib fragments. The patient in this case has a tension pneumothorax, not a flail chest.

A chest tube (choice C) is inserted after the needle is placed into the left 2nd intercostal space. By the time all of the required chest tube equipment is put together, a patient with a tension pneumothorax may die. Therefore, a needle insertion is the quickest life-saving procedure for this patient.

A chest x-ray (choice E) is not required to make the diagnosis of a tension pneumothorax. The diagnosis is clinical and a chest x-ray will only waste precious time.

 

A 45-year-old woman with severe reflux disease secondary to a hiatal hernia is admitted to the hospital with flank pain from a kidney stone. An abdominal CT shows multiple stones in the right ureter and renal pelvis. On the floor, she is given intramuscular meperidine every 4 hours for pain control. Early in the morning the patient is found to be obtunded in moderate respiratory distress with some evidence of vomitus on her lips and bed shirt. She had been given 3 additional doses of meperidine for pain control in the past 5 hours. A chest radiograph will most likely show a

  A. diffuse bilateral airspace disease
  B. diffuse bilateral interstitial infiltrates
  C. right lower lobe opacification
  D. right pleural effusion
  E. widened mediastinum
Explanation:

The correct answer is C. Aspiration of gastric contents causes severe lung inflammation. The traditional dogma that the acidic nature of the aspirate is critical has recently been reevaluated and it is now clear that large volumes of gastric contents of any pH are dangerous to the lung. Patients with severe reflux often regurgitate frequently throughout the day and at night will have small aspiration events, which will wake them from sleep by coughing. Once sedated, these people develop depressed cough reflexes and therefore are more likely to be unable to protect their airway during such regurgitations. This is most certainly what has occurred with this patient. The most common radiological finding is right lower lobe opacification (alveolar filling) or collapse.

Diffuse bilateral airspace disease (choice A) is characteristic of acute respiratory distress syndrome (ARDS) or very late stage aspiration which can lead to ARDS.

Diffuse bilateral interstitial infiltrates (choice B) are characteristic of pulmonary edema. This may be a late manifestation (a few days) of severe aspiration, but not an early one.

Pleural effusions (choice D) are not present in aspirations. A unilateral effusion can be found in cases of liver abscess or right sided diaphragmatic irritation or with Meigs syndrome (ovarian cancer and ipsilateral pleural effusion).

A widened mediastinum (choice E) is characteristic of an aortic arch dissection or of a pulmonary disease such as sarcoid.

 

A 45-year-old man with a history of recurrent deep venous thrombosis and known to have the factor V Leiden mutation comes to the emergency department because of an abrupt onset of severe dyspnea that started while he was walking to work. He also complains of sharp chest pain on inspiration and says that he has coughed up small amounts of blood. He is not currently taking any medications and he has no known drug allergies. His temperature is 37.8 C (100.1 F), his blood pressure is 100/60 mm Hg, pulse is 110/min, respirations are 30/min, and his oxygen saturation is 91% on room air. He appears mildly anxious and is clearly tachypneic, but he is able to speak in full sentences. Cardiac examination reveals a prominent P2. His lungs are clear. His right lower extremity has 2+ pretibial edema and his left lower extremity has no edema. A chest x-ray is unremarkable. A ventilation/ perfusion lung scan is consistent with a high probability for bilateral pulmonary emboli. The next most appropriate step in management is to

  A. administer intravenous unfractionated heparin and warfarin; discontinue the heparin as soon as the INR is therapeutic
  B. administer intravenous unfractionated heparin and warfarin; discontinue the heparin 2 days after a therapeutic INR is achieved
  C. begin therapy with subcutaneous low molecular weight heparin and warfarin and send the patient home to follow up in anticoagulation clinic in 1 week
  D. obtain an ultrasound of his right lower extremity to rule out deep venous thrombosis
  E. perform a CT angiogram to confirm the diagnosis of pulmonary embolism
Explanation:

The correct answer is B. In the setting of a high pretest probability for pulmonary embolism (as in this case with a patient who is known to be hypercoagulable and presents with classic symptoms), a high probability ventilation/perfusion scan is sufficient to make the diagnosis. Ventilation/perfusion scans may be read as negative, but are otherwise described in terms of the probabilities (low, intermediate, or high) of pulmonary embolism. The patient should be immediately anticoagulated with heparin to prevent further progression of thrombosis. Studies have demonstrated that low molecular weight heparin and unfractionated heparin have similar efficacies in this setting. Given the severity of his symptoms and abnormal vital signs, the patient should not be sent home until he has been monitored in the hospital and his condition stabilizes. This patient should be started on intravenous unfractionated heparin and warfarin and the heparin should be discontinued 2 days after a therapeutic INR is achieved.

Start intravenous unfractionated heparin and warfarin; discontinue the heparin as soon as the INR is therapeutic (choice A) is incorrect because the heparin must overlap with the warfarin for at least 2 days after the INR is therapeutic. Warfarin inhibits the synthesis of factors II, VII, IX, and X. The initial increase in INR seen with warfarin is due to inhibition of the factor with the shortest half-life, factor VII (the half life is approximately 7 hours). The antithrombotic effect of warfarin is thought to rely mainly on inhibition of factor II. Due to its longer half life, it can take up to 2 days for factor II levels to sufficiently decline. Therefore, if the heparin is discontinued as soon as the INR is therapeutic, the patient will be left essentially un-anticoagulated for 1-2 days.

Beginning subcutaneous low molecular weight heparin and warfarin and sending the patient home to follow up in anticoagulation clinic in 1 week (choice C) is incorrect due to the reasons described above. While low molecular weight heparin has similar efficacy to unfractionated heparin in this setting, there is no data to suggest nor any consensus that patients with pulmonary embolism can safely be treated as outpatients. This patient’s tachycardia, low blood pressure, high respiratory rate, and low oxygen saturation warrants cardiac monitoring and an inpatient stay.

Obtaining an ultrasound of his right lower extremity to rule out deep venous thrombosis (choice D) is incorrect. The patient already has a known history of deep venous thrombosis. While the physical finding of asymmetric lower extremity edema is strongly suggestive of a deep venous thrombosis in the right lower extremity, knowing this for certain would not change management. The patient will receive anticoagulation for his pulmonary embolism regardless of any findings on lower extremity ultrasound.

Performing a CT angiogram to confirm the diagnosis of pulmonary embolism (choice E) is incorrect because a CT angiogram is neither more sensitive nor more specific than ventilation/perfusion lung scan. Pulmonary angiogram (via fluoroscopy, not computed tomography) is the gold standard to make the diagnosis of pulmonary embolism. As noted above, a high probability scan is sufficient to make the diagnosis of pulmonary embolism in this setting. The PIOPED study provides data on ventilation/perfusion lung scanning for the diagnosis of pulmonary embolism, validating its use. While a CT angiogram is commonly used in practice, its routine use has not yet been validated in large trials.

 

 

A 72-year-old smoker is admitted to the hospital for COPD exacerbation. Admission vitals are respirations 18/min, with a blood pressure of 180/100 mm Hg, and an oxygen saturation of 91%. He is started on nebulized albuterol and ipratropium bromide, as well as prednisolone intravenously. Admission chest radiograph reveals flattened hemidiaphragms, increased retrosternal clear space, and hyperlucent lungs. Given a suspicion of pulmonary embolus, a ventilation perfusion scan is performed demonstrating nonsegmental perfusion defects of the left upper lobe, with a small left lung and a complete absence of perfusion and ventilation of the entire right lung. The patient becomes acutely short of breath in the nuclear medicine department. His respirations are 30/min with otherwise normal vital signs. After supplemental oxygen (4 L/min by nasal cannula) and nebulizers are administered, the respirations become 29/min, with a blood pressure of 80/40 mmHg, and an oxygen saturation of 82%. A repeat chest radiograph is pending. The most appropriate management is to

  A. administer heparin, intravenously
  B. insert a chest tube on the left side
  C. insert a chest tube on the right side
  D. obtain a surgical consult for emergent lung volume reduction surgery
  E. send him for coronary artery catheterization
Explanation:

The correct answer is C. The patient is exhibiting clinical signs of a tension pneumothorax, including pulmonary and cardiac failure. The ventilation perfusion scan demonstrates lack of ventilation and perfusion of the right lung, which is consistent with a pneumothorax. A tension pneumothorax must be suspected given the diminished size of the left lung. A tension pneumothorax is a unilateral pneumothorax that becomes loculated by a one-way valve mechanism and compromises the contralateral lung and the venous return to the chest. Diagnosis is made by the lack of ipsilateral lung sounds due to cardiopulmonary collapse or chest radiograph. Treatment is immediate chest tube insertion to relieve the pressure in the right hemithorax.

The ventilation perfusion scan is consistent with a pneumothorax of the right lung with signs of tension, given the small left lung. The ventilation perfusion mismatches of the left lung apex are consistent with bullous disease, which is common in patients with COPD. A pulmonary embolus creates ventilation perfusion mismatches on the ventilation perfusion scan. Heparin would be an appropriate treatment if the ventilation perfusion scan was positive for pulmonary embolus (choice A).

The chest tube must be ipsilateral to the tension pneumothorax, not on the left side (choice B).

Volume reduction surgery (choice D) is a controversial method for reducing lung volumes in patients with emphysema. It is an elective procedure and is inappropriate for the management of tension pneumothorax.

The patient’s low blood pressure is due to the effect of the tension pneumothorax on preload, not due to intrinsic coronary artery disease as seen in a cardiac catheterization (choice E).

 

You are called to see a patient in the intensive care unit who was admitted with pneumonia and intubated for hypoxemia and acidosis. You learn that he is a 57-year-old man with severe asthma for which he was being treated with chronic steroids along with multiple inhalers. He has no other medical problems. Currently his temperature is 39 C (102 F), blood pressure is 75/40 mm Hg, pulse is 140/min, and respirations are 17/min. His physical examination is significant for bilateral wheezes with decreased breath sound at the right base. Despite aggressive hydration with normal saline, his blood pressure remains low. You suspect he is septic. You decide to place a Swan Ganz catheter to clarify his volume status and better understand his hemodynamic picture. The most likely finding to support your suspicion that he is septic would be

CI SVR PAWP
  A. Decreased Increased Normal
  B. Decreased Increased Decreased
  C. Increased Decreased Normal
  D. Decreased Normal Normal
  E. Increased Increased Decreased
Explanation:

The correct answer is C. This patient has septic shock. Septic shock is characterized by decreased blood pressure despite euvolemia and is often seen with severe infection, especially with Gram-negative organisms. Release of inflammatory mediators is responsible for decreasing SVR. Patients are thought to be in a hyperdynamic state with increased cardiac output. Volume status is reflected by PAWP and is classically normal but can be normal, low, or high depending on the aggressiveness of hydration.

Decreased CI, increased SVR, and normal PAWP (choice A) is an example of cardiogenic shock. This is common with cardiac tamponade or myocardial infarction. It should be thought of as pump failure. The cardiac index is low because the heart is not pumping well. The SVR is increased in an effort to maintain blood pressure. The PAWP is normal in these patients since volume is typically not the primary problem.

Hypovolemic shock (choice B) is demonstrated by a low cardiac index, an increased SVR, and a decreased PAWP. Simply, CI and PAWP are low because there isn’t enough blood volume to be pumped by the heart (low volume = low pre-load). SVR is elevated to attempt to increase blood pressure.

Obstructive shock, characterized decreased CI and normal SVR and PAWP (choice D), is typically caused by massive pulmonary embolus. Supportive care with IV fluids and vasoconstrictors along with possible embolectomy is indicated. Note: The only indication for embolectomy is hemodynamic instability.

Increased CI and SVR, and decreased PAWP (choice E) is not likely to be seen since Starling principles would suggest that it would be difficult to increase your cardiac output against a high pressure system (increased SVR) and a low intravascular volume (decreased SVR).

 

A 72-year-old woman with a 6-month history of non-small cell lung cancer comes to the office because of neck and facial swelling. She denies any shortness of breath or hemoptysis. Physical examination shows dilated neck veins and edema of the face and right arm. A CT scan of the chest shows a right paratracheal mass with diminished opacification of the central venous structures. The most appropriate next step in the management of this patient is to

  A. administer dexamethasone every 6 hours
  B. begin chemotherapy
  C. biopsy the mass
  D. give her intravenous morphine
  E. recommend radiation therapy
Explanation:

The correct answer is E. The patient has a classic case of superior vena cava syndrome (SVCS), which is due to obstruction of the superior vena cava. The vast majority of cases of SVCS are caused by malignancies, with lung cancer being the most common. The most feared complication of SVCS is upper airway obstruction. Radiation therapy is the treatment of choice for most patients with SVCS.

Corticosteroids, such as dexamethasone, (choice A) are not the primary treatment of SVCS. In some malignancies which are steroid responsive or if there is significant inflammation, steroids can be started as an adjunct to radiation therapy.

Chemotherapy (choice B) is not the initial treatment of choice for patients with non-small lung cancer who present with SVCS. After the patient has been started on radiation, a chemotherapeutic regimen can be offered to the patient if it is appropriate.

In patients with known lung cancer, a biopsy of the mass (choice C) causing the SVCS is usually not necessary and treatment can commence once the clinical diagnosis is made. In patients without a history of cancer, every effort should be made to obtain a diagnosis before starting treatment, as there are benign causes of SVCS (e.g., thyroid enlargement, thrombosis).

Narcotics (choice D) are not direct therapy for SVCS. Of course if the patient has any significant pain due to their cancer then narcotics should be prescribed to help alleviate it. This patient is not complaining of any pain by history but like any cancer patient she should be asked directly whether she is experiencing any pain or discomfort.

 

A 67-year-old man comes to the clinic for an initial visit. He and his wife have just moved to the area from out of the state. He brought along his medical records which show that he has hypertension, mild peripheral vascular disease, and that he carries the diagnosis of emphysema. He tells you that he smokes 1 pack of cigarettes per day but refrains from all but social alcohol. His medications include thiazide, captopril, quinine, and albuterol inhalers as needed. He has never had pulmonary function testing. His temperature is 37.0 C (98.6 F), blood pressure is 135/85 mm Hg, pulse is 72/min, and respirations are 14/min. He has diffuse bilateral expiratory wheezes with a mildly prolonged expiratory time. His abdomen is obese, but non-tender and there is no fluid wave. The most appropriate intervention for this patient is to

  A. change captopril to lisinopril
  B. encourage him to quit smoking immediately
  C. increase his thiazide diuretic dose
  D. initiate home oxygen therapy
  E. obtain pulmonary function testing
Explanation:

The correct answer is B. The two interventions that have been shown to affect mortality, smoking cessation and oxygen therapy, should be foremost in the minds of all caregivers who manage patients with COPD. At any stage of the disease, smoking cessation is the most important intervention that can be taken to improve lifestyle and longevity. Other management strategies such as medications, rehabilitation, and even surgery are less effective, sometimes ineffective, when smoking is still practiced.

Changing ACE inhibitors from captopril to lisinopril, a 3 times per day drug to a once daily drug (choice A) is not required for this patient. Since this is a compliance/lifestyle issue, it should not take priority of a critical medical intervention. There is no difference in efficacy between the two drugs.

This patient has reasonably controlled blood pressure so that increasing his thiazide diuretic dose (choice C) may improve his blood pressure slightly, but its benefits are minimal when compared to those obtained with smoking cessation.

The decision to initiate home oxygen therapy (choice D) is based upon arterial oxygen pressures of less than 55 mm Hg (saturations less than 88%) and therefore a resting arterial blood gas is required before a decision to initiate long-term oxygen therapy is made.

Pulmonary function testing (choice E) is an important tool to stratify patients with COPD and to determine if they have an element of reversible bronchoconstriction. Since they are not therapeutic however, they do not take precedence over immediate medical interventions that have life-prolonging consequences.

 

A 31-year-old woman comes to the office for a follow-up visit. Two weeks ago, the patient underwent an echocardiogram for the evaluation of a systolic murmur. Her valves appeared normal but the echocardiogram disclosed elevated right ventricular systolic and diastolic pressures consistent with pulmonary hypertension. She has no primary lung disease and reports no symptoms of dyspnea or tachypnea. Her other past medical history is unremarkable and she takes only oral contraceptive pills for medications. The most appropriate next step in the management of this patient is to

  A. prescribe calcium channel blockers, orally
  B. prescribe nitric oxide, inhaled
  C. prescribe prostaglandin, intravenously
  D. refer the patient for oxygen diffusion capacity testing
  E. refer the patient for vasodilator response testing
Explanation:

The correct answer is E. The management of patients with pulmonary hypertension focuses on three issues: is the disease secondary to primary pulmonary disease (secondary pulmonary hypertension), is the patient responsive to vasodilator therapy, and, can the pulmonary pressures be made normal with medication. For this patient, the assumption is, given her age and lack of medical history, that her pulmonary hypertension is primary. In addition to pulmonary function testing to help verify this assumption, vasodilator testing to determine whether the pulmonary vasculature is responsive or not is the first step in the management of such patients.

Calcium channel blockers (choice A), inhaled nitric oxide (choice B) or intravenous prostaglandin (choice C) are all agents used in the management of primary pulmonary hypertension. The choice of these agents depends on whether the patient is vasodilator responsive and the side effect profile. Systemic agents such as calcium antagonists and prostaglandins are associated with often profound systemic hypotension which limits their utility is normalizing pulmonary pressures.

Referring the patient for oxygen diffusion capacity testing (choice D) is not necessary. Standard spirometry as part of a full battery of pulmonary function tests are indicated to assess whether the hypertension is primary or secondary, but diffusion testing is only useful in cases where hypoxemia coexists with existing pulmonary disease.

 

A 30-year-old man is brought to the emergency department because of shortness of breath. He had been diagnosed with asthma the previous month, but had not required medication. He has no other medical history, is on no medications, and has no allergies to any medications. He smokes a pack of cigarettes a day and drinks 6 cans of beer a week. He is anxious and is using his accessory muscles of respirations. His blood pressure is 135/88 mm Hg, pulse is 102/min, respiratory rate is 36/min, and room air oxygen saturation is 93%. His pulmonary examination is significant for diffuse expiratory wheezes and a markedly prolonged expiratory phase. He is already receiving supplemental oxygen by face mask. The most appropriate next step is to administer

  A. albuterol by nebulizer
  B. cromolyn, orally
  C. epinephrine, intravenously
  D. montelukast, orally
  E. magnesium, intravenously
Explanation:

The correct answer is A. Inhaled beta agonists such as albuterol constitute the first line of treatment of an asthma exacerbation. Both nebulizer and meter dose inhalations of beta agonists have been shown to work well during an asthma exacerbation.

Cromolyn (choice B), a mast cell stabilizer, is useful in the chronic setting in preventing asthma exacerbations, but has no use in the setting of an asthma exacerbation.

Epinephrine (choice C) is reserved for those cases when bronchospasm is refractory to beta agonists. Given the risk for hypertension and tachyarrythmias, epinephrine is not used as a first-line agent.

Montelukast (choice D), a leukotiene antagonist, as with cromolyn is used in the chronic setting to prevent asthma exacerbations, but has no role in the management of an acute exacerbation.

Magnesium (choice E), presumably through its smooth muscle relaxing properties, has been proposed as an alternative agent in the management of an asthma exacerbation. Studies have, however, found no role for it during an asthma exacerbation.

 

A 53-year-old man is admitted to the hospital because of rapid onset of shortness of breath. He reports that a little less than 2 weeks ago he noticed that he was short of breath and since that time it has progressed to the point where at rest, he is barely able to breath, and he is unable to walk without “nearly passing out.” He denies chest pain, pressure, any altered mental status, cough, or fever. His past medical history is remarkable only for hypertension treated with atenolol. The patient denies any recent travel, occupational exposures, or sick contacts. On arrival to the emergency department, the patient is mildly cyanotic and breathing at 24-28/min. He is conversant and appropriate, but visibly short of breath. There are no obvious signs of accessory muscle engagement. His room air oxygen saturation is 82%. The most appropriate management of this patient at this time is to

  A. administer heliox
  B. administer high flow oxygen via non-rebreathing mask
  C. administer 3 liters/min oxygen via nasal prongs
  D. administer 3 liters/min oxygen via simple face mask
  E. perform endotracheal intubation
Explanation:

The correct answer is B. This patient has severe hypoxia of unknown etiology. The nature of his illness and rapid course suggests a disease such as interstitial pulmonary fibrosis. Most rapid cases such as this are idiopathic (Hammond-Rich syndrome). Regardless of the cause, immediate management is the same, provide adequate oxygen to determine if the hypoxia can be corrected. High flow oxygen delivered via NRB mask offers about 82-86% inspired oxygen concentration. If the shunt fraction is less than 50%, inspired oxygen of this amount will be able to correct the hypoxia. The patient can then continue on oxygen until he can no longer protect his airway, his work of breathing becomes too great, or he begins to desaturate.

Heliox (choice A) is a mixture of helium and oxygen that is used in patients with severe bronchoconstriction. The combination gas is more laminar with its’ flow and allows better delivery of oxygen to the distal airways. It has no role in the correction of hypoxia since it is a low oxygen concentration mixture.

Oxygen via nasal prongs (choice C) is inadequate for this patient. 3 L/min offered in this manner is essentially 26-28% inspired oxygen concentration. With this marginal escalation over ambient tensions, the patient will improve minimally, or more likely, not at all.

In order to use a simple face mask for oxygen delivery (choice D) the flows need to be greater than 6 L/min in order to effectively evacuate the expired carbon dioxide from the mask and prevent rebreathing.

There is no indication to place an endotracheal tube at this time (choice E). Although the patient is exerting tremendous effort to breathe, he is not in distress, has no accessory muscle use, and is not discoordinate. The first attempt at management should be to determine if oxygen, delivered via external devices, can augment his oxygenation. If this is successful, his respiratory rate will decline and his work of breathing will decrease substantially.

 

A 4-year-old girl is brought to the emergency department because of a sudden episode of drooling and coughing that began when the babysitter left the patient alone in the kitchen for 1-2 minutes. She is generally very healthy and all of her vaccinations and routine preventive care measures are current. Vital signs are temperature 37.2 C (99 F), pulse 90/min, blood pressure 100/50 mm Hg, and respirations 16/min. The physical examination reveals a well-developed girl in mild respiratory distress with mild stridor. Notably, there is nothing in the mouth and the lungs are clear on auscultation. A chest x-ray is normal. Twenty minutes later the patient is still in mild respiratory distress but is improving. The oxygen saturation on room air is 97%. The next step would be to

  A. begin amoxicillin therapy
  B. begin levofloxacin therapy
  C. order a C reactive protein laboratory test
  D. order a lateral x-ray of the neck
  E. perform endotracheal intubation
Explanation:

The correct answer is D. When young children have acute onset of drooling and coughing, a foreign body aspiration must be suspected. Alternative diagnoses of croup or epiglottitis should also be considered. In this case a coin was found in the posterior oropharynx. Radiographic evaluation of the neck, chest, and abdomen is necessary to exclude a foreign body. Some objects such as hard food may not be radiopaque, so endoscopy is sometimes necessary.

Antibiotic therapy (choice A and B) should not be pursued, as there are no clinical signs of infection at this time. A lateral x-ray of the neck is necessary first to evaluate for epiglottitis.

C reactive protein (choice C) is a nonspecific indicator of inflammation and would not be helpful in this case. Foreign body aspiration must be suspected and alternative diagnoses of croup or epiglottitis must also be considered.

Endotracheal intubation (choice E) may be undertaken in the case of respiratory failure or to protect the airway. This patient is not symptomatic of respiratory failure, but is certainly at risk of losing the airway. Preparation for intubation must be made, but is not necessary at this time. Foreign body aspiration must be suspected and alternative diagnoses of croup or epiglottitis must also be considered.

 

An 83-year-old female nursing home patient is brought to the emergency department after she is found down on her bedroom floor next to her walker. The nursing home staff reports that she appeared confused and disoriented. The patient suffered an embolic stroke 2 years ago, leaving her with residual dysarthria. The patient appears mildly dyspneic and cannot appropriately follow commands. Her temperature is 39.8 C (103.6 F), blood pressure is 110/70 mm Hg, and pulse is 70/min. Laboratory studies show a leukocyte count of 17,000/mm3. A chest x-ray shows a right lower lobe infiltrate. Gram stain of a sputum sample shows many neutrophils and Gram-negative rods. The most appropriate pharmacotherapy is

  A. cefuroxime
  B. clindamycin
  C. erythromycin
  D. levofloxacin
  E. penicillin G
Explanation:

The correct answer is D. The patient is a nursing home resident with a residual neurologic deficit from a stroke that affects her speech. She is found with an altered mental status by the nursing home staff. On evaluation, she has a temperature, elevated WBC, and obvious infiltrate on CXR. She probably an aspiration pneumonia. She is predisposed to aspiration pneumonia due to her stroke, which has affected her speech and likely her ability to swallow. In addition, the CXR infiltrate is in the right lower lobe, which is the likely place for aspirated contents to fall due to anatomy of the bronchi. Finally, she has Gram-negative rods in her sputum. Elderly individuals in long-term care facilities tend to have colonization of the oropharynx with Gram-negative rod bacteria. Levofloxacin has excellent coverage of most pathogens causing aspiration pneumonia and is a preferred antibiotic treatment. If the patient had known Pseudomonas colonization, a ceftazidime or piperacillin may have been better alternative choices.

Cefuroxime (choice A) is the drug of choice for community-acquired pneumonia, but does not have enough Gram-negative coverage in regards to aspiration pneumonias.

Clindamycin (choice B) is another traditional choice for aspiration pneumonia to cover anaerobic organisms. However, without evidence of anaerobic infection such as lung abscess on CXR, necrotizing pneumonia, severe periodontal disease, or putrid sputum, clindamycin is not indicated.

Erythromycin (choice C) covers atypical organisms, not commonly associated with aspiration pneumonia.

Penicillin G (choice E) was the traditional choice for aspiration pneumonia to cover Gram-positive organisms, but recent findings show that Gram-negative organisms predominate in elderly nursing home patients, making this an incorrect choice.

 

An 83-year-old female nursing home patient is brought to the emergency department after she is found down on her bedroom floor next to her walker. The nursing home staff reports that she appeared confused and disoriented. The patient suffered an embolic stroke 2 years ago, leaving her with residual dysarthria. The patient appears mildly dyspneic and cannot appropriately follow commands. Her temperature is 39.8 C (103.6 F), blood pressure is 110/70 mm Hg, and pulse is 70/min. Laboratory studies show a leukocyte count of 17,000/mm3. A chest x-ray shows a right lower lobe infiltrate. Gram stain of a sputum sample shows many neutrophils and Gram-negative rods. The most appropriate pharmacotherapy is

  A. cefuroxime
  B. clindamycin
  C. erythromycin
  D. levofloxacin
  E. penicillin G
Explanation:

The correct answer is D. The patient is a nursing home resident with a residual neurologic deficit from a stroke that affects her speech. She is found with an altered mental status by the nursing home staff. On evaluation, she has a temperature, elevated WBC, and obvious infiltrate on CXR. She probably an aspiration pneumonia. She is predisposed to aspiration pneumonia due to her stroke, which has affected her speech and likely her ability to swallow. In addition, the CXR infiltrate is in the right lower lobe, which is the likely place for aspirated contents to fall due to anatomy of the bronchi. Finally, she has Gram-negative rods in her sputum. Elderly individuals in long-term care facilities tend to have colonization of the oropharynx with Gram-negative rod bacteria. Levofloxacin has excellent coverage of most pathogens causing aspiration pneumonia and is a preferred antibiotic treatment. If the patient had known Pseudomonas colonization, a ceftazidime or piperacillin may have been better alternative choices.

Cefuroxime (choice A) is the drug of choice for community-acquired pneumonia, but does not have enough Gram-negative coverage in regards to aspiration pneumonias.

Clindamycin (choice B) is another traditional choice for aspiration pneumonia to cover anaerobic organisms. However, without evidence of anaerobic infection such as lung abscess on CXR, necrotizing pneumonia, severe periodontal disease, or putrid sputum, clindamycin is not indicated.

Erythromycin (choice C) covers atypical organisms, not commonly associated with aspiration pneumonia.

Penicillin G (choice E) was the traditional choice for aspiration pneumonia to cover Gram-positive organisms, but recent findings show that Gram-negative organisms predominate in elderly nursing home patients, making this an incorrect choice.

 

A 53-year-old widowed woman comes to the office for a health maintenance examination. She is a new patient who recently moved to your city after her husband died in an office fire 6 months ago. She says that she has no complaints, except for a cough that she began to notice 4 months ago. She denies nasal discharge, “a tickle in the throat,” frequent throat clearing, heartburn and the sensation of regurgitation, fever, sputum production, cigarette smoking, illegal drug use, sexual activity, occupational exposures, and any other symptoms associated with a respiratory infection. She says that the cough is not seasonal or associated with wheezing. Her temperature is 37.0 C (98.6 F), blood pressure is 135/90 mm Hg, pulse is 70/min, and respirations are 14/min. Physical examination is unremarkable. The most appropriate next step is to

  A. order an electrocardiogram
  B. order an x-ray of the chest
  C. question her about medications
  D. refer her for fiberoptic bronchoscopy
  E. schedule her for pulmonary function tests
Explanation:

The correct answer is C. This patient has a chronic cough, which is usually considered chronic because it is lasting more than 3 weeks. It may be due to a variety of things. However, the important lesson in this question is that before you turn to diagnostic studies you need, to make sure that you have obtained a detailed history. The case history will provide the answer to almost every question that you will need to ask her, except what medications she takes. Since she is a new patient, you will need to find out if she is taking an ACE inhibitor, such as captopril or enalapril, which is a frequent cause of a chronic cough in hypertensive patients. They cause a cough in up to 20% of people taking them. The exact mechanism is unknown, but it is thought to somehow be related to bradykinin and substance P. The treatment for the cough is the discontinuation of the ACE inhibitor.

An electrocardiogram (choice A) is unnecessary at this time in this patient, complaining of a chronic cough. She is not complaining of chest pain and there is nothing in her history that suggests an arrhythmia. The most important next step, is to take a detailed history before you order diagnostic tests.

An x-ray of the chest (choice B) may be appropriate in the near future, but it is not the next step at this time. Before you order diagnostic studies, you need to make sure that you ask her any questions that might help you figure out the etiology of her cough. Asking her about medications is very important because ACE inhibitors cause a chronic cough in up to 20% of patients taking this medication.

A fiberoptic bronchoscopy (choice D) is used to obtain histologic and cytologic specimens and to visualize an endobronchial tumor. Before you turn to such a specialized study, you need to first obtain a detailed history. If the patient is not taking an ACE inhibitor, a chest x-ray should usually be performed, and if this is abnormal, sputum cytology, a high resolution CT scan, and fiberoptic bronchoscopy should be considered.

Pulmonary function tests (choice E) are used to assess airway hyperresponsiveness for patients in which you suspect asthma, and lung volumes and diffusion capacity in patients in which you suspect a diffuse interstitial lung disease. A detailed history is necessary before using any of these studies.

 

A 36-year-old man is brought to the emergency department because of shortness of breath and stridor. His shortness of breath has been progressive for the past few months but has worsened significantly in the past week. He was in a serious motor vehicle accident 14 months ago after which he was intubated and ventilated for nearly 3 months. His intensive care unit stay was complicated by ventilator-associated pneumonia. He was eventually weaned from the ventilator and extubated 7 months ago. Since that time, he has been convalescing well but his shortness of breath has become increasingly troublesome. In the last 2 or 3 days his wife has noticed the stridor. His temperature is 37.0 C (98.6F), blood pressure is 140/75 mm Hg, pulse is 72/min, and respirations are 24/min. His lungs are clear bilaterally, but inspiratory stridor is appreciated. A chest CT scan shows a tracheal stenosis of 7mm at the level of C6 vertebral body. The most appropriate management at this time is to

  A. admit the patient for observation
  B. give the patient bronchodilator therapy
  C. initiate antiinflammatory therapy
  D. intubate the patient immediately
  E. obtain a thoracic surgical consult
Explanation:

The correct answer is E. This patient has tracheal stenosis secondary to long-term intubation. This is a common complication of long-term intubation and is one of the main reasons that tracheostomy tubes are placed in patients that are in need of long-term mechanical ventilation. Stridor is caused by the inflow of air across a narrow obstruction in the airway. It signifies that some part of the trachea or main stem bronchi are quite narrow and portends an airway disaster if the disease process is allowed to progress. The CT scan demonstrates tracheal stenosis of moderate degree (3-8 mm is moderate) that will require surgical correction in the near term.

Simply admitting the patient for observation (choice A) fails to address management of a clearly abnormal airway. It is critical to act in cases such as this and triage the patient to the appropriate management as soon as possible.

The trachea is not responsive to bronchodilator therapy (choice B), only the distal bronchioles.

The inflammation and fibrosis that are responsible for the stenosis have already occurred and therefore, antiinflammatory therapy (choice C) will offer little benefit at this time.

There is no need to intubate the patient immediately (choice D). His ventilation and oxygenation are adequate and he does not appear to be in respiratory distress. His airway can in fact become much more narrow (1-2 mm) before any major respiratory distress is appreciated. In addition, since his stenosis is at the level of the cricoid cartilage (C6), an endotracheal tube will not be able to pass.

 

A 67-year-old man comes to the office complaining of a 12–month history of shortness of breath at rest and with mild exertion. He also reports wheezing on occasion that seems to correspond to the times when he is most short of breath. He has a cough that is persistent most of the year and is occasionally productive of scant sputum. He has a long smoking history of over 100 packs/ year, but has recently quit. His only other medical history is hypertension and hyperlipidemia for which he takes atenolol and simvastatin. A chest radiograph shows hyperinflation but clear lung fields and no evidence of a parenchymal or mediastinal mass. The most appropriate next step in management is to

  A. obtain spirometry
  B. order a chest CT scan
  C. prescribe albuterol inhalers
  D. prescribe corticosteroids
  E. prescribe home oxygen
Explanation:

The correct answer is A. This patient likely has COPD based upon his social history and symptoms. All patients who present complaining of shortness of breath, are over the age of 40, and have a smoking history or environmental exposure history to asbestos, beryllium, or dust, should have screening spirometry. Spirometry measures forced inspiratory and expiratory effort. The hallmark of COPD is decreased forced expiratory effort. The tests are noninvasive and can be performed in fully clothed patients.

A chest CT scan (choice B) is of minimal value in this case since the chest radiograph revealed no evidence of any abnormality except hyperinflation, which is to be expected in a case of COPD.

Therapeutic interventions such as albuterol inhalers (choice C), steroids (choice D), or home oxygen (choice E) are options to be considered once a diagnosis of COPD is established and its severity is quantified. Inhalers are excellent first-line drugs for patients that have reversible airflow obstruction demonstrated by full pulmonary function testing. Home oxygen is usually given for late-stage COPD in patients who have room air arterial oxygen pressures of less than 60 mm Hg. Steroids are very useful agents in acute exacerbations of COPD.

 

A 54-year-old African American woman comes to the office because of fatigue, anorexia, weight loss, and fever. The patient reports that over the past few months, these symptoms, which were new, have progressively worsened. She has a past medical history significant only for temporal arteritis for which she takes low-dose prednisone daily. Her physical examination is remarkable for a temperature of 37 C (98.6 F), blood pressure is 130/85 mm Hg, pulse is 80/min, and respirations are 20/min. Her lungs are clear, there are no murmurs and no cyanosis or clubbing of the digits. A chest radiograph discloses bilateral hilar lymphadenopathy. After obtaining a thorough occupational, environmental, and medication-use history, the most appropriate diagnostic step is to

  A. obtain the ratio of CD4/CD8 T-lymphocytes
  B. order serum ACE levels
  C. perform the Kveim-Siltzbach skin test
  D. schedule a mediastinal lymph node biopsy to obtain evidence of noncaseating granulomas
  E. schedule a skin biopsy to obtain evidence of noncaseating granulomas
Explanation:

The correct answer is E. The clinical manifestations of sarcoidosis can be widespread or may involve only one organ system at a time. Sarcoidosis is a systemic disorder of unknown cause that is characterized by its pathological hallmark, the noncaseating granuloma. Because the lungs and thoracic lymph nodes are almost always involved, most patients report acute or insidious respiratory problems, variably accompanied by symptoms affecting the skin, eyes, or other organs. Once the diagnosis is suspected by evidence on chest radiography, the diagnosis should be confirmed by biopsy to exclude infection or malignant conditions. In most centers, skin and transbronchial lung biopsies have supplanted biopsy of mediastinal lymph nodes (choice D) and the liver because of their high yield, greater specificity, and low morbidity.

There are no definitive diagnostic blood, skin, or radiologic imaging tests specific for the disorder. Therefore, a ratio of CD4/CD8 T-lymphocytes (choice A), or serum ACE levels (choice B), although often used, are not of diagnostic value because of their very low specificity.

The Kveim-Siltzbach skin test (choice C) in which spleen or lymph node homogenate from a patient with suspected disease is injected intradermally and later subjected to biopsy, is not widely available, not well standardized, and not approved for general use by the FDA.

 

A 73-year-old woman is found apneic and pulseless on the medical floor. She had been admitted to the hospital 2 days prior for management of her hypothyroidism. She was diagnosed with severe hypothyroidism and started on intravenous thyroid hormone replacement. Her nurse found the patient unresponsive in the bathroom. The patient was lying on the floor, blue, and without a palpable pulse. An emergency was called. The patient was unable to be intubated by either an anesthesiologist or a medical house officer present at the emergency. By the time you arrive, you are told that there were at least three attempts to secure the airway by laryngoscopy, none of which were successful. The patient has no pulse, she is not breathing, and is non-responsive. The most appropriate immediate intervention is to

  A. attempt a surgical airway
  B. attempt to re-intubate the patient with a new laryngoscope
  C. begin chest compressions and the ACLS algorithm for ventricular fibrillation
  D. call a surgeon for an emergent cricothyroidotomy
  E. deliver 100% oxygen by face mask
Explanation:

The correct answer is D. Although the details of the emergency airway algorithm are not required for either ACLS or ATLS certification, the basics of airway management are required for both. The first rule of resuscitation is “A” for airway. This patient has not had their airway secured so that they can be ventilated and oxygenated. A surgeon should be called to immediately attempt a surgical airway.

Trying to attempt a surgical airway (choice A) is not wise in a situation where there is surgical backup personnel available. These procedures are difficult and, if not done properly, will result in no chance of survival for a patient with a difficult airway.

Any attempt to re-intubate the patient with a new laryngoscope (choice B) should be undertaken only after a surgical airway has been called for. Three attempts at laryngoscopy by experienced operators is reasonable evidence that this approach will be unsuccessful at securing the airway rapidly.

To begin chest compressions and the ACLS algorithm for ventricular fibrillation (choice C) is incorrect for two reasons. First, the “A” airway has not been secured. Secondly, there is no evidence that this patient is in VF. In order to determine which ACLS algorithm to follow, a rhythm or must be determined.

To deliver 100% oxygen by face mask (choice E) is incorrect since there is no way to deliver the oxygen to the lungs without positive-pressure mask ventilation. Face mask oxygen is only useful for patients who are spontaneously breathing.

 

A 37-year-old man with a history of allergic rhinitis comes to the office with a 3-day history of fever and cough. He was in his usual state of health until 3 days ago when he developed a cough productive of yellow-green sputum and fevers to 38.3 C (101.8 F). The fevers have been accompanied by drenching sweats. He has been experiencing right sided pleuritic chest pain. He denies shortness of breath, abdominal pain, weakness, or numbness. He has not had any sick contacts and has no recent travel outside of the United States. He has a 15-pack year history of smoking but denies any alcohol use or injection drug use. His temperature is 38.5 C (101.3 F), blood pressure is 132/74 mm/Hg, pulse is 82/min, respirations are 14/min, and oxygen saturation is 96%. Physical examination shows crackles at the right base A complete blood count and biochemical profile are all within normal limits. A chest x-ray shows a right lower lobe infiltrate. The most appropriate next step in the management of this patient is to

  A. admit the patient to the hospital for intravenous ceftriaxone therapy
  B. obtain a CT scan of the chest
  C. request a pulmonary consultation for bronchoscopy
  D. treat the patient as an outpatient with oral azithromycin therapy
  E. treat the patient as an outpatient with oral ciprofloxacin therapy
Explanation:

The correct answer is D. The patient’s history, exam, and x-ray are all consistent with a diagnosis of community acquired pneumonia (CAP). CAP can be safely treated as an outpatient in most circumstances. Exceptions to this rule are when the patient has an underlying medical condition (cardiac disease, pulmonary disease, diabetes, HIV, cirrhosis, renal disease, or malignancy), advanced age, or presents with a severe pneumonia manifested by unstable vital signs or bilobar pneumonia. The appropriate treatment for this patient is either a macrolide antibiotic or an extended spectrum fluoroquinolone as they will cover typical and atypical organisms.

As discussed previously this patient does not require intravenous therapy or hospital admission (choice A) unless his condition deteriorates. In addition ceftriaxone would not be adequate coverage as it does not cover the atypical organisms.

In the management of routine cases of CAP, a CT scan (choice B) is not necessary. A CT scan might be appropriate in situations where there is a concern for malignancy, underlying pulmonary disease, or non-resolving pneumonia. None of these conditions are present in this case.

This patient does not require a bronchoscopy (choice C). Potential indications for bronchoscopy are when there is a concern for an obstructive lesion on imaging studies, recurrent lobar pneumonia, or significant hemoptysis.

Ciprofloxacin (choice E) is not the antibiotic of choice for CAP. It does not provide adequate coverage against streptococcal pneumonia, which is the most common cause of CAP.

 

 

A 16-year-old boy is admitted to the hospital for pneumonia. The patient reports that over the past 3 days he has had an increasing cough, productive of thick, green sputum and pleuritic chest pain. He has a history of cystic fibrosis and has been hospitalized for pneumonia 9 times in the past 3 years. He has never been intubated, but has required prolonged hospital stays at times in order to manage his infections. His medications include pancreatic enzymes and acetylcysteine nebulizers. The most appropriate management of this patient is to

  A. begin aggressive chest physiotherapy
  B. give him inhaled beta agonists
  C. enroll him in gene therapy trials
  D. evaluate him for lung transplantation
  E. obtain a sputum culture and await results for directed antibiotic therapy
Explanation:

The correct answer is A. Cystic fibrosis is a pulmonary/gastric disorder caused by mutation in a protein responsible for maintaining salt and water gradients across cell membranes. The clinical manifestations of the disease stem from the presence of thick, copious secretions in the airways and ducts of the pancreas. The pulmonary manifestations are frequent infection such as pneumonia and eventually bronchiectasis. In addition to antibiotics, aggressive chest physiotherapy to loosen and remove impacted secretions is critical to clearing hyper-acute infections.

Inhaled beta agonists (choice B) offer no benefit for these patients since they have no element of bronchoconstriction to their disease. All of the airway issues in these patients relates to the thick mucous plugs that they are unable to clear.

Despite some of the early successes in gene therapy for CF, early enrollment in gene therapy trials (choice C) is still not considered a standard of care and does not replace in any way the most basic management principles of caring for patients with CF which is antibiotics and chest physical therapy.

It is appropriate to begin evaluation for lung transplantation (choice D) at any time during the course of CF. However, such an evaluation does not in any way assist in managing the acute infection that the patient is currently suffering.

Most patients with CF have defined pathogenic flora such as pseudomonas. For this reason, a sputum culture for directed antibiotic therapy (choice E) to direct therapy is not critical and antibiotic coverage can be initiated prior to any definitive culture data being returned.

A 65-year-old man, his wife, and 38-year-old son have been your clinic patients for the last 15 years. In the evaluation of some mild hemoptysis of the 65-year-old man, a chest x-ray reveals a 4 cm right sided lung mass, hilar and mediastinal adenopathy, and several lytic lesions in his ribs and humerus. None of these findings were present on an x-ray performed 4 years earlier. He has a 50-pack year smoking history. When he returns to your office, you inform him that he likely has stage IV lung cancer and that you would like to refer him to an oncologist for further evaluation. He states that he wants no therapy whatsoever, and that he wants to keep this a secret from his family. The most appropriate response would be to

  A. call his son as soon as he leaves the office
  B. inform him that treatment will likely be curative and that he should really reconsider his decision
  C. investigate what it is that makes him feel uncomfortable in telling his family and provide counseling
  D. realize that he will likely “come to his senses” and give him a referral to the oncologist anyway
  E. tell him that he is probably just in denial and try to persuade him to tell his wife when he gets home
Explanation:

The correct answer is C. Patient confidentiality is one of the most important medical ethical issues facing physicians, and it certainly can pose dilemmas at times. This patient has just received horrible news and is likely just reacting without really thinking about the ramifications of his decision. However, there may be very important personal, social, or cultural reasons for his decision. It is important for you, as a physician, to explore these with him.

Calling his son (choice A) is inappropriate because it breaks confidentiality.

Although you will likely try to get the patient to reconsider his decision (choice B) telling him that therapy will likely be curative for stage IV lung cancer is not true. There is very little chance at a cure and palliative therapy is a much more reasonable expectation.

Giving him a referral to the oncologist because he is will “come to his senses” (choice D) is inappropriate. He obviously needs counseling, and the feelings as to why he does not want treatment and why he does not want his family to know, should be explored.

Although the patient may be in denial (choice E), patient confidentiality precludes you from unilaterally deciding to tell his wife. It is appropriate to try to understand the reasons why he does not want to tell his family, as opposed to trying to persuade him to tell his wife when he gets home.

 

An 8-year-old boy is brought to the office by his mother because of recurrent episodes of “shortness of breath” and wheezing. These episodes typically occur when he is playing in the park with friends or when he is in the house at night. The symptoms are worst in the springtime and when he is watching television with his mother’s boyfriend. The mother’s boyfriend, who happens to smoke cigarettes, has been spending more and more time at the house, trying to bond with the patient. Pulmonary function tests show that the peak expiratory flow and forced respiratory volume per second are reduced during an attack and are normal during symptom-free intervals. Skin testing shows that he is allergic to grass and tree pollen, dust mites, animal dander, and a variety of other allergens. Laboratory studies show:

The most appropriate next step is to

  A. administer immunotherapy against identified allergens
  B. advise him to avoid all exercise
  C. advise him to try to avoid respiratory irritants, especially cigarette smoke
  D. advise the patient’s mother to use a humidifier and air cleaners at home
  E. prescribe inhaled sodium cromoglycate, oral corticosteroids, and oral theophylline
Explanation:

The correct answer is C. This patient has asthma, and the most crucial step in the management of asthma is avoidance of the triggering factors, e.g., allergens. Unfortunately, it is difficult to avoid specific types of allergens, such as pollens. Specific measures to eliminate or reduce exposure to dust mites and animal dander at home lead to a reduced frequency of attacks and hospitalization rates. Regardless of the allergens involved, elimination of respiratory irritants, especially cigarette smoke, is of crucial importance. The bronchial tree of asthmatic patients is highly reactive to any form of chemical or physical irritation. Thus the avoidance of passive smoke is important. The mother should ask her boyfriend to go smoke outside alone if he needs to, but he should not be allowed to smoke in the house.

It is not practical to administer immunotherapy against identified allergens (choice A) in this case because he is allergic to multiple airborne allergens, and it seems like he is especially responsive to cigarette smoke. Immunotherapy is of some benefit when a single allergen is identified. The most important step is to try to reduce exposure to avoidable allergens (smoke).

Avoidance of all exercise (choice B) is not appropriate because even though exercise triggers asthmatic attacks in some patients, this does not seem to be his main trigger.

Humidifiers and air cleaners (choice D) at home is not the appropriate management. Humidifiers favor the growth of dust mites, and air cleaners have not been shown to be uniformly effective in getting rid of dust mites.

It is inappropriate to prescribe inhaled sodium cromoglycate, oral corticosteroids, and oral theophylline (choice E) for this patient because the fewest number of drugs at the lowest effective doses should be used. Typically, a one drug regimen (a bronchodilator or an inhaled corticosteroid) for mild to moderate asthma or two drugs for more severe cases is sufficient to control asthma exacerbations. Oral corticosteroids are indicated in cases of severe asthma and are therefore, not for this patient.

 

A 49-year-old man comes to the office for a health maintenance examination. He has had 5-7 episodes of blood-tinged sputum in the past month that he is very concerned about. He denies any other symptoms. He has been a patient of yours for 15 years and has been generally healthy. He does not have any chronic medical conditions. He is a superintendent of a large apartment building, gets regular exercise, eats a low-fat diet, and smokes 2 packs of cigarettes a day for the past 30 years. His temperature is 37.0 C (98.6 F), blood pressure is 130/80 mm Hg, pulse is 65/min, and respirations are 16/min. Physical examination is unremarkable. You order a chest x-ray, a complete blood count and coagulation profile, electrolytes, BUN and creatinine, a urinalysis, and send sputum samples for Gram, fungal, and acid-fast stains, cytology and schedule a follow-up visit in 2 weeks. He arrives for the appointment and you review the results with him, all which came back normal. He tells you that he has had increasing episodes of “blood in the sputum” and has even coughed up about 10mL blood over the past 2 weeks. Physical examination and vital signs are unchanged since the last visit. The most appropriate next step is to

  A. admit him to the hospital for immediate thoracic surgery
  B. order a chest x-ray
  C. schedule a bronchoscopy
  D. schedule a high-resolution CT scan
  E. reassure him that all of the tests were normal
Explanation:

The correct answer is C. This patient has non-massive hemoptysis, which is defined as less that 100 mL of expectorated blood over 24 hours. The work-up begins with history and physical examination. A chest x-ray and laboratory studies should be ordered initially. If the chest x-ray is normal and the patient has risk factors for cancer, like smoking 2 pack of cigarettes a day for the past 30 years, a bronchoscopy should be ordered to localize the bleeding site and look for an endobronchial mass. If none is found, a high resolution CT scan should be considered. If a mass is found on a study, referral to a thoracic surgeon is necessary.

It is inappropriate to admit him to the hospital for immediate thoracic surgery (choice A) at this time. He has non-massive hemoptysis and needs a bronchoscopy to be ordered to localize the bleeding site and look for an endobronchial mass. Surgery may be indicated in the future, but now, since he is hemodynamically stable, he needs further evaluation for his hemoptysis.

It is unnecessary to order another chest x-ray (choice B) at this time. Since his physical examination has not changed, it is unlikely that a chest x ray would have changed in 2 weeks. He has no symptoms that suggest pneumonia which may appear on a later x-ray. He requires a bronchoscopy to look for a mass that cannot be visualized on the x-ray.

A high-resolution CT scan (choice D) is usually only indicated after a chest x-ray and a bronchoscopy. An exception is when the chest x-ray suggests parenchymal disease, then the HCRT is usually performed before the bronchoscopy.

In this patient with hemoptysis and a significant smoking history, reassuring him that all of the tests were normal (choice E) is inappropriate. You need to find the cause of his symptoms by continuing with the diagnostic evaluation. A bronchoscopy is indicated at this time.

 

 

A 66-year-old man with diabetes mellitus comes to the emergency department because of a 4-day history of a cough with rusty-colored sputum. His wife died from breast cancer 2 months ago and his daughter, who lives 45 minutes away, comes to check on him a couple of times a week. He says that he normally passes his days watching talk shows on television, but lately he has felt so “crummy” that he has stayed in bed. His temperature is 38.7 C (101.6 F), blood pressure is 110/70 mm Hg, pulse is 100/min, and respirations are 31/min. His arterial PO2 on room air is 59%. Physical examination shows bronchial breath sounds, egophony, and dullness to percussion over the right upper lobe and the left lower lobe. A chest x-ray shows consolidation of the right upper lobe and the left lower lobe. Gram stain of his sputum sample shows neutrophils and lancet-shaped Gram-positive diplococci. The sensitivities are still pending. Other patients that you have seen lately have had penicillin-resistant strains of this disease. The next step should be to

  A. admit him to the hospital for intravenous vancomycin therapy
  B. have him call his daughter to come pick him up and take care of him
  C. order a bronchoscopy with a sampling of respiratory secretions
  D. send him home with erythromycin therapy
  E. send him home with cefuroxime therapy
Explanation:

The correct answer is A. This patient most likely has community-acquired pneumonia (Streptococcus pneumoniae), is acutely ill, and requires hospitalization. Criteria for hospitalization are ages >65, significant comorbidities, leukopenia, pneumonia caused by Staphylococcus, anaerobes, or Gram-negative bacilli, suppurative complications, failure of outpatient management, an inability to take oral medication, a respiratory rate >30/min, heart rate >140/min, hypotension, a PO2 <60 mm Hg, a change in mental status, and a poor social-support system. Each case needs to be evaluated on an individual basis, however, this patient has multiple risk factors, and should therefore be hospitalized. Vancomycin is given empirically in this case before the sensitivities return because he is severely ill, has diabetes, and there have been cases of penicillin-resistant strains. Therapy can be switched when the results return.

It is incorrect to have him call his daughter to come pick him up and take care of him (choice B) because he is very ill and should be hospitalized.

A bronchoscopy with a sampling of respiratory secretions (choice C) is necessary if he fails initial therapy or if he has a rapidly progressive downhill course. It is not indicated at this time.

It is inappropriate to send him home with erythromycin therapy (choice D) or cefuroxime therapy (choice E) because he should not be treated as an outpatient. This patient requires hospitalization based on his physical findings, comorbidity, and lack of social support (visits 2 times a week by his daughter is not enough).

 

You are the physician-on-call and are asked to see a 57-year-old woman who has been complaining of progressive shortness of breath over the past 2 days. The woman was admitted to the medical service 6 days ago after a fall and has been on bed rest for a nondisplaced pubic ramus fracture. She has been on deep vein thrombosis prophylaxis with subcutaneous heparin. Her past medical history is significant for type-II diabetes and dialysis-dependent renal failure secondary to diabetic nephropathy. She makes no urine at baseline. Her last dialysis run was 4 days ago, though she is usually dialyzed 3 times per week. She is complaining of shortness of breath but no chest pain. She is anxious-appearing, has a blood pressure of 160/105 mm Hg, respiratory rate of 30/min, and heart rate of 110/min. She has an oxygen saturation of 80% on room air and appears cyanotic. Physical examination reveals a jugular venous pressure of 10 cm and inspiratory crackles half way up from the bases upon auscultation of the lungs. An electrocardiogram shows a rate-related right bundle branch block but no ischemic changes. Chest x-ray obtained yesterday revealed interstitial edema and vascular redistribution to the apices. The patient’s husband has arrived from home and is very concerned about the recent events. In particular, he asks what has caused the current problem with her breathing. You suspect that a scheduling error may have delayed her last dialysis run and caused volume overload, but you are not completely sure. The most appropriate initial approach to take with the husband given that you have just met the patient for the first time is to

  A. acknowledge sympathetically that a mistake may have been made but will not likely cause permanent harm
  B. ask him to kindly wait until the hospital lawyer can be present before speaking to him about the matter
  C. explain that hospitals are complex institutions, making scheduling a difficult process at times
  D. explain that you are the physician on-call tonight and are not privy to all of the scheduling details, but that you will explore in detail whether a mistake has been made and that you will communicate these findings in a timely manner to both him and the patient
  E. speculate on how recent health maintenance organization’s cuts in reimbursement may have made it necessary to decrease the number of dialysis runs for each hospitalized patient
Explanation:

The correct answer is D. When you suspect a mistake has been made in the hospital, the first approach should always be to make an earnest effort to get all of the facts. This approach will immediately build a rapport with the husband as you become an advocate for the truth, readily demonstrate your interest in the patient’s specific case, and help identify process errors in the hospital.

While acknowledging that a mistake has been made (choice A) is often an important step, it usually does not go far enough because patients and their families are interested in why mistakes happen and whether they will happen again. It is simply unprofessional to not go further and explore all of the details of the mistake regardless of whether an injury actually occurs.

While seeking legal advice (choice B) may be an important step in dealing with this issue, the initial approach again should focus on a thorough investigation of the facts by the physician and relevant administration.

Philosophical approaches (choice C) are unsatisfactory because they seek to diminish any individual responsibility for a mistake happening and are unlikely to result in correcting a serious process error.

Speculating on cuts in insurance reimbursement (choice E) is inappropriate and unprofessional. The job of physicians is to care for their patients and be their advocates. This must occur regardless of the practice environment one finds themself in. Specifically, whether the patient population is wealthy and cash paying or whether they depend on insurance, medical decisions are based upon medical need and not insurance mandates. To acknowledge any differently is to commit medical malpractice.

A previously healthy 31-year-old woman comes to your office complaining of 1-day history of a cough and a fever. She reports that she was celebrating a job promotion 3 days prior and drank quite a bit of alcohol at a local bar. She had 2 episodes of vomiting that evening. She takes no regular medications and has only been using acetaminophen for fever suppression. Her temperature is 38.2 C (100.8 F). Her lungs have decreased breath sounds in the left base and right upper lobe. She has a cough that is productive of foul-smelling sputum. The remainder of her examination is unremarkable. The most appropriate management is to

  A. admit the patient to the hospital for clindamycin therapy
  B. admit the patient to the hospital for penicillin therapy
  C. admit the patient to the intensive care unit for levofloxacin therapy
  D. begin outpatient cefuroxime therapy
  E. begin outpatient erythromycin therapy
Explanation:

The correct answer is A. This is a patient who likely has pneumonia in the setting of likely aspiration. Since most pneumonia never have the etiologic agent identified, the treatment is empirical based upon patient locale at time of infection and presumed organisms based upon epidemiology. In this case, the presumed aspiration indicates that coverage for Gram-negative and anaerobic organisms is required. Clindamycin is a macrolide derivative that has activity against these agents. It is effective and is well-tolerated orally. Uncomplicated pneumonia such as community acquired or atypical infections rarely require hospitalization. For this patient with a likely anaerobic, purulent infection, a more monitored setting for therapy is required.

Penicillin (choice B) is an excellent choice for community acuquired pneumonia with the caveat that an increasing number of isolates of S. pneumonia are resistant. In some centers, this number is as high as 20%. However, penicillin has no activity against Gram-negative or anaerobic organisms.

Levofloxacin (choice C) is a fluoroquinolone that has broad activity against Gram-positive, Gram-negative, and some anaerobes. However, it does not have adequate coverage of anaerobic organisms to provide effective coverage for presumed aspiration. This patient has no objective findings that would warrant an ICU admission. Hemodynamic instability or respiratory distress requiring intubation would be classical reasons why patients with severe pneumonia may require an ICU stay.

Outpatient cefuroxime (choice D) is a second-generation cephalosporin that is standard outpatient therapy for community acquired pneumonia. It does not have the required broad Gram-negative (although it has some) coverage and it has no anaerobic coverage. This patient should however be hospitalized for observation during initial therapy.

Erythromycin (choice E) is a macrolide antibiotic that is also effective for both typical and atypical community acquired pneumonia but is only minimally useful in cases of aspiration pneumonia.

 

A previously healthy 21-year-old college student comes to the clinic because of a headache, sore throat, muscle aches, and a constant, irritating, dry cough for six days. He says that he is “never sick” and has only been to this clinic for his “immunizations”. He exercises regularly, does not smoke cigarettes, and has an “occasional beer on the weekends with buddies.” His temperature is 38.8 C (101.8 F), blood pressure is 120/80 mm Hg, pulse is 68/min, and respirations are 16/min. Scattered rhonchi are heard in the left lower lobe. A chest x-ray shows diffuse interstitial infiltrates in the left lower lobe. A single dose of erythromycin therapy is given in the clinic. The most appropriate next step in management is to

  A. admit him to the hospital and begin administration of erythromycin, intravenously
  B. admit him to the hospital and begin administration of trimethoprim-sulfamethoxazole, intravenously
  C. admit him to the hospital for a cold agglutinin test
  D. give him a prescription for erythromycin and send him home
  E. recommend aspirin, fluids, and rest at home
Explanation:

The correct answer is D. This patient most likely has Mycoplasma pneumonia, which is a common cause of pneumonia in young adults and is typically treated with oral erythromycin as an outpatient. It is characterized by a dry cough, headache, myalgia, malaise, and fever. Physical examination is usually unremarkable except for diffuse rhonchi or fine rales. A chest x-ray shows diffuse interstitial or reticulonodular infiltrates, typically in the lower lobes. Given the patient’s age, history, physical examination, and chest x-ray findings, it is reasonable to assume that he has a community-acquired pneumonia. This is most likely due to Mycoplasma pneumoniae and empiric antimicrobial therapy with erythromycin should be prescribed. In these patients, a microbial diagnosis (with a sputum culture, transtracheal aspiration, bronchoscopy, or a blood culture) is often impractical and unnecessary. A cold agglutinin response is often associated with Mycoplasma pneumoniae. However, it is nonspecific and detected in less than 50% of cases.

Admitting him to the hospital and beginning administration of erythromycin, intravenously (choice A) is incorrect because a patient with Mycoplasma pneumonia, which is what this patient most likely has, is usually treated as an outpatient. The criteria for hospitalization of patients with pneumonia are ages >65, significant comorbidity, leukopenia, pneumonia due to Staphylococcus aureus, Gram-negative bacilli or anaerobes, suppurative complications, failure of outpatient management, inability to take oral medication, respirations >30/min, heart rate >140/min, hypotension, hypoxia, or acute alteration of mental status. The patient in this case does not meet any of these criteria.

Admitting him to the hospital and beginning administration of trimethoprim-sulfamethoxazole intravenously (choice B) is the management for patients with severe Pneumocystis carinii pneumonia (PCP), which is characterized by shortness of breath, a dry cough, fever, night sweats, rales or rhonchi, and bilateral patchy alveolar infiltrates. This is a common cause of pneumonia in immunocompromised patients, especially those with HIV and AIDS. The patient in this case does not appear to be immunocompromised, and he is not short of breath, which makes the diagnosis of PCP unlikely.

Admitting him to the hospital for a cold agglutinin test (choice C) is inappropriate because even though this patient most likely has Mycoplasma pneumonia, it can be treated with erythromycin as an outpatient, and a cold agglutinin test can be performed as an outpatient. A cold agglutinin response is often associated with Mycoplasma pneumoniae. However, it is nonspecific and detected in less than 50% of cases.

Recommending aspirin, fluids, and rest at home (choice E) is inappropriate treatment for this patient who most likely has Mycoplasma pneumonia, which needs to be treated with an antibiotic such as erythromycin.

 

A 53-year-old woman who is a heavy smoker presents to the emergency department complaining of increasing shortness of breath for the past 3 days. She denies any history of asthma or coronary artery disease. Her temperature is 37.3 C (99.2 F), blood pressure is 150/90 mm Hg, heart rate is 110/min, and respiratory rate is 34/min. On examination, she is awake, alert, and oriented. Diffuse bilateral wheezes are heard on lung auscultation. Pulse oximetry measures 90% oxygen saturation on room air. An arterial blood gas is drawn and the results show:

A chest radiograph demonstrates bilateral, hyperinflated lungs with a flattened diaphragm. Sputum Gram stain shows a few polymorphonuclear cells, moderate number of epithelial cells, and a moderate number of Gram-positive cocci. She receives supplemental oxygen, albuterol nebulizer treatments, and steroids. Her symptoms improve and pulse oximetry now reads 93% saturation. The most appropriate next step is to

  A. add antibiotics to the treatment regimen
  B. do diffusion capacity testing by carbon monoxide
  C. intubate and begin mechanical ventilation
  D. obtain lung spirometry measurements
  E. start non-invasive positive pressure ventilation
Explanation:

The correct answer is A. The patient is a smoker who presented with progressive shortness of breath. Physical examination found diffuse wheezing and chest X-ray noted emphysema. In addition, she had an elevated pCO2 with acute respiratory acidosis and moderate hypoxia. These findings are consistent with an exacerbation of chronic obstructive lung disease. Such flares are treated with β2-agonists, anticholinergics, and steroids. In addition, antibiotics have also been shown to improve clinical outcome, and so they are part of the treatment regimen for chronic obstructive lung disease flares.

Diffusion capacity (choice B) for this patient will likely be low given her emphysema and is an important measurement for diagnosis, but it is not required in the acute management of this condition.

The patient has a normal mental status and is able to protect her airway. Her symptoms and oxygenation also improve with treatment. Thus, there is no current indication for intubation (choice C). Intubation is required if the patient has severe CO2 retention and/or hypoxia refractory to medical therapy. Intubation is also indicated if her condition is refractory to non-invasive ventilation, if she has severe acid-base disturbances, or if there is any change in her mental status that would compromise the airway.

Lung spirometry (choice D) will aid in the diagnosis of her disease but is not useful in management of her clinical course.

Non-invasive positive pressure ventilation (choice E) is indicated in patients with severe chronic obstructive pulmonary disease that is refractory to medical therapy. It is also useful in patients with increasing respiratory fatigue. A patient must be able to initiate breathing and tolerate the breathing mask. This patient has a normal mental status and her symptoms improve with treatment. Thus, she currently does not require any assistance in ventilation.

 

 

 

A 68-year-old woman comes to the office for a health maintenance examination. She has had 5-7 episodes of “expectorated blood” in the past month that she describes as a “bit concerning.” She denies any other symptoms. She has been a patient of yours for 20 years and you have treated her for various “colds and flus” in the past, but she does not have any chronic medical conditions. She is a retired schoolteacher, gets regular exercise, and smokes a pack of cigarettes a day. She and her husband have become “world travelers” since both of their retirements. Her last mammogram, Pap smear, and colonoscopy were 1 year ago, and were normal, as they have always been. Her temperature is 37.0 C (98.6 F), blood pressure is 130/80 mm Hg, pulse is 65/min, and respirations are 16/min. Physical examination is unremarkable. The most appropriate next step is to

  A. obtain a sputum sample by transtracheal aspiration for cytology
  B. order a chest x-ray
  C. schedule fiberoptic bronchoscopy
  D. schedule a high-resolution CT scan
  E. reassure her that it is most likely nothing but to come back if she continues to have “expectorated blood”
Explanation:

The correct answer is B. This patient comes in for a routine examination but tells you something that could possibly be serious—that she has nonmassive (less than 100mL) hemoptysis (“expectorated blood”). Since she is a smoker and travels very frequently, you should not ignore this symptom. Since it is likely that the blood-streaked sputum is from the respiratory tract, a chest x-ray is the first diagnostic procedure that should be ordered.

Obtaining a sputum sample (choice A) by transtracheal aspiration is not indicated at this time because it is too invasive. Expectorated sputum should first try to be obtained. Blood in the sputum may occur in cases of bronchitis, pneumonia, bronchiectasis, a lung abscess, or an endobronchial tumor. Gram, fungal, and acid-fast stains will help diagnose an infectious cause, while cytology may be helpful to diagnose a tumor.

Fiberoptic bronchoscopy (choice C) is part of the evaluation of a patient with hemoptysis, but it is typically performed after a chest x-ray. It is the next step if a chest x-ray shows a mass, if the chest x-ray is normal and there are major risk factors for cancer, or if the chest x-ray is normal and there are no risk factors for cancer, but there is a recurrence of hemoptysis after weeks to months of observation.

A high-resolution CT scan (choice D) is usually only indicated after a chest x-ray is performed. If the chest x-ray shows a mass and a bronchoscopy fails to suggest a specific diagnosis, a HRCT is ordered. Also, if a chest x-ray shows parenchymal disease, a HRCT may be indicated for further evaluation.

It is inappropriate to reassure her that it is most likely nothing but to come back if she continues to have blood-streaked sputum (choice E) because hemoptysis can be the sign of serious disease, especially because she is a smoker and a “world traveler.” Even though she came to the office for a routine physical examination, a chest x-ray should be ordered at this time. Keep in mind that a chest x-ray is not part of a routine physical examination of an asymptomatic smoker.

 

 

A 66-year-old woman with a history of small-cell carcinoma presents to the emergency department with stridor and shortness of breath. She was diagnosed with cancer 2 years ago and underwent a resection of her left upper lobe 1 year ago. Six months ago, she presented with a recurrence of her tumor causing right-sided postobstructive pneumonia and bronchial constriction. She underwent chemotherapy and radiation therapy at that time. Over the past week, she has had an increasing cough and shortness of breath at rest and, in the last day, stridor. The most important diagnostic test at this time is a

  A. chest radiograph
  B. CT scan of the chest
  C. flexible bronchoscopy
  D. pleural drainage
  E. transtracheal biopsy
Explanation:

The correct answer is B. This patient has a history of aggressive lung cancer with recurrence that presented with bronchial obstruction. Her symptoms at present, cough, shortness of breath, and stridor, suggest that again the patient has a bronchial obstruction. Stridor is caused by the inflow of air across a narrow obstruction in the airway. It signifies that some part of the trachea or main stem bronchi are quite narrow and portends an airway disaster if the disease process is allowed to progress. This patient requires a CT scan to delineate the extent of her lung malignancy and to assess the extent of her airway obstruction.

A chest radiograph (choice A) will offer no critical information for management of this patient. The resolution of the image is such that it will allow visualization of tumor recurrence and of any infiltration such as a pneumonia, but offers no details of the airways or mediastinum; the structures likely to be involved in this disease process.

A flexible bronchoscopy (choice C) is a tool that allows relatively easy visualization of the airways. It is however not useful for defining the extent to which an extrinsic mass impinges upon the airway. Flexible bronchoscopy is often used to ensure that a tumor has not invaded the bronchial structure prior to performing a lung resection of a patient with cancer of the lung.

Since there is no evidence as of yet, that there is a pleural effusion, pleural drainage (choice D) is not indicated.

A transtracheal biopsy (choice E) is used as a minimally invasive method to sample lung parenchyma when a diagnosis of a lung mass is needed. In this case, the patient has a clear diagnosis so there is no need for a tissue sample.

 

A 2-year old boy is brought to the emergency department at 2:00 a.m. because of episodic coughing “fits”. He was well until one day earlier when he developed rhinorrhea and a temperature of 38.3 C (101 F). He has no prior history of respiratory illnesses, and no one else in the family is ill. On arrival to the hospital, he is coughing in a rapid, “barking” fashion, but appears otherwise well. His temperature is 38.2 C (100.8 F), heart rate is 120/min, respiratory rate is 50/min, and oxygen saturation is 96% on room air with coughing. Physical examination shows clear lungs, a normal cardiac and abdominal examination, and no rash. He continues to have repeated, episodic coughing with inspiratory stridor at rest. A frontal radiograph of the chest at this time is most likely to reveal

  A. cardiomegaly with pulmonary venous congestion
  B. left lower lobe infiltrate
  C. pneumothorax
  D. subglottic swelling
  E. thumb-shaped epiglottis
Explanation:

The correct answer is D. This child’s clinical picture is most consistent with infectious croup (laryngotracheobronchitis), commonly caused by parainfluenza viruses. The prodrome of upper respiratory illness with fever followed by a spasmodic, barking cough, is typical of this diagnosis. The classic sign on chest x-ray is the subglottic swelling ( “steeple sign,”), which is the result of viral-induced swelling of the subglottic tissue. This swelling can cause an incomplete airway obstruction, leading to stridor either at rest or during crying. Stridor at rest is an indication for treatment with steroids to decrease inflammation.

Dyspnea in a child can be the result of congenital heart disease and resulting congestive heart failure, manifested as cardiomegaly and pulmonary venous congestion on x-ray (choice A), but given the absence of prior symptoms, the normal oxygen saturation, and the lack of rales on lung examination, this is unlikely.

Fever and cough in a child also raises the possibility of pneumonia as a diagnosis. A lobar pneumonia (choice B) would be a surprising x-ray finding in this case given the child’s clear lung fields on examination and the presence of stridor, which indicates upper airway involvement.

Pneumothorax (choice C) can be a cause of sudden dyspnea in a child, especially in tall, male adolescents who may be predisposed to spontaneous pneumothoraces. This child’s prodrome and stridor on examination make this diagnosis unlikely.

Prior to the introduction of the vaccine against Haemophilus influenzae type b, epiglottitis was a common and much-feared diagnosis in pediatrics. Infection of the epiglottis by this organism led to rapidly progressive airway obstruction and potentially death. A lateral neck radiograph would demonstrate a thumb-shaped epiglottis (choice E) in these cases. This sign is not seen on frontal views of the chest.

 

A 29-year-old man is admitted to the hospital with fever and cough. The symptoms began roughly 1-month prior and have been intermittent. He states that his cough is often productive of thick secretions and that, despite normal food intake, he has lost about 10 pounds in the past month. He is a volunteer at a local hospital and has received no special health care personnel vaccinations or screening tests. On examination, the patient appears somewhat thin, tired, and is coughing intermittently. His temperature is 38.0 C (100.4 F) and respirations are 16/min. He has patchy bilateral rhonchi over all lung fields. Prior to initiating therapy for this condition, the laboratory test required to confirm the suspected diagnosis is a

  A. chest radiograph
  B. sputum acid-fast stain
  C. sputum culture
  D. sputum Gram stain
  E. tuberculin skin test
Explanation:

The correct answer is B. The patient likely has tuberculosis. Virtually all M. tuberculosis is transmitted by airborne particles that are 1 to 5 µm in diameter. The symptoms of tuberculosis are protean and nonspecific and can be classified as either systemic or organ-specific. Classic systemic symptoms include fever, night sweats, anorexia, weight loss, and weakness. However, since tuberculosis is associated with other illnesses that have similar symptoms, this lack of specificity can result in a delayed diagnosis or even a misdiagnosis. Organ-specific symptoms of pulmonary tuberculosis include cough, pleuritic pain, and hemoptysis. The requirement for diagnosis is the presence of the organism that appears by acid-fast staining in a sputum sample.

In patients with primary tuberculosis, chest radiographs (choice A) often show infiltrates in the middle or lower lung zones, with ipsilateral hilar adenopathy. These findings are non-specific and are not used for confirmation of the diagnosis.

A sputum culture (choice C) is not useful in this case since the organism responsible for TB is fastidious and is difficult to culture, and certainly does not grow rapidly.

The organism responsible for TB does not stain with traditional Gram stain dyes (choice D) and therefore requires special staining such as acid-fast in order to detect it.

Although it is imperfect, the gold standard for diagnosing latent tuberculosis infection remains the intradermal injection (choice E) of purified protein derivative (5 TU) into the volar or dorsal surface of the forearm (Mantoux method). The test has no role in the diagnosis of active infection.

 

A 68-year-old man comes to the clinic because of progressive dyspnea on exertion (DOE) and shortness of breath over the last 7 months. He denies chest pain, orthopnea, or paroxysmal nocturnal dyspnea. His past medical history is significant only for mild osteoarthritis and an episode of pneumonia 20 years ago. His temperature is 37 C (98.6 F), blood pressure is 128/76 mm Hg, pulse is 98/min, respirations are 18/min, and oxygen saturation is 98%. His lungs are clear to auscultation and his heart is slightly tachycardic with no murmurs, rubs, or gallops. His abdomen is soft, nontender, with normal bowel sounds. His extremities have no edema. Rectal examination shows brown guaiac-positive stool. An electrocardiogram shows sinus tachycardia with a single PVC. Chest x-ray shows minimal scarring in the right lower lobe. Laboratory studies show a hematocrit of 27%, hemoglobin of 9.1 g/dL, platelets of 298,000mm3 , MCV 78 mm3, sodium of 139 mEq/l, potassium of 4.1 mEq/l, blood urea nitrogen of 16 mg/dL, and creatinine 0.9 mg/dL. The most appropriate next step in the patient’s management is a

  A. cardiac stress test to rule out 3 vessel coronary artery disease
  B. colonoscopy to rule out colon cancer
  C. high resolution CT scan (HRCT) to rule out pulmonary fibrosis
  D. iron pills and follow up in 3 months
  E. ventilation-perfusion (V/Q) lung scan to rule out chronic pulmonary emboli
Explanation:

The correct answer is B. It is important to remember that anemia can present with dyspnea on exertion and a complete blood count should always be part of this work up. This elderly patient has an iron deficiency anemia with hemoccult positive stool. A GI malignancy needs to be ruled out and colon cancer is the most likely etiology in this patient population. Therefore, a colonaoscopy is imperative.

Although ischemic cardiomyopathy or silent ischemia can certainly present with progressive dyspnea on exertion, there are other things in the patient’s history to suggest the cause of his symptoms. A cardiac workup (choice A) at this time is not the most pressing issue.

HRCT (choice C) is a very good test to evaluate for pulmonary fibrosis. It is likely that the CXR would show more abnormalities. The minimal scarring mentioned at the right lower lobe is likely from his prior pneumonia mentioned in the past medical history, and is not causing the patient any symptoms.

The patient is presenting with a microcytic anemia. The most likely cause is iron deficiency. However, it must always be remembered that the finding of a microcytic anemia should always prompt the immediate search for an underlying cause. So although the patient will likely require iron supplementation (choice D), follow up in 3 months with no other diagnostic test is not appropriate.

A V/Q scan (choice E) can rule out chronic pulmonary emboli, which is certainly a cause of progressive DOE, but as mentioned previously, there are other things in the patient’s history to suggest the cause of his symptoms.

 

A 49-year-old woman comes to the office because of difficulty breathing, fevers reaching 40 C (104 F), and a productive cough with blood tinged sputum. She was recently diagnosed with ductal carcinoma of the breast and underwent a radical mastectomy with four rounds of adjuvant chemotherapy with vinblastine and doxorubicin. Her last infusion of chemotherapy was 5 days ago. A chest radiograph shows focal infiltrates in both lungs. Laboratory studies show:

You admit her to the hospital and start her on vancomycin and ceftazidime. Over the next 3 days, her clinical status continues to worsen. Blood cultures are negative. A bronchoscopy is performed and biopsy samples are obtained. The biopsy specimen shows septated, branching hyphae that are locally invading tissue. The most appropriate pharmacotherapy at this time is

  A. amphotericin B, intravenously
  B. fluconazole, intravenously
  C. fluconazole, orally
  D. itraconazole, intravenously
  E. rifampin plus isoniazid, orally
Explanation:

The correct answer is A. This is a case of pulmonary aspergillosis in a neutropenic patient after she underwent chemotherapy. The treatment of choice in neutropenic or immunosuppressed patients is amphotericin B intravenously.

Fluconazole (choices B and C) and itraconazole (choice D) are antifungal agents that have limited activity or no activity against aspergillus infections. Also, she should get intravenous therapy at this time.

Rifampin plus isoniazid (choice E) is a combination used to treat mycobacterial infections.

 

A 52-year-old woman is admitted to the hospital with shortness of breath, a productive cough with “yellowish sputum,” fevers, and chills. She has a medical history significant for non-insulin dependent diabetes and depression for which she takes glyburide and sertraline. She has an allergy to penicillin, to which she gets severe hives. Her temperature is 38 C (100.4 F), blood pressure is 123/67 mmHg, pulse is 102/min, and respirations are 25/min. Her oxygen saturation on room air is 96%. Physical examination shows decreased breath sounds over the lower right lung field with dullness to percussion. A chest radiograph shows consolidation in her right lower lobe. The most appropriate next step is to

  A. obtain a sputum sample for Gram stain and culture
  B. obtain a surgery consult for an open lung biopsy
  C. order a CT scan of the chest
  D. order a ventilation-perfusion scan
  E. send her for bronchoalveolar lavage
Explanation:

The correct answer is A. By history, exam, and chest radiography, this patient has pneumonia. In someone with a pneumonic process, it is advisable to send sputum for Gram stain and culture prior to the initiation of antibiotics in order to ensure that the choice of antibiotic agent will be appropriate. Empiric antibiotics are often started anyway, but it is good to know if the organism that is present is covered by the chosen antibiotic.

An open lung biopsy (choice B) is not used in the routine diagnosis of a lobar pneumonia. It is instead used in the diagnosis of more complex pulmonary pathologies such as the idiopathic interstitial pneumonias that are not amenable to diagnosis using routine methods.

A chest CT (choice C) will not add any additional information at this juncture since a chest radiograph is sufficient to confirm the presence of pneumonia in this patient.

A ventilation-perfusion scan (choice D) is used to diagnose the presence of a pulmonary embolus, which is not a part of the differential diagnosis with this patient. This type of scan is often uninterpretable in the setting of a lobar pneumonia.

A bronchoalveolar lavage (choice E) is not used in the diagnosis of a lobar pneumonia associated with a productive cough.

 

A 59-year-old man with diabetes mellitus is admitted to the hospital because of a 4-day history of a cough with yellowish-brown sputum, fever, and chills. He tells you that he is allergic to penicillin, to which he gets severe urticaria. He reports a positive sick contact with a friend who was recently admitted with pneumonia. His temperature is 39 C (102.2 F), blood pressure is 123/67 mm Hg, pulse is 107/min, and respirations are 25/min. His room air oxygen saturation is 96%. Physical examination shows decreased breath sounds over his left lower lung field associated with dullness to percussion. A chest radiograph shows consolidation of the left lower lobe. A sputum Gram stain and culture shows Gram-positive cocci sensitive to cephalosporins and quinolones. A suitable treatment regimen given this data is to start the patient on

  A. ampicillin-sulbactam, intravenously
  B. cefuroxime, intravenously, if a penicillin skin test is negative
  C. cefuroxime, intravenously, if a penicillin skin test is positive
  D. gentamicin, intravenously
  E. vancomycin, orally
Explanation:

The correct answer is B. It is known that patients who are allergic to penicillins have a higher rate of allergy to cephalosporins. While the true rate of cephalosporin allergy under these circumstances is controversial, most authorities believe it is about 7-8%. However, it has been established that if a patient with a penicillin allergy has a negative penicillin skin test, he/she is at no increased risk of an allergy to a cephalosporin.

Starting the patient on a regimen of intravenous ampicillin-sulbactam (choice A) is not appropriate given the patient’s history of allergy to penicillin.

Starting the patient on intravenous cefuroxime if a penicillin skin test is positive (choice C) is inappropriate since a positive skin test to penicillin indicates an elevated risk of a coexistent allergy to cephalosporins. Under these conditions, based upon the available data, a quinolone agent would be reasonable.

Starting the patient on intravenous gentamicin (choice D) is not appropriate since the laboratory data specifies an organism sensitive to cephalosporins.

Starting the patient on oral vancomycin (choice E) is inappropriate since oral vancomycin has poor systemic absorption and as a rule, it is also best to avoid broad- spectrum antibiotics if a more specific agent (such as a cephalosporin with this patient) is available.

 

 

A 24-year-old woman comes to the office because of a cough with “yellowish sputum production” for the past 2 days. She states that the cough has been keeping her up at night and it is bothering her co-workers. They insisted that she “go get medicine” so that she does not “infect the entire office.” She has no history of respiratory disease. Her temperature is 37 C (98.6 F), blood pressure is 110/80 mm Hg, pulse is 70/min, and respirations are 18/min. Physical examination is normal. The most appropriate next step in management is to

  A. admit her to the hospital for medical management
  B. obtain a sputum culture
  C. order a chest x-ray
  D. prescribe erythromycin, orally
  E. send her home with no medications
Explanation:

The correct answer is E. This patient most likely has acute bronchitis. Acute bronchitis in a healthy patient with no other medical conditions is often due to viral infection that is usually self-limited. Given that this patient has only had 2 days of symptoms, an antibiotic is not necessary and is inappropriate. If the symptoms persist for longer than 1 week, a macrolide antibiotic may be given. A chest x-ray and a sputum culture are not indicated.

Admission to the hospital for medical management (choice A) is inappropriate for a healthy patient with acute bronchitis.

A sputum culture (choice B) is used to identify organisms, but should only be used in an elderly patients with chronic disease that fail antibiotic therapy. This patient’s acute bronchitis is most likely due to a self-limited viral infection.

A chest x-ray (choice C) has no role in the diagnosis of acute bronchitis in a healthy patient.

Send the patient home with antibiotic therapy (choice D) is appropriate management for acute bronchitis in an elderly patient with chronic disease. A macrolide (erythromycin, azithromycin, clarithromycin) is the treatment of choice. It is not part of the initial treatment in a previously healthy patient.

 

A 102-year-old man is brought to the clinic from his apartment by a neighbor because of a cough productive of green sputum. He has a past medical history of a stroke 10 years ago with residual left arm weakness. The patient is widowed and lives alone. He takes no medications. Vital signs are: temperature 38.8 C (101.8 F), blood pressure 100/50 mm Hg, pulse 110/min. On physical examination, the patient has rhonchi in the right lower lung field. A chest x-ray reveals an infiltrate in the right lower and middle lobes. The patient has a score of 30/30 on a mini mental status examination. The patient states that he has “lived a good life and now wants to go home”. He refuses any intravenous medications and will only take pills. After discussing that the pneumonia could potentially be fatal without intravenous antibiotic treatment, the patient continues to ask to be sent home. The neighbor insists that the patient should be admitted to the hospital. The next step in the care of this patient is to

  A. admit the patient to hospital care with intravenous antibiotics
  B. admit the patient to hospital care without intravenous antibiotics
  C. admit the patient to psychiatric ward with antibiotics
  D. discharge the patient home with oral antibiotics
  E. discuss the case with the hospital attorney
  F. initiate antidepressant therapy and administer intravenous antibiotics
Explanation:

The correct answer is D. The principle of individual autonomy is central to medicine. The only exceptions are if the patient has no clearly expressed wishes documented and is unable to make a decision or if the patient is deemed incompetent to make a decision. This patient is clearly competent to refuse treatment despite the wishes of his physician or neighbor. It is important to make the consequences of his decision clear to the patient before discharge and to tell him that he should return immediately should he change his mind.

Hospital care with or without intravenous antibiotics (choices A and B) is inappropriate as this patient is refusing hospital care.

This patient is clearly competent to refuse treatment, so he should not be admitted to the psychiatry ward (choice C) or given antidepressant therapy (choice F).

Consultation with an attorney (choice E) is not necessary as the patient is clearly competent to refuse medical care.

           

You are called to see a 67-year-old woman with severe COPD who was intubated on the medicine floor 30 minutes earlier because of respiratory distress. Her temperature is 37.0 C (98.6 F), blood pressure is 90/60 mm Hg, pulse is 133/min and regular, and respiratory rate is 24/min. Her ABG reveals the following: pH 7.21, PaCO2 80 mm Hg, PaO2 69 mm Hg, and her oxygen saturation is 91%. Her current ventilator settings are assist control (AC), respiratory rate (RR) of 12/min, tidal volume (TV) of 500 ml, fraction of inspired oxygen (FiO2) is 50%, positive end expiratory pressures (PEEP) of 5 mm/Hg. The most appropriate intervention at this time is to

  A. decrease set respiratory rate
  B. increase the fraction of inspired oxygen
  C. increase PEEP
  D. increase tidal volume
  E. make no adjustments at this time
Explanation:

The correct answer is D. The key to this question is to recognize that this patient remains acidotic and hypercarbic despite intubation. In order to “blow off” more carbon dioxide and normalize the blood pH, you must increase minute ventilation (MV). If you remember that MV=TV X RR, you can quickly recognize that the two ways to decrease carbon dioxide is to increase the RR or increase the TV.

Decreasing RR (choice A) will increase carbon dioxide because we are decreasing minute ventilation as described above. This will result in worsening acidosis. In addition, our set respiratory rate is lower than the patient’s actual rate. If we decreased the respiratory rate, our patient would continue to breathe at a high rate and receive the same tidal volume (thereby not even changing the MV).

Increasing FiO2 (choice B) will not be beneficial. First, the patient already has a paO2 of greater than 60 mm Hg. If you remember the hemoglobin binding curve, you should remember oxygen saturations remain above 90% for paO2>60 mm Hg. Therefore, our goal is to keep paO2>60 mm Hg for most patients. Second, maintaining patients on greater than 60% oxygen may have some associated lung toxicity. Therefore, if possible, keeping the inspired oxygen less than 60% is an important part of management. Of course, if high levels of inspired oxygen are necessary, you should not hesitate to use them. Please remember that patients on ventilators need not (and should not) have oxygen saturations of 100% or paO2 much greater than 60 mm Hg.

PEEP (choice C) is the use of positive airway pressures at the end of expiration. PEEP is useful in hypoxic respiratory failure such as ARDS or cardiogenic pulmonary edema. Low levels of PEEP can be used in COPD to keep airways open. Our patient’s oxygenation is acceptable so increasing PEEP would not be beneficial now. In addition, our patients blood pressure is only borderline acceptable. Increasing PEEP will decrease venous return to the heart and might lead to further reductions in blood pressures. High levels of PEEP might also predispose patients to barotrauma which is a form of ventilator induced lung damage.

Making no adjustments (choice E) is not acceptable. This patient is severely acidotic and hypercarbic and should not be left in this state.

 

A 71-year-old retired clerk comes to the clinic because of hoarseness. He says that he had a “chest cold” 4 weeks ago with congestion and a sore throat. He attributed the hoarseness of his voice to the cold, but has been worried for the last 2 weeks because the hoarseness has been persistent. He is a diabetic taking insulin twice a day. He smokes 1-2 packs of cigarettes a day and stopped drinking alcohol about 2 years ago. His only other medication is an aspirin a day. He has had no prior surgeries and no other medical problems. Laryngoscopy performed in the office demonstrates paralysis of the left vocal cord but no evidence of masses or inflammation in the larynx. The most likely etiology of this patient’s vocal cord paralysis is

  A. diabetic neuropathy
  B. laryngeal carcinoma
  C. lung carcinoma
  D. pneumonia
  E. stroke
Explanation:

The correct answer is C. Hoarseness of the voice is a symptom of vocal cord paralysis. Vocal cord paralysis is a result of injury to the ipsilateral recurrent laryngeal nerve, which is a branch of the vagus nerve supplying all the intrinsic muscles of the larynx. Unilateral vocal cord paralysis is much more common than bilateral disease by a factor of 3 to 1 and the left side is more commonly affected than the right side due to the longer course of the left recurrent laryngeal nerve. The most common cause of unilateral vocal cord paralysis is lung cancer. This patient’s history of smoking puts him at high risk for developing lung cancer.

Laryngeal carcinoma (choice A) can also present very early with symptoms of hoarseness. The most common laryngeal cancer is a glottic tumor arising from the true vocal cords. 90% of these tumors are squamous cell carcinomas. This is not a likely cause of this patient’s hoarseness because the laryngoscopy did not demonstrate a laryngeal mass or lesion. Laryngoscopy very readily detects lesions and is a good means for obtaining biopsies for further cytologic studies.

Diabetic neuropathy (choice B) is not the cause of this patient’s symptoms. Most diabetics over time will develop some neurologic involvement due to peripheral vascular disease. Classic signs and symptoms of diabetic neuropathy include numbness and loss of sensation in the distal extremities, particularly in the feet. Diabetic neuropathy is not known to cause damage to the recurrent laryngeal nerve and is thus not a cause of vocal cord paralysis.

Pneumonia (choice D) is not a cause of vocal cord paralysis. Primary lung cancers that occur in the lung apices are the most common cause of injury to the recurrent laryngeal nerve and thus vocal cord paralysis. Pneumonia is an infectious process that does not cause mass effect or damage to the nerve.

Stroke (choice E) can be a cause of vocal cord paralysis when an infarct occurs in the nucleus ambiguus of the medulla. The nucleus ambiguus is the origin of the vagus nerve in the brain. However, if a stroke were to occur in the medulla, there would be many more deficits than a focal vocal cord injury. This patient is presenting with isolated unilateral vocal cord paralysis which means the injury occurred somewhere after the left recurrent laryngeal nerve branched off the vagus nerve.

 

 

A 69-year-old woman is brought to the emergency department by ambulance after calling 911 because of severe shortness of breath. By the time the ambulance arrived, the patient was having pronounced difficulty breathing. She was placed on 100% oxygen and was brought to the emergency department. Her medical-alert bracelet disclosed that she is allergic to penicillins and that she has COPD. Her temperature is 37.0 C (98.6 F), blood pressure is 190/85 mm Hg, pulse is 112/min, respirations are 34/min, and oxygen saturation on 100% oxygen is 82%. She appears to be in moderate to severe respiratory distress. The decision is made to intubate the patient. The most appropriate management of this patient after intubation is to

  A. allow the patient to breath spontaneously
  B. deliver rapid, shallow breaths to increase oxygen delivery
  C. deliver slow breaths with 6-10cc/kg tidal volumes
  D. deliver slow breaths with 15-20cc/kg tidal volumes
Explanation:

The correct answer is C. This patient has severe COPD and is suffering from a COPD flare. She is in severe respiratory distress, hypoxic, and almost certainly hypercarbic. Once her airway is secured, 100% oxygen should be delivered via slow breaths (8-12 per minute) with smaller tidal volumes than most patients would receive. Larger tidal volumes are associated with a risk of volu-trauma such as bursting of a bleb or bulla. Slow breaths minimize the risk of air trapping and so called “intrinsic-PEEP” or “auto-PEEP.”

Allowing the patient to breathe spontaneously (choice A) is inappropriate. She was unable to ventilate and oxygenate well prior to insertion of the endotracheal tube. Now, with it present, and the increased resistance to breathing it confers, there is minimal chance that her success with ventilation would be any greater.

Using rapid, shallow breaths to increase oxygen delivery (choice B) will almost certainly be harmful. These patients require prolonged expiratory time to exhale completely. Each rapid breath essentially “stacks” on top of an unfinished exhalation and therefore augments the end expiratory volume (therefore pressure) in the lungs. This is known as auto-PEEP and can become so severe that it impedes venous return to the heart and precipitates in circulatory collapse.

As discussed above, slow breaths with 15-20cc/kg tidal volumes (choice D) are associated with a significant risk of volu-trauma in these patients so traditional tidal volumes are not to be used.

           

S3 RESP

S3 RESP